You are on page 1of 277

Nguyn Hu in

OLYMPIC TON NM 2000 (Tp 1)


t
NH XUT BN GIO DC

.c

52 THI V LI GII

v n m a t h .c o m

Li ni u
th gi lnh lamdethi.sty ti bin son mt s ton thi Olympic, m
A cc hc tr ca ti lm bi tp khi hc tp L TEX. ph v cc bn ham hc ton ti thu thp v gom li thnh cc sch in t, cc bn c th tham

c thi gian sa li, mong cc bn thng cm.

Rt nhiu on v mi hc TeX nn cu trc v b tr cn xu, ti khng H Ni, ngy 2 thng 1 nm 2010 Nguyn Hu in

51 89/176-05 GD-05

ti liu ting Vit v ch ny, ti c xem qua v ngi dch l chuyn v ngnh Ton ph thng. Bn c th tham kho li trong [1].

.c

Rt nhiu bi ton dch khng c chun, nhiu im khng hon ton chnh xc vy mong bn c t ngm ngh v tm hiu ly. Nhng y l ngun

kho. Mi tp ti s gom khong 50 bi vi li gii. Tp ny c s ng gp ca Bi Th Anh, V Th Hng Hnh, Cao Th Mai Len, T Xun Ha, Nguyn Th Loan, Nguyn Th Qu Su, Nguyn Th nh, Nguyn ngc Long.

M s: 8I092M5

Mc lc
Li ni u . . . . . . . . . . . . . . . . . . . . . . . . . . . . . . . . . . . . . . . . . . . . Mc lc . . . . . . . . . . . . . . . . . . . . . . . . . . . . . . . . . . . . . . . . . . . . . . . Chng 1. thi olympic Belarus . . . . . . . . . . . . . . . . . . . . . . . . . Chng 2. thi olympic Bungari . . . . . . . . . . . . . . . . . . . . . . . . Chng 3. thi olympic Canada . . . . . . . . . . . . . . . . . . . . . . . . Chng 4. thi olympic Trung Quc . . . . . . . . . . . . . . . . . . . . Chng 5. thi olympic Tip khc . . . . . . . . . . . . . . . . . . . . . . Chng 6. thi olympic Estonia . . . . . . . . . . . . . . . . . . . . . . . . 3 4 5 16 29 32 41 46 51 56 59

Ti liu tham kho. . . . . . . . . . . . . . . . . . . . . . . . . . . . . . . . . . . .

Chng 8. thi olympic India . . . . . . . . . . . . . . . . . . . . . . . . . . .

Chng 7. thi olympic Hungary . . . . . . . . . . . . . . . . . . . . . . .

.c

Chng 1 thi olympic Belarus


1.1. Hai ng cho AC v BD ca t gic ABCD ct nhau M. ng phn gic ca gc ACD ct tia BA K . Nu MA.MC +MA.CD = MB.MD th BKC = CDB. Li gii: Gi N l giao im ca CK v BD. p dng nh l v ng

MC.DN MD = (MA.MC) MN MN Hay MA.MC = MB.MN V M nm trong t gic ABCN, theo nh l v phng tch ca mt im th A, B, C v N cng nm trn mt ng trn. T : KBD = ABN = ACN = NCD = KCD Suy ra K, B, C v D cng nm trn mt ng trn. Do c BKC = CDB.

khi c MB.MD = MA.MC + MA

CD =

Hay

MC CD = ND MN

MC.DN MN

.c

phn gic cho tam gic MCD

Nguyn Hu in, HKHTN H Ni 1.2. Trong mt tam gic u xp n.(n+1) ng xu v n ng xu xp dc theo 2 mi cnh v lun c mt ng xu ngn( trn cng) Mt php th v xc nh bi cp ng xu v tm A, B v lt mi ng xu nm trn on thng AB. Hy xc nh nhng yu t ban u- gi tr ca n v v tr ban u ca ng xu c mt tri m t c th khin cho tt c ng xu hin ra mt tri sau mt s php th v. Li gii: V mi php th v ca 0 hoc 2 ng xu trong 1 gc, tnh chn l ca s ngn trong gc l c bo ton. Nu ng xu cho thy mt tri khng trong mt gc, lun c 3 ng xu trong gc l ngn, th lun c s ngn trong gc l l. Nh vy, s lun c 3 gc khng ng thi cho mt tri ca ng xu. Ngc li, nu trong mt gc c ng xu mt tri, chng ta s chng

Li gii: Li gii th nht im P nm trn ng trn tm A bn knh AC. ng trn ct ng CH, MH v PH ti D, N v Q. V MA = MC, A = khi v ch 3 khi tam gic ACM u. Ngha l khi v ch khi MN. iu khng nh PM l phn gic gc HP B khi v ch khi MN Tht vy, AH l ng cao thuc y ca tam gic cn ACD, H l trung im ca CD, CD l mt dy cung ca ng trn , theo nh l v phng tch ca mt im c P H.HQ = CH.HD = CH 2 .

1.3. Cho tam gic ABC v gc C = gi M l trung im ca cnh huyn 2 AB, H l chn ng cao CH v P l im trong tam gic sao cho AP = AC. Hy chng minh rng PM l phn gic BP H khi v ch khi A = . 3

cc ng xu s cho thy mt tri. Do yu t ban u cn la chn l c ng xu c mt tri nm trong 1 gc.

.c

Trong mi (n - 1) ng ngang c hai hoc nhiu ng xu. Ta chn hai ng xu k nhau v lt tri tt c cc ng xu trong ng ny. Tt c

minh rng th lm cho tt c cc ng xu hin mt tri Ta hng tam gic sao cho gc i n vi mt cnh nm ngang;

thi olympic Belarus

V v CH l ng cao thuc cnh huyn ca tam gic vung ABC nn CH 2 = AH.HB. Vy P H.HQ = AH.HB. Do H l giao im ca AB v P Q nn t gic AP BQ ni tip. Xt trn ng trn QAB = QAN = 2.QP N = 2.HP N Nh vy HP B = QP B = QAB = 2.HP N V v N l giao im ca HB v PN phn gic ca gc HP B. Do PM l phn gic ca gc HP B khi v ch khi M N

Li gii th hai Khng mt tnh tng qut ta gi s AC = 1. Dng h trc ta vung gc vi C lm gc, A c ta (0; 1) cn B c ta (n; 0) vi n > 0 Nu n = 1 th MN v PM khng th l phn gic ca gc BPH. Trong trng hp ny c A = = iu ny tri vi kt qu mong i 4 3

khong cch ta tnh c

S dng h thc trong tam gic vung AHC v ACB c AH = b = CA, c = AB; t

BP = n2 + 1 HP
b2 c

m nm gia 0 v 2. Ta ca M l ( n ; 1 ) v v CH c i n v H 2 2 n2 n trn AB, nn H cn tm c ta ( n2 +1 ; n2 +1 ) . S dng cng thc tnh

.c

Chnh iu cho php ta chn n = 1 S dng cng thc khong cch c AP = AC khi v ch khi P c ta dng ( m.(2 m); m) v

vi

MB = MH

c 2 b2 2

c 2

c2 n2 + 1 = 2 c2 + 2.b2 n 1

Theo nh l ng phn gic PM l phn gic BP H khi v ch khi BP MB = M H . Gii phng trnh tng ng ta tnh c nghim khi v ch HP khi n2 (n2 3) = 0 v n > 0 nn PM l phn gic gc BPH khi v ch khi n = 3, ngha l khi v ch khi A = . 3 1.4. C tn ti mt hm f : N N sao cho f (f (n 1)) = f (n + 1) f (n)

8 vi mi n 2?

Nguyn Hu in, HKHTN H Ni

Li gii: Khi khng nh tn ti mt hm nh vy s dn n mu thun. T phng trnh f(n -1) - f(n) > 0 vi n 2 iu ny khng nh hm f tng nghim ngt vi n 2 nh vy, f (n) f (2) + (n 2) n 1 vi n 2 Chng ta c th lm nn f (n) nh sau: T phng

trnh cho mc nhin c f (f (n 1)) < f (n + 1) vi n 2 hay l f (f (n)) < f (n + 2) vi n 1. V f l hm tng vi nhng bin ln hn 1, cho f (n) = 1 hoc f (n) < n + 2. T n 1 n 2. Ly n nguyn bt k b hn 4 Mt mt f (n) 2 v (n 1) 2 (n + 2) (n 1) = 3 f (n) n + 1 vi mi

f (f (n 1)) = f (n 1) f (n)

Ngoi ra, m s cnh bn trn mi mt v tng ca F cao nht t c l mt s t nht l 3m + 4(F - m). Mi cnh bn c m 2 ln theo cch ny, suy ra 2E 3m + 4(F m) Qua biu thc Euler cho biu phng, F + V E = 2. Kt hp vi 2E = 4V ng thc ny l 2E = 4F 8 Nh vy Hay m 4F 8 = 2E 3m + 4(F m)

8 s cn bng t c nu v ch nu mi mt ca a din l

tam gic hoc t gic, mt hnh tm mt u c nhng hnh nh vy.

ca 4 cnh bn, chng ta m nh 2 ln theo cch ny. Nh vy 2E = 4V

mi nh. t F, E v V l s mt, cnh bn v nh ca a din. vi mi cnh bn, m hai nh v cc u mt. V mi nh l u mt

Li gii: Ly 1 a din vi m mt tam gic v 4 cnh bn gp nhau ti

1.5. Trong mt a din li vi m mt tam gic( cn cc mt khc vi hnh dng khc), Ta lun c 4 cnh bn gp ti mi nh. Tm gi tr nh nht c th ca m.

.c

Nh vy,(n 3) 3 v bt k n > 4 l iu v l. iu ny cho thy khng nh ban u l khng ng v cho kt lun khng tn ti mt hm nh th.

thi olympic Belarus

Suy ra m = 8 l gi tr t c. 1 1.6. a) Chng minh rng n 3 > n3 vi tt c s nguyn dng n, trong {x} c hiu l phn s ca x. c b) C t ti bt bin c > 1 m n 3 > n3 cho mi n nguyn dng?. c Li gii: iu kin n 3 > n3 c th p dng vi n = 1 nu ch c nu 1> 3 v d 3 > c. t 1 c < 3 l mt bt bin vi mi c c n, n 3 = n 3 n 3 ln hn n3 nu ch nu n 3 n3 . V c < 3 < 3n2 , hai v ca bt dng thc ny l dng, chng ta ch c th bnh phng mi v m khng lm i du bt ng thc. 3n2 2c + c2 > n 3 2 3n
2

(*)

tt c n. Nh vy (*) l bt ng thc p dng cho tt c n c2 Tuy nhin, nu c> 1 th 3n2 2c + 3n2 3n2 2 cho tt c cc s ln n tha mn. Nh vy, tn ti mt s n vi iu kin thm l 3n2 2 phi l s chnh phng. Vi n ny (*) v ng thc b) l sai Vy cu tr li i vi phn b) l "khng".

1.7. Cho tp hp M = {1, 2, . . . , 40}. Tm gi tr n nh nht(n: s nguyn) m c th chia tp M thnh n tp con ri nhau m bt k a, b v

D rng chng minh rng m2 3n2 = mk 3n2 . Nh vy, do ng k+1 k+1 k thc 3n2 2 = m2 p dng vi k = 0, p dng vi tt c k 1. Do k k

n1 , n2 , . . . l mt chui tng n dn n 3n2 2 l mt s chnh phng vi n ty . 2 c2 n 3 cho Nu c = 1 nh vy 3n2 2c + 3n2 > 3n2 2c = 3n2 2c

Xc nh (m0 , n0 ) = (1, 1) v (mk+1 , nk+1) = (2mk + 3nk , mk , 2nk ) vi k 1.

l s chnh phng. Chng ta yu cu rng s cn bng p dng ty vi n.

nh hn hoc bng 3n2 ti a 3n vi cn bng nu v ch nu 3n2 2

.c

Vi mi n, 3n2 1 khng phi l s chnh phng v khng c s chnh phng no ng d 2 mod3, v 3n2 cng khng phi l s chnh phng. 3n2 s nguyn ln nht m bnh phng ca n Nh vy, n 3 =

10

Nguyn Hu in, HKHTN H Ni 0(khng nht thit khc bit) nm trong cng tp con, tha mn a = b+c. Li gii: Gi s cho mc ch mu thun, c th chia tp M thnh 3 tp X, Y v Z. Khng mt tnh tng qut ta gi s rng |X| |Y | |Z|

suy ra 2|X|-1 40 hay |X| 20. Ta c 3 |X| |X| + |Y | + |Z| = 40, suy ra |X| 14. Ta c |X| . |Y | 1 |X| (40 |X|) i trong X.Y. Tng 2 ca cc s trong mi cp i nh nht l 2 v ln nht l 80 c c 1 79 gi tr c th xy ra v 21 |X| 14 v hm t 2 + (40 t) l hm lm trn on 21 t 14 chng ta c 1 1 min 2 .14(26), 2 .21(19) =182 > 2.79.

cho cc x1 , x2 , . . . , x|X| l cc thnh phn ca X c sp xp theo th t tng dn. Nhng s ny, bn cnh cc chnh lch xi x1 vi i = 2, 3, |X|, phi l nhng thnh phn khc bit ca M. C 2|X|-1 s nh vy,

1 2

|X| .(40 |X|)

b = (x3 x2 ), a = (x3 x1 ) ta c a = b+ c v a, b, c Z, suy ra mu thun Nh vy gi s ban u ca chng ta sai v khng th phn chia M thnh 3 tp tha mn yu cu t ra. By gi c th chng minh chia M thnh 4 tp vi yu cu t ra. Nu ai {0, 1, 2} vi tt c i N v nu ai = 0 vi n > N, sau t (. . . a2 a1 a0 ) v (aN aN 1 . . . a0 ) c hiu l s nguyn n ai 3i ng i=0

nhin gi tr nguyn m c th vit di dng (. . . a2 a1 a0 ) theo mt cch chnh xc vi c s 3. Ta t s nguyn m = (. . . a2 a1 a0 ) vo tng A0 , A1 . . . nu a0 = 1 thay m vo A0 . Mt khc v a = 0, ai1 = 0 vi mt s i1 , bi v ch hu hn ai = 0, ai2 = 0, vi mt vi i2 > i1 , tip

1 j < k 3 gi tr xk xj nm trong M v khng th nm trong X v mt khc xj + (xk xj ) = xk . Tng t yj yk Y vi 1 j < k 3 /

Nh vy, 3 s chnh lch bng nhau x2 x1 = y2 y1 , x3 x2 = y3 y2 , x3 x1 = y3 y1 nm trong M \ X Y = Z. t a = (x2 x1 ),

l khng th xy ra v cp (xi , y i) l khc bit. Nh vy, chng ta c th gi s, khng lm mt tnh tng qut rng x1 < x2 < x3 vi

.c

Theo nguyn tc Pigeonhole tn ti 3 cp i (x1 , y1), (x2 , y2),(x3 , y3 ) X.Y vi (x1 + y1 ) = (x2 + y2 ) = (x3 + y3 ) Nu bt c xi no bng nhau th tng ng yi s bng nhau, iu ny

thi olympic Belarus

11

n al = 0, al+1 = 0, vi mt vi l. Chn l nh nht vi thuc tnh ny v thay m tai Al+1 = 0. Nu m1 , m2 A1 v c s 3 biu din m1 + m2 c nhng n v s 2 nh vy m1 + m2 A1 . Nu m1 + m2 Al vi mt s l>1, nh vy: /
l l l l

0 11 1 < m1 , m2 < 1 00 0 suy ra 0 22 2 < m1 + m2 < 2 00 0 nu m1 + m2 = (. . . a3 a2 a1 a0 ), th al = 1 suy ra m1 + m2 Al /

1s mi 1 (3k 1) A1 c s 3 trong tt c cc s khc trong S c 2 1 s 0 trong 3k1 v tr m mi s nguyn trong S nm chnh xc 1

By gi, t k > 1 l mt s nguyn dng v t S = 1, 2, . . . , 1 (3k 1) biu thc c s 3 ca 1 (3k 1) bao gm tt c 2 2

t c nh yu cu.

1.8. Mt s nguyn dng gi l monotonic(u) nu nhng ch s ca n trong c s 10, c t tri sang phi theo th t khng gim dn. Chng minh rng vi mi n N tn ti n ch s monotonic l s bnh phng Li gii: Bt k s c 1 ch s l bnh phng( v d 1,4 hoc 9) l monotonic chng minh yu cu bi vi n = 1 Chng ta gi s n > 1 Nu n l s l, vit n = 2k - 1 cho mt s nguyn k 2 t xk = (10k + 2)/6 = 166 67
k2

nhng S(n)=44.

ln hn gii hn tn ti vi tt c S(n) khng c dng tng qut no bit n, gii hn nh hn c tm thy trong gii php cho n =1,2,3

s phn chia no {1, 2, . . . , k} vo n tp hp. S nguyn ny c biu th bi S(n) v c gi l nth Schur number. Mc d nh hn v

Ghi ch Vi n, k N v s phn chia ca {1, 2, . . . , k} trong n cp 3 (a,b,c) sao cho a + b = c v a,b,c trong cng tp gi l Schur triple vi mi n N tn ti k nguyn ti a m khng c Schur triplec cho

.c

trong cc tp hp A0 , A1 , . . . , Ak1 . Nh vy, S c th l mt phn trong k tp hp. A0 S, A1 S, . . . , Ak1 S m a = b = c vi bt c a, b v c trong cng mt tp. Suy ra kt qu k = 4 cho thy n = 4 l c th

12 Nh vy

Nguyn Hu in, HKHTN H Ni

x2 = (10k 2k + 4.10k + 4)/36 = k Quan st thy: 2 77 7 + 7 9


2k2 102k 36 72 = 102k2 .( 36 + 28 ) 36

102k 10k 1 + + 36 9 9

(*)

= 2.102k2 + 7 .102k2 = 9

Vy v phi ca ng thc (*) bng: 2 77 7 + 7 + 11 1 + 1 = 9 9 2 77 7 9 88 8 l mt s c n ch s monotonic l s bnh phng. Nu n l s chn, vit n = 2k vi s nguyn k 1 v yk = (102 k + 2)/3 = 4 33 3
k1 k2 k1 2k2 k

Nh vy

2k

.c

2 yk = (102k + 4.10k + 4)/9 =

102k 10k 4 +4 + 9 9 9 1 4 = 1 11 1 + + 4 44 4 + = 11 1 6 55 5 9 9
k k k1

cc chui dch chuyn, lun phin nhau dch v dng (i) v dng (ii) cho mt vi cp vect ban u. a) C th t c cp ((1, 0), (2, 1)) trong tr chi vi cp ban u ((1, 0), (0, 1)) nu s dch chuyn u tin l dng (i)? b) Tm tt c cc cp ((a, b), (c, d)) c th t c trong tr chi vi cp ban u ((1, 0), (0, 1)) trong dch chuyn u tin l mt trong hai dng trn ? Li gii: t || || biu th cho chiu di ca vect v t |z| biu z z th cho gi tr tuyt i cu s thc z

1.9. Cho cp ( , ) vect trong mt my bay mt dch chuyn gm chn r s mt s nguyn khc khng k v sau thay i ( , ) thnh hoc (i) r s + 2k , ) hoc (ii) ( , + 2k ) Tr chi gm ly mt hu hn (r s s r s r

mt s gm n ch s monotonic chnh phng (pcm)

thi olympic Belarus

13

a) t ( , ) l cp vect m v c th thay i qua tr choi r s r s , l vect nh l || || > || || quan st thy rng nu x y x y Nh vy: ||2k || || || > 2|| || || || = || || y x y y y Sau ln dch chuyn u tin dng (i) ta c = (1, 2) v = (0, 1) r s || > || || p dng kt qu trn vi > v cho s k = 0 m || r s x s > chng ta c th thy trong dch chuyn tip theo (dng(ii)) y r di ca khng thay i trong tng cao hn || ||, p dng kt r s r > v > chng ta c th thy trong dch qu trn ln na vi x r y s || + 2k || x y

chuyn tip theo (dng(i)) di ca khng thay i trong s r ||, tip tc nh vy ta thy rng || || v || || khng tng vt qua || s r s b) Chng ta thay i tr chi bng cch khng yu cu dch chuyn lun phin gia dng (i) v (ii) v bng cch cho php s la chn k = 0. ng nhin bt c cp no c th t c theo quy nh ban u

D dng kim tra gi tr ca z xy v tnh chn l ca x v y l khng thay i theo bt c dc chuyn no trong tr chi. Trong mt tr chi m bt u vi ((, x), (y, z)) = ((1, 0), (0, 1)), ta phi lun lun c z xy = 1 v b c 0(mod2). Bi v x v y lun lun chn v z khng i mod4, ta phi c z 1(mod4) thng qua tr chi.

chi vi cp ban u ((1, 0), (0, 1)) phi tha mn v by gi ta chng minh iu ngc li. Gi s, nhm mc ch thy c s mu thun rng c nhng cp i ((a, b), (c, d)) tha mn iu kin a ra. t ((e, f ), (g, h)) l cp m

Gi 1 cp ((a, b), (c, d)) tha mn khi ad - bc = 1, a d 1(mod4) v b c 0(mod2) trn ta thy rng bt c cp i t c trong tr

u v c cng 1 cp. ((, x), (y, z)) i din cp ca nhng vect vi , x, y v z thay i qua tr chi.

quy nh mi vi k = 0 v kt hp bt c dch chuyn mi no ca cng dng vo 1 dch chuyn ta t c chui dch chuyn theo quy lut ban

phi t c theo nhng quy nh mi ny. iu ngc li ng v bng cch loi bt c dch chuyn no theo nhng

.c

bao gi gim. Bi v sau ln dch chuyn u tin vect u tin c di hn 1, s khng bao gi t c ((1, 0), (2, 1))

14

Nguyn Hu in, HKHTN H Ni ti thiu ha |ac| Nu g = 0 th eh = 1 + fg = 1 bi v e h 1(mod4) e = h = 1,

((e, f ), (g, h)) dn ti mu thun Nh vyg = 0 by gi g l s chn e l s l, |e| > |g| hoc |e| < |g| ta c e 2k0 g nm trong on (-|e|, |e|) cho k0 {1, 1}. Thc hin dng (i) dch chuyn n ((e, f ), (g, h)) vi k = k0 th t c mt cp mong mun khc ((e , f ), (g, h)). Bi v |e| < |e| v g = 0, chng ta c |eg| < |eg|. Nh vy, bng khi nim ti thiu ((e, f ), (g, h)) cp i mi c th t c t ((1, 0), (1, 0)) vi mt dy dch chuyn S no . Nh vy, chng ta t c ((e, f ), (g, h)) t ((1, 0), (0, 1)) bng cch p dng trc tin dch chuyn trong S ti ((1, 0), (0, 1)) sau p dng

nu f = 0 cp ny chc chn c c. Mt khc, bng cch dch chuyn dang (i) vi k = f chng c th thay i dng ((1, 0), (0, 1)) thnh dng 2

Vi tt c cc dng s thc a, b, c Li gii: Qua chng minh khng cn bng ca Holder (a + x


3

1.10.Chng minh:

(a + b + c)3 a3 b3 c3 + + x y z 3(x + y + z)

th kt lun rng bt c cp i t c u thc s tha mn. iu ny hon ton c chng minh.

Mt chng minh tng t nu |e| < |g| , khi chng ta thay la chn r0 vi g 2k0 e (|g|, |g|) v thc hin dng dch chuyn (ii). Nh vy trong tt c cc trng hp chng ta c s mu thun. Hay chng ta c

.c

thm dch chuyn dng (i) vi k = k0 . Nh vy cp i cc tiu t c dn n mu thun.

m
1

b3 y

1 c3 3 ) (1 z

+ 1 + 1) 3 (x + y + z) 3 (a + b + c)

ly tha 3 c 2 v v chia c 2 cho 3(x + y + z) ta c pcm 1.11.Gi P l giao im ca hai ng cho AC v BD ca t gic li ABCD trong AB = AC = BD. Gi O v I l circumcenter v tm ni tip ca 3 phn gic ca tam gic ABP. Chng minh rng nu O = I th ng thng OI v CD vung gc.

thi olympic Belarus

15

Li gii: u tin ta chng minh mt lun rt hu ch XY v UV , t X v Y l chn gc vung ca X v Y, ni ng thng UV. S dng khong cch trc tip, XY UV nu ch nu UX - XV = UY - YV v UX + XV =UV = UY + YV, php tnh trn t c nu ch nu UX 2 X V 2 = UY 2 Y V 2 , hoc UX 2 XV 2 = UY 2 Y V 2 .

Nh vy n tha mn ng thc DO 2 CO 2 = DI 2 CI 2 . t AB= AC=BD = p, PC = a v PD = b nh vy AP = p - a v BP = p b. t R l bn knh ng trn ngoi tip tam gic ABP. ta c pb = DP.DB = DO 2 R2 ngoi ra pa = CO 2 R2 . Nh vy DO 2 CO 2 = p(b a), v tam gic ADB l cn vi BA =

rng.

= p(b a) = P O 2 CO 2 (pcm)

DI 2 CI 2 = AI 2 BI 2 = AT 2 BT 2 = (AT + BT )(AT BT )

.c

vi cnh AB. Nh vy BT = (p + a - b)/2 v IT vung gc vi AB, AI 2 BI 2 = AT 2 BT 2 . t cc tham s li vi nhau chng ta thy

BD v I nm trn ng phn gic ca gc ABD, ID = IA ngoi ra IB = IC t T l im tip xc ca vng trn ni tip tam gic ABC

Chng 2 thi olympic Bungari


2.12.Mt ng thng l i qua trc tm ca tam gic nhn ABC. CMR cc

Ta cn chng minh: A1 P ,B1 Q,C1 R ng quy V hai ng thng AC v BC khng song song, nn hai ng thng B1 Q v A1 P khng song song. Ly S l giao im ca A1 P v B1 Q V SA1 C+SB1 C=P A1 C+QB1 C=P HC+QHC= nn t gic SA1 CB1 l im hi t ng trn Do , S l giao im ca ng thng B1 Q v ng trn w. Tng t, hai ng thng B1 Q v C1 R khng song song v giao im ca chng cng chnh l giao im ca B1 Q v ng trn w. Do vy, cc ng thng A1 P , B1 Q, C1 R ng quy ti mi im nm trn ng trn ngoi tip tam gic ABC

thng BC, CA, AB. Khi , A1 ,B1 ,C1 nm trn ng trn ngoi tip w ca tam gic ABC (Ch :A1 CB = BCH = HAB = A1 AB =)

xng vi ng thng AB v ta c th gi s R nm trn tia BA. LyA1 , B1 , C1 , tng ng l cc im i xng vi H qua cc ng

qut chng ta gi s l ct AC v BC ti P v Q. Nu l AB, ly R l im ty trn ng thng i xng vi l qua ng thng AB. Nu l khng song song vi AB th ly R l giao im ca ng thng l i

.c

Li gii: Gi H l trc tm ca tam gic ABC. V tam gic ABC nhn nn trc tm H nm trong tam gic ABC. Khng mt tnh tng

ng thng i xng vi l qua cc cnh ca tam gic ng quy.

thi olympic Bungari

17

2.13.C 2000 qu cu trng trong mt chic hp. Bn ngoi chic hp cng c cc qu cu trng, xanh v vi s lng khng hn ch. Trong mi ln thay i , chng ta c th thay i 2 qu cu trong hp bi 1 hoc 2 qu cu theo cch sau: 2 qu trng bi 1 qu xanh, 2 qu bi 1 qu xanh, 2 qu xanh bi 1 qu trng v 1 qu , 1 qu trng v 1 qu xanh, bi 1 qu hoc 1 qu xanh v 1 qu bi 1 qu trng. (a) Sau mt s hu hn ln thc hin nh trn cn li 3 qu cu trong hp. CMR c t nht 1 qu xanh trong 3 qu cu cn li. (b) Liu c th xy ra sau mt s hu hn ln thc hin nh trn trong hp cn li ng mt qu cu. cu , v -1 cho mi qu cu xanh. Ta c th kim tra li rng cc php thay th cho khng lm thay th cc gi tr ca cc qu cu Li gii: Ta gn gc gi tr i cho mi qu cu trng, i cho mi qu

BC tng ng ti M v N. ng thng t tip xc vi cung nh MN, t giao vi NC v MC tng ng ti P v Q. Gi T l giao im ca hai ng thng AP v BQ. (a) Chng minh T thuc MN . (b) CM: Tng din tch cc tam gic AT Q v BT P t gi tr nh nht khi t AB Li gii: (a) Hnh lc gic suy bin AMQP NP c ngoi tip bi cc ng trn ni tip tam gic ABC. Theo nh l Brianchon, cc ng cho AD, MN , QB l ng quy. Do , T thuc MN. Chng ta c th s dng cch gii s cp hn. Gi R v S tng ng l

2.14.ng trn ni tip tam gic cn ABC tip xuc vi cc cnh AC v

xanh, -1 cho mi qu cu hoc trng.

Hn na, v khng c qu no c gi tr 1 nn trong hp phi cha t nht hai qu cu. Do , khng th xy ra trng hp trong hp cn li 1 qu( chng minh (a), chng ta c th gn gi tr 1 cho mi qu

cc gi tr ca chng s l : i, mu thun. Do , nu trong hp cn li ba qu, th phi c t nht 1 qu mu xanh, (a) c chng minh.

.c

trong hp. Tch cc gi tr ca cc qu cu ban u l i2000 = 1. Nu trong hp cn li ba qu cu khng c qu no mu xanh th tch

18

Nguyn Hu in, HKHTN H Ni cc tip im ca ng trn ni tip vi cc cnh AB v P Q : Gi T1 , T2 tng ng ca tam giac ABC l cc giao im ca BQ vi MN v SR. V QMN = P NM =
MN 2

nn ta c: sin QMN = sinP NM = sinBNM .

p dng nh l hm s sin trong tam gic cho cc tam gic MQT1 v NBT1 .
QT1 = sin QM N = sin BN M = BT1 . QM BN sin QT1 M sin BT1 N QT1 MQ QT2 SQ hay BT1 = BN . Tng t: BT2 = BR .

QT1 Theo tnh cht ca tip tuyn, BN= BR v QM= QS. Do : BT1 = QT2 . V T1 v T2 u thuc BQ nn ta phi c T1 T2 . Do , BQ,MN , BT2

SR ng quy Mt cch tng t, ta chng minh c AP , MN , SR ng quy. T

cho tam gic CP Q ta c :


2. sin
p+q . cos pq 2 2

trn ni tip cu tam gic ABC, do I l tm ng trn bng tip ca tam gic CP Q. Do , P C+CQ+QP = 2.CM khng i. t CP Q=p v CQP =q. Th p + q= khng i. p dng nh l hm s sin

tip. f t gi tr nh nht khi P Q t gi tr nh nht. Gi I l tm ng

= AB.(AB + P Q). sin Trong : AQ + P B = AP + QP v t gic ABCD c ng trn ni

.c

V tam gic ABC cn, MN AB, suy ra [ABT ] l hng s. Do , f t gi tr nh nht [ABC] + [ABP ] t gi tr nh nht. rng : 2f = AB.(AQ + P B). sin

T MN . Gi = CAB = CBA v B= ACB. Gi f = [AQT ] + [BP T ] = [ABQ] + [ABP ] 2.[ABT ]

CM PQ

= 1+

CP PQ

CQ PQ

= 1+

sin p+sin q sin

1+ . sin P Q t gi tr nh nht khi cos pq ln nht. T , [AT Q] + [BT P ] 2 nh nht khi p = q, tc l khi P Q AB. 2.15.Cho n im trn mt phng (n >= 4) sao cho khong cch gia 2 im bt k trong n im l mt s nguyn. CMR t nht 1 trong s cc 6 khong cch chia ht cho 3. Li gii: Trong bi gii ny, cc ng d xt theo modul 3. Trc ht

thi olympic Bungari

19

ta chng minh nu n = 4, th t nht c hai im ri nhau m khong cch gia chng chia ht cho 3. K hiu 4 im lA, B, C, D. Gi s cc khong cch AB, BC, CD, DA, AC, BD khng chia ht cho 3. Khng mt tnh tng qut, ta gi s BAD = BAC = CAD. Gi x=BAC v y = CAD. Gi = 2.AB.AC. cos x, = 2.AD.AC. cos y v =

2.AB.AD. cos (x + y). p dng nh l hm s cosin cho cc tam giac ABC, ACD, ABD ta c BC 2 = AB 2 + AC 2 CD2 = AC 2 + AD 2 BD2 = AB 2 + AD 2 V bnh phng mu khong cch l mt s nguyn nn , v cng l cc s nguyn. Do :

dng ti gin t s l s khng chia ht cho 3. t p = 2.AB.AC v q = 2.AD.AC, do cos x =


p

v cos y = 2 q2 2 2 (p ).(q p ) V sin x.siny = pq L s hu t nn t s v phi cng l mt s nguyn. T s chia ht cho 3 v p2 1( modul 3) v 2 1 (modul 3 Nhng mu s khng chia ht cho 3. Do , khi sin x. sin y vit di dng ti gin th t ca n chia ht cho 3, iu ny mu thun. Do , iu gi s ban u l sai. Vy c t nht mt khong cch chia ht cho3

v sin x. sin y =

(1 cos2 .x).(1 cos2 .y) l mt s hu t, khi vit di

= . 4.AB.AD. sin x. sin y. l s nguyn. V vy: 4.AC 2 .AB.AD. sin x. sin y l mt s nguyn chn

= 4.AC 2 .AB.AD (cos x. cos y sin x. sin y)

.c

2.AC 2 . = 4.AC.AB.AD. cos (x + y)

20

Nguyn Hu in, HKHTN H Ni vi n = 4 4 Xt trng hp n 4. T mt tp n im, c Cn cc tp con cha 4 im c t nht hai im trong mi tp ri nhau c khong cch chia 2 ht cho 3, v mi khong cch c m trong t nht Cn1 tp con. vy c t nht
4 Cn 2 Cn2 2 Cn 6

cc khong cch chia ht cho 3

2.16.Trong tam gic ABC, CH l ng cao v CM v CN tng ng l cc ng phn gic ca cc gc ACH v BCH. Tm ng trn ngoi tip ca tam gic CMN trng vi ng trn ni tip ca tam gic ABC. CMR: [ABC] =
AN.BM . 2

Tng t, BM = BC. Do vy: 1 AN.BM = 1 .AC.BC = [ABC]. 2 2 2.17.Cho dy s (an ): a1 = 43, a2 = 142 v an+1 = 3.an + an1 vi mi n CMR 2.

(a) an v an1 l nguyn t cng nhau vi mi n >= 1. (b) Vi mi s t nhin m, tn ti v hn s t nhin n sao cho an 1 Li gii: (a) Gi s c n, g > 1 sao cho Khi g chia ht an1 = an+1 3.an v an+1 1 u chia ht cho m.
g an

T suy ra, CNA = (ACN + NAC) = BAC = ACN v AN = AC. 2 2

Do , HCB = CBH = BAC v 2 1 ACN = ACB 2 .HCB = BAC 2 2

. 2

V NI = NC, nn hai tam gic ny l ...(congruent), v NF = IE = IF . Tam gic NF I l tam gic vung cn, F IN = v ACB = 2.F IN = 4

.c

Do F IN = ECI. ta cng c NF I = = IEC. Nn NF I IEC. 2

Li gii: Gi I l tm ng trn ni tip ca tam gic ABC, gi tip im ca ng trn ni tip tam gic ABC vi cc cnh l AC, AB ln 1 lt l E v F . V IM = IN v IF IM, nn ta c F IN = 2 .MIN . Hn na , v I l tm ng trn ngoi tip ca tam gic CMN nn: 1 .MIN = MCN = 1 .ABC = ECI. 2 2

g . an+1

thi olympic Bungari

21

Nu n 1 > 1 th g chia ht an+1 , an , ...a2 , a1 , nhng iu ny khng th xy ra v WCLN (a2 , a1 ) = 1. Do , an v an+1 l nguyn t cng nhau vi mi n 1. (b) Xt dy (an ) c xc nh nh sau: a1 , a2 an+1 = 3.an + an1 vi mi n 2. D thy: a3 = 4, a4 = 13, a5 = 43, a6 = 142 Tc l a1 = a5 , a2 = a6 . M hai dy (an ) v dy (an ) c cng cng thc truy hi nn ta c: an = an+4 , vi mi n 1. Gi bn l s d khi chia an cho m, v xt cp s (bn , bn+1 ) vi n 1. V c mt s v hn cc cp s nh vy nhng ch l m2 cp cc s nguyn (r, s) vi 0 r, s < m do phi c hai trong s cc cp trng nhau, chng hn (bi , bi+1 ) = (bi+t , bj+t ) vi t > 0.

v BEF bng nhau, suy ra BC = BF v BF E = BCE = EDA. Do , t gic ADEF l t gic ni tip ng trn. V AEB = v 2 CEB = BEF nn ta c F EA = AED. T suy ra F DA = F EA = AED = AF D. Do : AF = AD v AB = AF + BF = AD + BC Nu AB = BC + AD th c im F thuc AB sao cho AF = AD v BF = BC. Khi hai tam gic BCE v BF E l bng nhau v t gic ADEF l t gic ni tip c ng trn. Cng c F DA = AF D. Do , F EA = F DA = AF D = AED, do ng thng AE l phn gic ca gc F ED. V BCE = BF E nn EB l phn gic ca gc CEF do vy AE

Li gii: Nu AEB = th CEB < . T suy ra c im F nm 2 2 trn cnh AB sao cho BEF = BEC. Khi , c hai tam gic BEC

CMR: AEB =

khi v ch khi AB = AD + BC.

2.18.Cho t gic li ABCD c BCD = CDA, ng phn gic ca gc ABC ct CD ti im E.

(b1+kt , b2+kt ) = (b1 , b2 ) = (1, 1) vi mi k akt2 1 u chia ht cho m vi mi k 4.

S dng cng thc truy hi, ta d dng chng minh c bng qui np theo n: bi+n = bi+n+t vi mi n tha mn (i + n) > 1. Do ,

.c

1. Do , akt3 1 v

22 BE v AEB = /2. 2.19.Trong h ta Oxy,

Nguyn Hu in, HKHTN H Ni

mt

tp

gm

2000

im

(x1 , y1 ), (x2 , y2), ...(x2000 , y2000 ) c gi l tt nu 0 xi 83, 0 y 1 vi i = 1, 2, .., 2000 v xi = xj khi i = j .Tm s nguyn dng n ln nht sao cho vi mi tp tt phn trong v bin ca hnh vung n v no cha ng n im trong tp l phn trong v phn bin ca tp tt ; Li gii: Trc ht ta chng minh rng vi mi tp tt, mt hnh vung n v no cha ng 25 im ca tp tt . Ta gi mt hnh vung n v l proper (ring) nu 2 cnh ca n nm trn cc ng thng y = 0 v y = 1.

trong cnh bn tri ca Si nm trn ng x = ti vi i = 1, 2..., k 1, c nhiu nht mt trong cc im cho trc nm trong min c xc nh bi zi x < zi+1 , c nhiu nht 1 trong cc im cho trc nm trong min c xc nh bi zi+1 < x zi+1 = 1. Do , vi mi i s cc im trong Si khc vi cc im trong Si+1 hoc l 1, 0 hay 1. V c Si1 cha t nht 25 im v c Si2 cha nhiu nht 25 im , t suy ra c Si3 (i3 nm giai1 v i2 ) cha ng 25 im. By gi ta chng minh rng 83 t d = 2. 1999 , xi = (i 1). 1 .d vi i = 1, 2, ..., 2000 v y2k1 = 0, y2k = 1 2 vi k = 1, 2, ..., 2000. Vi 2 im phn bit bt k (x1 , y1 ) m cng nm trn ng nm ngang

dng x = xi hay x = xi 1. Th t cc hnh vung n v t tri qua phi gi s l: S1 , S2 , ...Sk ,

t hn 26 im. Hn na trong 83 hnh vung n v , xt cc hnh vung n v proper m cc cnh bn tri nm trn cc ng thng

V 83.24 < 2000, nn mt trong cc hnh vung phi cha nhiu hn 25 im. V 83.26 82 > 2000 nn mt trong cc hnh vung cha

.c

Min R c th c chia thnh cc hnh vung n v proper m cc cnh bn tri nm trn cc ng thng c phng trnh : x = i vi i = 0, 1, .., 8.

Mi im cho trc u nm trong min R = (x, y)|0 ex 83, 0 y 1

thi olympic Bungari

23

2 (y = 0hocy = 1) th khong cch gia chng thp nht l d > 25 Gi XY ZW l 1 hnh vung n v. Vi j = 0, 1 min Ro b chn bi

hnh vung giao vi mi ng thng y = j trong mt khong ng c di ri . Nu t nht mt trong cc s ro , r1 l 0 th khong tng ng cha nhiu nht mt im (xi , yi ). Khong khc c chiu di nhiu nht 2, v do c th cha nhiu nht [ d2 ] + 1 18 cc im nh

vy ni chung khng vt qu 19. Ta cng c, nu XY ZW c mt cp cnh nm trn cc ng nm ngang th Ro cha nhiu nht [ 1 ]+1 25 d cc im nh vy/
2

Mt khc, Ro giao vi ng y = 0 v y = 1 ti cc im P, Q v R, S, trong P v R nm bn tri Q v S. Ta cng c P Q v RS cha nhiu nht [ PdQ ] + 1 v [ RS ] + 1 cc im chn. d

cng c: TR + R S + S V = TU = 1

Mt khc, theo bt ng thc tam gic T R + S V = R V + S V > R S T suy ra R S < 1 . 2 V P Q = R S , s cc im (xi , yi) nm trong XY ZW nhiu nht: [ PdQ ] + [ RS ] + 2 d
P Q +R S d 1 + 2 < d + 2 < 15 Bi ton c chng minh.

2.20.Cho tam gic nhn ABC (a) CMR c duy nht ba im A1 , B1 , C1 tng ng nm trn BC, CA, AB tha mn:nu ta chiu hai trong ba im ln cnh tng ng (cn li), th trung im ca hnh chiu l im cn li.

Gi K v L l cc nh trn cng ca cc cnh thng ng ca R2 (v cng thuc min b chn R1 ). Ta c: KT R S V R S V L Ta = =

Gi T v U l cc nh bn tri v bn phi ca min R2 trn ng y = 1 v gi V l nh ca R1 trn ng y = 1.

c P Q = R S do tnh i xng. Gi R2 l min thu c bi php quay R1 quanh tm ca n. Khi min R1 R2 R1 R2 l hp ca 8 min tam gic bng nhau.

.c

v gi P , Q , R v S l giao ca n vi cc ng y = 0 v y = 1 c xc nh tng t nh trn. Khi : P Q + R S = P Q + RS. Ta cng

Dch chuyn Ro theo hng song song vi cc cnh ca hnh vung n 1 v m tm ca n nm trn ng thng y = 2 . Gi R1 l nh ca Ro

24

Nguyn Hu in, HKHTN H Ni (b) CMR tam gic A1 B1 C1 ng dng vi tam gic c cc nh l trung im ca ABC Li gii: (a) Trc ht ta xem xt ngc li, gi s c tam gic A1 B1C1 c tnh cht nh vy. Gi T l trung im ca A1 B1 theo nh ngha C1 T AB Gi P l trng tm ca A1 B1 C1 . V P A1 BC, P B1 CA v P C1 AB, P xc nh duy nht A1 B1 C1 . R rng cc t gic AB1 P C, BC1 P A1 , CA1 P B1 l cc t gic ni tip c ng trn. t = CAB, = ABC, x = A1 B1 P v y = B1 A1 P B1 CD = y

Do cc ng CP v CF i xng qua ng phn gic ca gc ACB. Ta c kt qu tng t cho cc ng AP v AD, BP v BE, trong D v E l cc trung im ca cc cnh BC v CA T suy ra P l "isognal cnugate" ca G, G l trng tm ca ABC. Do , P l duy nht v bc ngc li ch ra rng P xc nh duy nht A1 B1 C1 tha mn iu kin ca bi ton. (b) Ko di AG v pha G n K sao cho GD = DK. Khi , BGCK l hnh bnh hnh v 2 CK = BG = 3 .BE, CG = 2 .CF 3
2 GK = AG = 3 .AD

V tam gic ABC nhn nn ta suy ra: A1 CP = x = ACF v B1 CD = y = BCF

= sin sin Trong , F l trung im ca cnh AB

sin ACE sin BCF

sin y sin x hay sin = sin Mt cch tng t ta CM c :

.c

p dng nh l hm s sin cho cc tam gic A1 T P v B1 T P ta c: sin y TP TP sin x = T A1 = T B1 = sin sin

V cc t gic AB1 P C1 v CA1 P B1 l ni tip c JP B1 = , JP A1 = , A1CP = x,

thi olympic Bungari

25

Do , tam gic CGK ng dng vi tam gic to bi cc ng trung bnh ca ABC. Ta cn chng minh A1 B1 C1 v CGK l ng dng Tht vy: B1 C1 A1 = B1 C1 P + A1 C1 P = B1 AP + A1 BP = BAG + GBA = KGB + GKC Chng minh tng t ta c: C1 A1 B1 = KCG 2.21.Cho p 3 l mt s nguyn t v a1 , a2 , ..., ap2 l mt dy cc s

nguyn dng sao cho p khng chia ht cho ak hoc ak 1 vi mi k k = 1, 2, ..., p 2. CMR tch ca mt s phn t ca dy ng d vi modulo p. Li gii: Ta chng minh bng qui np theo k = 2, ..., p 2 c cc s nguyn bk,1 ...bk,i sao cho: (i) mi bk,i hoc bng 1 hoc l tch ca mt s phn t ca dy a1 , a2 , ..., ap2 v

V cc s ak bk,i l khc nhau theo modulop nn phi ko sao cho cc s bk,1 , ..., bk,k , ak .bk,1 khng c hai s no *ng d thep modul p. t bk+1,1 , bk+1,2 , ..., bk+1,k+1 l cc s trn. Mi b k + 1 s ny u bng 1 hoc l tch ca mt s phn t ca dy a1 , a2 , ..., ap2 . Php quy np c chng minh hon ton. Xt cc s bp1,1 , ..., bp1,p1 . Chc chn mt trong cc s ny ng d vi 2 theo modul p v s khc 1 v ng d vi tch ca mt s s ak 2.22.Cho tam gic nhn ABC cn ti A. Gi D l trung im ca AB. Chn E trn AB, v ly O l tm ng trn ngoi tip ca ACE. Chng minh rng ng thng qua D vung gc vi Do , ng thng qua E

(ak bk,1 )(ak bk,2 )...(ak bk,i) bk,1 bk,2 , ...bk,i (modp) Do , chng ta khng th hon v (ak bk,1 ...ak bk,k ) sao cho mi phn t l ng d theo modulop vi phn t tng ng trong (bk,1 ..., bk,k ).

Gi s chng ta chn c bk,1 , ..bk,k . V ak 1 (mod p), ta c:

(ii) bk,m = bk,n (modp) vi m = n Vi k = 2, ta c th chn b1,1 = 1 v b1,2 = a1 1 (modp)

.c

26

Nguyn Hu in, HKHTN H Ni vung gc vi BC v ng thng qua B song song vi AC l ng quy. Li gii: Gi l l ng thng i qua B v song song vi ng thng AC, gi F1 v F2 l cc im trn ng thng l sao cho OD DF1 v BC EF2 Gi H1 vH2 ln lt l hnh chiu vung gc ca F1 v F2 l ng thng AB. V gc CAB nhn nn im 0 nm trong ABC. T , suy ra F1 nm gia hai tia AB v AC. V gc ABC nhn nn F2 cng nm gia hai tia AB v AC. Ta cn chng minh F1 H1 = H2 F2 . Gi G l tm ng trn ngoi tip ca tam gic ABC v gi O1 , G1 ln lt l hnh chiu vung gc ca 0 ln AB v G ln 001 . V OD DF1 , OO1D DH1 F nn
DH1 F 1 H1

a
= =

Do , cc tam gic vung GOG1 v F1 BH1 ng dng v: BH1 = OG1 (2) BF1 OG T (1) v (2) ta suy ra: F1 H1 =

. V DGB = ACB = 2.x, ta thu c F1 H1 = tan2x.O1 D

DH1 .O1 D BH1 .O1 D OO1 OG1 BD.O1 D BD.O1D = G1 O 1 GD

DH1.O1 D BH1 .O1 D = OG1 OO1

.c

ca gc AGC nn CGO = x. V CG OO1 , G1 OG = CGO = x

OO1 = O1 D (1) t BAC = CBA = x. V AG = GC v AO = OC, GO l phn gic

thi olympic Bungari Gi I l giao im ca BC v EF2 V BF2 AC, F2 BI = ACB = 2x v H2 BF2 = x rng BE = AB AE = 2.(AD AO1 ) = 2O1 D T suy ra:

27

F2 H2 = BF2 . sin H2 BF2 = BF2 . sin x

BI cos F2 BI

. sin x =

BI. sin x cos 2x

Ta c iu phi chng minh.

CMR: nu a, b, c A sao cho a + b + c = 0 th f (a) + f (b) + f (c) = 0

Li gii: Xt dy e1 = (1, 0, 0, ..., 0), e2 = (0, 1, 0, ..., 0), ..., en = (0, 0, ..., 0, 1). Vi mi i, 0 v ei khc nhau do phn t 1, nn f (0) v

f (ei ) cng khc nhau. Nh vy tc l f (ei = ej ) vi j no . Xt mi dy ty x = (x1 , x2 , ..., xn ) vi f (x) = (y1 , y2 , ..., yn ). Nu x c tham s 1 th f (x) cng c tham s 1. Nu f (e1 ) = ej v xi = 1 th ei v x c t 1 phn t khc nhau . iu ny ch xy ra nu yj = 1, v nu khng ej v f (x) s c t + 1 phn t khc nhau. Mt cch tng t , nu xi = 0 th yj = 0

Gi f : A A l nh x vi f (0) = 0 sao cho nu a v b c ng n phn t khc nhau th f (a) v f (b) cng c ng n phn t khc nhau.

c = (c1 2, ..., cn ) c gi l tng a + b ca cc dy a = (a1 , a2 , ..., an ) v b = (b1 , b2 , ..., bn ) nu ci = 0 khi ai = bi v ci = 1 khi ai = bi

2.23.Cho n l mt s nguyn dng. Mt dy s c gi l dy nh phn nu cc phn t ca n l 0 hoc 1. Gi A l tp tt c cc dy nh phn c n phn t , v gi 0 A l dy m cc phn t u l 0. Dy

.c

BE. cos x sin x = O1 D.tan2x = F1 H1 cos 2x

28

Nguyn Hu in, HKHTN H Ni Nu a = (a1 , a2 , ..., an ), b = (b1 , b2 , ..., bn ), c = (c1 , c2 , ..., cn ) v a+ b+ c = 0 th ai + bi + ci l chn vi i = 1, 2, ...n Vi mi ej ta c th chn ej sao cho f (ei ) = ej Cc phn t th j ca f (a), f (b), f (c) tng ng l ai , bi , ci nn tng ca chng l mt s chn . Do , f (a) + f (b) + f (c) c phn t th j l 0 vi j v f (a) + f (b) + f (c) = 0.

.c

Chng 3 thi olympic Canada


3.24.Cho a1 , a2 , ..., a2000 l mt dy s nguyn lin tip trong khong
i=1

ta c th t b1 bng ai no thuc [999, 1000]. n nh ch s i ny. Gi s chng ta xy dng dy b1 , b2 , ..., bk (1 k < 2000) vi k c n nh. Nu
k i=1

Chng ta gii hn bi . Khng phi tt c cc ai = 1000 do vy chng

i=1

ai c th khng xc nh hoc xc nh. V vy t nht mt ai l xc nh ( hoc khng xc nh). t bk+1 [1, 1000] hoc [1000, 0], c ngha l
k+1 i=1

bi [999, 0] hoc [1, 1000] th tng ca cc

sao cho

bi [999, 1000] vi n = 1, 2, 3, ..., 2000

thnh dy

b1 , b2 , ..., b2000

a1 , a2 , ..., a2000

Li gii: Ta thy rng c th sp xp li dy

.c

bi [999, 1000]

Chng minh rng iu kin xc nh l c dy con ca a1 , a2 , ..., a2000 c tng bng 0

[1000, 1000]. Gi s

2000

ai = 1

30

Nguyn Hu in, HKHTN H Ni C lp li qu trnh trn ta xy dng c dy b1 , b2 , ..., b2000 . Bng cch xy dng trn ta xy dng c dy tng ring n =
n i=1

bi (1 n 2000) bng 1 ca 2000 s nguyn thuc

[999, 1000]. Bi vy nu i = j vi i<j hoc tri li i = 0 vi mt vi i. Trong trng hp u tin ta c dy con bi+1 , bi+2 , ..., bj c tng bng 0. Trong trng hp th 2 c dy con b1 , b2 , ..., bi c tng bng 0. Vy ta c pcm. 3.25.Cho t gic ABCD c CBD = 2ADB, ABD = 2CDB, AB = CD. Chng minh rng AB=CD. Li gii: t x = ADB, y =CDB, CBD = 2x, ABD = 2y p dng nh l Sin trong tam gic ABD v tam gic CBD ta c:

Li gii: Vi i 3 ta c 0 ai a2 v suy ra ai (ai a2 ) 0 Du "=" xy ra nu ai {0, a2 } Suy ra


100 100

Tm max ca a2 + a2 + ... + a2 v ch ra cc ai t c. 1 2 100

3.26.Cho dy s thc a1 , a2 , ..., a100 tha mn a1 a2 ... a100 0(1) v a1 + a2 100(2), a3 + a4 + ... + a100 100(3)

1 1 (cos(y + x) cos(3y + x)) = 2 (cos(x + y) cos(3x + y)) 2 cos(3y + x) = cos(3x + y) Do 0 < x + y = 1 ABC < 0 < 3y + x + (3x + y) < 2 2 2

3y + x = 3x + y x = y ABD = CBD AD = CD

.c
a2 i

o
100 i=3

= BD = BD = sin((2x+y)) BA BC sin y sin(2y + x) sin y = sin(2x + y) sin x


sin((2y+x)) sin x

i=3

a2

ai
i=3

Theo (3) du "=" ch xy ra nu

ai = 100 hoc a2 = 0

T (1) v (2) suy ra 0 a2 100 a1 100 a2 hoc 0 a2 50

thi olympic Canada

31

2a2 (a2 50) 0 Du "=" xy ra nu a2 = 0 hoc a2 = 50


100

i=1

a2 = a2 + a2 + i 1 2

100

i=3

a2 (100 a2 )2 + a2 + 100a2 i 2

= 10000 + 2a2 (a2 50) 10000 Du "=" ch xy ra nu : (a) {a3 , a4 , ..., a100 } {0, a2 }
100

(b)
i=3

ai = 100 hoc a2 = 0

(d)a2 {0, 50} T iu kin trn dy a1 , a2 , ..., a100 c th l: 100, 0, 0, ..., 0

(c)a1 = 100 a2

.c

hoc 50, 50, 50, 50, 0, 0,...,0 Vy tng ln nht l 10.000

Chng 4 thi olympic Trung Quc


4.27.Cho tam gic ABC tha mn BC CA AB.Gi R v r ln lt l

p dng cng thc sau :

A B C sin sin = 4R sin x sin y sin z 2 2 2 Ta suy ra c s = 2R(sin2x + sin2y -1 4sinxxinyxinz). r = 4R sin +) nu ABC l tam gic vung ti C vi C = .Ta c : 2R = c v 2r 2 = a + b c s=0. Do ,chng ta nhm tha s chung cos2z trong biu thc s : s = 2 sin (x + y) cos (x y) 1 + 2(cos (x + y) cos (x y)) sin z 2R = 2 cos z cos (x y) 1 + 2 (sin z cos (x y)) sin z

2R =

a b c a b c = = = = = sin A sin B sin C sin2x sin 2y sin 2z

= 2z. Ta c :0 < x y z v x + y + z = + b 2R 2r.

.c

Li gii: t AB = c,BC = a,CA = b, gc A = 2x, gc B = 2y, gc C


2

?.

bn knh cc ng trn ngoi tip v ni tip tam gic ABC.Tm theo gc C ca tam gic BC + CA 2R 2r l dng ?,m hoc bng 0

t s =BC + CA 2R 2r = a

thi olympic Trung Quc = 2cos (x y) (cos z sin z) cos2z = 2cos (y x) .

33

cos2 z sin2 z cos2z cos z + sin z 2 cos (y x) = 1 cos2z cos z + sin z

T chng ta c th a vo gi tr cosz + sinz bi n l dng khi 0 < z < . 2 Ch rng : y x < min {y, x + y} min{z, z}. 2 V z
2

nn ta c:

cos (y x) > max cos z, cos T suy ra :

z 2

= max {cos z, sin z}

a1 , a2 , ...

fn = an + 2 (n ) an1 + 3 (n ) an2 + ... + n 1 2

1 v an = 2 nan1 + 1 n (n 1) an2 + (1)n 1 n 2 2 Vi n 3,tm mt cng thc nh ngha cho hm : n n1

a1 = 0, a2 = 1

c xc nh 1 cch quy nh sau :

h
a1

4.28.Dy s v hn

Li gii: Cch gii 1: Vit li mi quan h quy thnh : 1 1 an = (1)n + nan1 + n((1)n1 + (n 1) an2 ) 2 2 Nu (1)n1 + (n 1) an2 = an1 Ta c: 1 1 an = (1)n + nan1 + nan1 = (1)n + nan1 2 2

.c

V vy s = pcos2z i vi p > 0 hay s = BC + CA - 2R - 2r c th dng,bng 0 hoc m nu gc C tng ng l nhn,vung,t.

2 cos (y x) 1 >0 cos z + sin z

34

Nguyn Hu in, HKHTN H Ni Do dng phng php quy np t biu thc an = (1)n + nan1 ta d dng tm ra c: an = n! n! n! n! n! + + ... + (1)n 1! 2! 3! n!

V th,theo cng thc ni ting Euler an l chui s ca s xo trn ca b s (1, 2, . . . , n), ngha l s hon v ca b n s m khng c im c nh. mi cp (, j)ca hon v phn bit t 1 phn t hay 1 s nguyn j trong dy 1, 2, . . . , n,ta xc nh 1 im ch nu j l 1 im c nh ca Vi k xc nh k = 1, 2, . . . , n, c n nk ak hon v vi n-k im xc

.c

Xt tng cc hon v,ta c tng s im ch c xc nh:


n

n nk

xc nh j. V vy,xt ton b tng,ta c tng s im xc nh ch c ch ra l:


n j=1

((n 1)! 1) = n(n 1)! n

Cho 2 tng trn bng nhau,ta c: fn = 2.n! n 1 Lu : sau khi ch ra c fn = 2.n! n 1 i vi cc gi tr nh ca n,ta c th s dng mi quan h quy v ng nht ng thc cc phn t chng minh cng thc l ng vi mi n.

Mt khc: Vi mi j, j {1, 2, ..., n} c (n 1)! 1 hon v phn bit t phn t

k=1

Khi tng

ak m c n! 1 hon v t hn n im c nh.

k=1

(n

k) (n )ak nk

nh, c (n ) cch chn cc im c nh ny,v chui ak ca k im nk cn li.Vi mi hon v nh vy,c n-k cp (, j) c xc nh.

= fn

m
k=1

(n )ak = fn (n! 1) nk

thi olympic Trung Quc

35

Cch gii 2: Chng ti gii thiu 1 phng php chng t rng an l chui s ca hon v (1, 2, . . . , n).Vi n 3, ta c : an = nan1 + (1)n = an1 + (n 1) an1 + (1)n = (n 1) an2 + (1)n1 + (n 1) an1 + (1)n = (n 1) (an1 + an2 ) Gi bn l chui s ca hon v ca (1, 2, . . . , n).Mi hon v l mt trong s nhng dng sau y: a)Vi k = 1 1 nh x ti k v k nh x ti 1.Nh th s c n-1 gi tr cho k v vi mi k c bn2 hon v cho n-2 phn t cn li .Do vy,c (n 1) bn2 hon v nh trn. b) 1 nh x ti k nhng k khng nh x ti 1.gi tr k c nh. Nh vy

4.29.Mt cu lc b bng bn mun t chc 1 gii u i,mt lot nhng trn u m trong mi trn u mt cp ngi chi s thi u vi mt cp khc.Gi s trn u ca mt ngi chi trong mt gii u l s trn u m anh(c) ta tham gia. Cho dy s A = {a1 , a2 , ..., ak } phn bit,nguyn dng,chia ht cho 6. Xc nh s lng ngi chi ti thiu c th thit lp mt gii u i m :

minh.

V vy bn = (n 1) (bn1 + bn2 ) T a1 = b1 = 0 v a2 = b2 = 1, an = bn vi n 1 nh yu cu chng

Cho k bin thin t 2 n n,ta thy c (n 1) bn2 hon v ca dng (b).

y t l s chuyn i gia 1 v k.Bi c bn1 nh x trong ch s 1 l c nh ,nn c bn1 php hon v .

.c

tn ti 1 song nh gia cc hon v v cc hon v c ch s 1 l c nh,thng qua nh x

36

Nguyn Hu in, HKHTN H Ni (i) mi ngi tham gia nhiu nht l 2 cp u. (ii) bt k 2 cp khc nhau c nhiu nht 1 trn u gp nhau. (iii) nu 2 ngi chi cng mt cp,h khng bao gi phi thi u vi nhau. (iv) S lng cc trn u ca ngi tham gia c thit lp l A. Li gii: B . Gi s rng :k 1 v 1 b1 < b2 < < bk .Nh th tn ti mt

th c bk + 1 nh trong b s {b1 , b2 , ..., bk } l s ca cc nh trong bk + 1 nh. Chng minh: Ta chng minh b bng phng php quy np theo k.

{b2 b1 , ..., bi1 b1 } Ngoi ra dng cc ng thng c nh trong S1 l im cui.Mi nh trong S1 by gi c bi , mi nh trong S3 c b1 , v ca cc nh trong S2 c xc nh t tp hp {b2 , ..., bi1 }. T kt hp li tt c cc ca bi + 1 nh trong th G c xc nh t tp {b1 , b2 , ..., bi }. iu ny hon tt bc quy np v c iu phi chng minh. Gi s rng ta c 1 gii u i trong n ngi chi tha mn iu kin a ra.C 1 nht 1 ngi chi,ta gi l X,c s trn u l max(A). Gi m l s cp khc anh(c) y phi thi u.Mi cp ny c 2 ngi

Theo gi thit quy np,ta c th dng cc ng thng gia cc nh trong S1 trong ca cc nh c xc nh t tp hp

im c bi + 1 nh,v chia chng ,v chia chng thnh 3 b gi tr S1 , S2 , S3 vi |S1 | =b1 , |S2 | = bi1 b1 + 1, v |S3 | = bi (bi1 + 1).

k < i .Ta dng th G ca bi + 1 nh,to thnh cc ng thng gia hai im v t thay i ca cc nh trong mi nh.Chn nhng

.c

2 nh bng mt ng thng khi v ch khi mt trong s cc nh l phn bit. Ta cn chng minh b ng vi k = i 3 v gi s n ng khi

Nu k = 1,ton b th gm nhng nh b1 tha mn. Nu k = 2,ly b2 + 1 nh, phn bit cc nh ny vi nh b1 v ni

thi olympic Trung Quc

37

chi v c tnh l 2m.Bt k ngi chi no c tnh nhiu nht 2 ln theo cch cu thnh ny bi v mi ngi chi thuc nhiu nht 2 cp . Do ,ngi chi X s phi u vi t nht m ngi chi khc.Nu X trong j cp (vi j = 1 hoc 2),s c nhiu nht tng s m + j + 1 ngi chi. Ngoi ra X chi nhiu nht jm trn,ko theo jm max (A) . Do n m + j + 1 max (A) /j + j + 1 min {max (A) + 2, max (A) /2 + 3}.

V max (A) 6 ,ta c max(A) + 2 > max(A)/2 + 3,ko theo n max (A) /2 + 3 T b ,ta c th dng th ca xc nh t tp hp { a61 , a62 , ..., a6k }.

Ta cn chng minh n = max(A)/2 + 3 l s trn nhiu nht c th t c.

l lin k. Gi s rng ta c 1 i c sp xp nh v

ngi chi vi nhng cu th chi tng ng) ng vi cc nh ca th G,v 2 i c xc nh thi u khi v ch khi cc nh tng ng

Chia n ngi chi trong max(A) + 1 thnh ba phn ,v 2 ngi chi 6 cng trong 1 i khi v ch khi h cng nm trong mt phn ca ba phn trn .Gn cho mi phn (v cng mt thi im,hnh thnh 3 cp

.c

max(A) 6

+ 1 nh trong c

nh w lin k vi v, l i phi u trong mt phn ba c sp xp cho nh w tng cng ai trn. 2 Mi ngi chi c xp trong v l trong 2 i,t c s trn u l 2. ai = ai . 2 V vy s lng cc trn u ca ngi tham gia chi l {a1 , a2 , ..., ak} ,khi cn thit. 4.30.Cho s nguyn n 2 . i vi bt k tp hp n s ca dy s thc A = (a1 , a2 , ..., an ) Cho li im ca A l s k {1, 2, ..., n} trong ak > aj vi mi gi tr 1 j < k .

ai .Vi 6

mi

ai 6

38

Nguyn Hu in, HKHTN H Ni Xt tt c cc hon v A = (a1 , a2 , ..., an ) ca (1, 2, . . . , n) vi im li im l 2.Xc nh v chng minh ngha s hc ca phn t u tin a1 trong hon v ny?. Li gii: Vi mi tp hp n s ca dy s thc A = (a1 , a2 , ..., an ) Nu ak > aj vi mi gi tr 1 j < k, ta gi ak l 1 li im. Nu mt hon v A = (a1 , a2 , ..., an ) ca (1, 2, . . . , n) c s li im l 2 th 2 li im ny phi l a1 v n,trong n = ak i vi nhng gi tr k tha mn 2 k n. C nh m trong dy s {1, 2, ..., n 1}.Ta gi cc s m + 1, m + 2, ...n

V vy ngha s hc ca phn t u tin a1 c trung bnh cng mong mun l:


n1 m=1 m xm n1 m=1 xm

s b vo m 1 v tr cn li. n1 V th,ta c xm = nm (n m 1)! (m 1)! =

tin l cc s ln,t n ti v tr u v sp xp n m 1 cc s ln khc vo phn cn li ca cc v tr chn.Sau sp xp tt c cc


(n1)! nm

.c

l cc s ln, v 1, 2, ..., m 1 l cc s b.Trong mt hon v vi 2 li im ak = m, n s phi xut hin trong hon v trc tt c cc s ln khc.V vy, xc nh tt c cc hon v ny, ta chn n m v tr u

hon v

(n 1)!

(n 1)!

n1 m m=1 nm n1 1 m=1 nm

n1 m m=1 nm n1 1 m=1 nm

n1 m n1 n m=1 m m=1 m n1 1 m=1 m

=n

n1 1 + + ... +
1 2

1 n

4.31.Tm tt c cc s nguyn dng n trong n1 , n2 , ..., nk > 3 vi n = n1 n2 ...nk = 2 2k (n1 1)(n2 1)...(nk 1) 1
1

thi olympic Trung Quc

39

nguyn m duy nht nh hn 10 lm cho n = 2m 1 tha mn iu kin a ra.

Li gii: Nu mt s nguyn dng n tha mn iu kin a ra,th n = 2m 1 vi m l s nguyn dng.D dng kim tra thy 3 l s

Cho m 10,ta phi chng minh 2m 1 khng tha mn iu kin a ra. Gi s, ch ra s mu thun, lp phng trnh biu din mt s k v n1 , n2 , ..., nk : m= 1 (n1 1) (n2 1) ... (nk 1) 10 2k
3 3

n = 2m 1 > n1 n2 ...nk = n iu ny mu thun. 4.32.Mt bi thi bao gm 5 cu hi nhiu la chn,mi cu c 4 la chn khc nhau.C 2000 sinh vin lm bi thi,v mi sinh vin ch c chn ng 1 p n trong mi cu hi. Tm gi tr nh nht ca n v nhng bi thi ca sinh vin tha mn iu kin: trong bt k n bi thi,tn ti 4 bi thi trong bt k 2 bi no c nhiu nht 3 p n ging nhau. Nh vy gi thit a ra l sai v n = 23 1 = 7 l p n duy nht.

Cho i = 1, 2, . . . , k. t (1) v (2) cng nhau,ta thu c

ni 1 2

5.Do

V n = 2m 1 l l, ni l chn,v vi mi ni > 3, ni nh nht phi l 4

2 1 >m =

n1 1 2

.c

o
3

quy np 2 1 > V vy

vi cc s ngyn

10 n2 1 2

Vi 10 ,ta c +1 < 5 < 2 4 S dng kt qu ny,ta d dng chng minh c bng phng php

nk 1 ... 2

. (1)

ni 1 > ni (2) 2

40

Nguyn Hu in, HKHTN H Ni Li gii: Trc tin,ta chng minh n 25 .Gi 1,2,3,4 l cc la chn ca tng cu hi. Th hin mi cu tr li ca sinh vin bng mt chui c trt t bao gm 5 phn t (a1 , a2 , a3 , a4 , a5 ), ai {1, 2, 3, 4} Trong p n ca sinh vin trong cu hi i l ai .Ta ni rng 2 bi thi l cng loi nu chui s 5 phn t ca n thuc vo mt b ca dng sau { (k, a2 , a3 , a4 , a5 ) |k {1, 2, 3, 4} }, Trong a2 , a3 , a4 , a5 {1, 2, 3, 4} . V v c 256 b,v 2000 = 256x7 + 208,nn c t nht 8 cu bi thi l cng loi theo nguyn l Dirichlet. Trong 1992 bi thi cn li,c 8 bi na cng loi.Cui cng,trong 1984 bi cn li,thm 8 bi cng loi na.

Do n 25 By gi chng ta ch ra rng n=25 l kt qu chp nhn c.Xc nh b s

mu thun vi gi thit c 4 bi thi trong tp A,trong c 2 bi bt k c nhiu nht l 3 cu tr li ging nhau.

.c
5 i=1

Xt tp A ca 24 bi thi ny.Cho 2 bi thi bt k trong tp A,chng phi cng loi,ngha l,p n cho 4 bi thi cui cng l ging nhau.iu ny

Khi |S| = 44 = 256 v bt k 2 bi thi c nhiu nht 3 p n ging nhau nu 5 chui phn t tng ng ca chng l cc yu t phn bit ca S. Chn bt k 250 yu t ca S,v gi nh rng chnh xc c 8 bi thi tng ng vi mi chui 5 phn t trong 250 yu t ny.V 25 > 3 nn c 8 bi thi,c 4 bi v c 5 chui tng ng l cc yu phn bit ca S,v tha mn iu kin a ra. V vy, n = 25.

S = { (a1 , a2 , a3 , a4 , a5 ) |

ai 0 (mod4) , ai {1, 2, 3, 4}}.

Chng 5 thi olympic Tip khc


5.33.Chng minh rng:
3 3 3

2(a + b)

vi x, y > 0. p dng bt ng thc trung bnh ta c 3x4 y 2 x6 + x3 y 3 + x3 y 3 Du "=" xy ra nu v ch nu x6 = x3 y 3 = y 6 hoc x=y. Cng 2 v ca 2 bt ng thc vi nhau v cng x6 + y 6 vo 2 v ca bt ng thc ta c: x6 + y 6 + 3x2 y 2(x2 + y 2 ) 2(x6 + y 6 + 2x3 y 3) Bt ng thc (*) xy ra khi x=y hoc a=b.

x2 + y 2

t 3 a = x, 3 b = y, chng ta thy rng iu tha mn chng t rng

a2 +

b2

2(x3 + y 3 )2 ()

h
3

vi mi s thc a, b. Du "=" xy ra khi no ? 3 Li gii: Nhn 2 v ca bt ng thc vi ab ta c bt ng thc tng ng

.c

2(a + b)2

a + b

b a

1 1 + a b

42

Nguyn Hu in, HKHTN H Ni

5.34.Tm tt c t gic li ABCD m tn ti mt im E nm bn trong t gic tha mn iu kin sau y: Bt k ng thng no qua E v ct cc cnh AB v CD u chia t gic li ABCD thnh 2 phn bng nhau. Li gii: Gi s rng t gic li ABCD c tnh cht . Ly X1 , X2 , X3 l 3 im nm trn cnh AB vi AX1 < AX2 < AX3 . Nh vy cnh Xk E ct on CD ti Yk vi k=1,2,3. Do t gic ABCD li v CY1 < CY2 < CY3 nn ta c: 1 0 = 2 [ABCD] 1 [ABCD] = [AX1 Y1 D] [AX2 Y2 D] 2 = [EY1 Y2 ] [EX1 X2 ] = 1 sin Y1 EY2 (EY1 .EY2 EX1 .EX2 ) 2

Bi vy chng c din tch bng nhau.(pcm) 5.35.Bi 3: Cho tam gic u ABC vi cnh y AB v ng cao CD. im P nm trn cnh CD. im E l giao im ca AP vi BC v F l giao im ca BP vi AC. Gi s rng ng trn ni tip tam gic ABP v t gic PECF bng nhau. CMR ng trn ni tip tam gic ADP v BCP cng bng nhau. Li gii: Cho 1 v 2 l ng trn ni tip t gic CEPF v tam gic ABP, v hai im I1 v I2 ln lt l tm ca ca 2 ng trn 1 v

Khi mi t gic AXYD v BXYD l hnh thang hoc hnh bnh hnh c cng ng cao v cng chiu di cnh y.

V XM1 vi Y M2 iu ta c XM1 = Y M2 v AX + DY = BX + CY

v CD. Gi s mt cnh i qua E v ct cnh AB ti X v ct cnh CD ti Y. Kt qu l n i qua M l giao im ca AB v CD.

Mt khc ta c ABCD l t gic li vi AB CD. C E l trung im ca on M1 M2 . im M1 v M2 tng ng l trung im ca on AB

.c

Do EX1 /EY1 = EX3 .EY3 v Y1 EY3 X1 EX3 Cho nn X1 X3 Y1 Y3 . iu c ngha l AB CD

Suy ra EX1 .EX2 = EY1 .EY2 Tng t c EX2 .EX3 = EY2 .EY3

thi olympic Tip khc

43

2 . Bi v chng i xng qua CD, I1 v I2 l 2 im trn on CD vi P nm gia 2 im . Bi v 1 v 2 l hai ng trn bng nhau v v ni tip trong gc i nh, chng i xng qua mi im P. Do P I1 = P I2 Do t gic ADBP v BDP ng dng, chng ta ch cn chng minh rng bn knh ng trn ni tip r1 ca tam gic BCD bng bn knh ng trn ni tip BDP. Gi X v Y tng ng l giao im ca 3 ng phn gic trong ca tam gic BCP v BDP. Nhn xt rng I1 nm trong tam gic CBF, nh vy I1 nm trn ng phn gic ca gc CBF, tc l gc CBP.

gm trng tm. Chng minh din tch ca cc tam gic ny nh hn

5.36.Cho 2000 im trong tam gic ca mt b mt thuc mt mt phng. Vi mi nh ca hnh tam gic qua mt php tnh tin l 1 tam gic bao

Do r1 = r2

r1 (P I1 + BP ) = 2 ([I1 P X] + [XP B]) = 2 [I1 BP ] = 2 [P I2 B] = 2 ([P I2 Y ] + [P IB]) = r2 (P I2 + BP )

.c

Nh vy X nm trn on BI1 v tng t Y nm trn BI2 Do P I1 = P I2 , [BI1 P ] = [BI2 P ] nn

22 . 9

Li gii: nh hng ca mi hnh trong bi ton ca 2000 im cho trong mt tam gic c 1 cnh cng vi nh i din trn. cho tam gic ABC c 1 tam gic vi AB nm ngang v A nm bn tri B. Nh vy khng c 1 im no khc trong 1999 hnh tam gic c b mt nm ngang bn di cnh AB. Chng ta bt u bi khi nim v khong cch t khong cch Oclit v chng ta nh ngha chnh thc m t mt vi mi lin h gia 2000 tam gic. Xc nh mt im hay k mt ng thng song song vi BC. cho khong cch d t mt im khc tn ti khong cch gia hai ng

44

Nguyn Hu in, HKHTN H Ni thng song song n BC i qua tam gic . Chn a = d (BC, A) nh ngha tng t vi , v khong cch gia CA v AB vi : b = d (CA, B) > 0, c = d (AB, C) > 0 . Chng ta bit rng s tnh tin nh qua 1 khong cch no cho nh ging nhau vi khong cch gia v . Gi s rng XY Z v X0 Y0 Z0 l 2 hnh tam gic trong s 2000 tam gic vi XY X Y AB v Y Z Y Z BC. Php tnh tin bin XY Z thnh X0 Y0 Z0 . Bi v trng tm ca XY Z l im trn pha tri ca Y Z = T (Y Z), 1 d (Y Z, T (Y Z)) 3 a do :
1 d (X, X ) = d (X, T (X)) 3 a suy ra

theo th t trong tam gic ABC bao gm trng tm ca 1 tam gic nhng im ca tam gic phi nm trn hoc bn tri l2 hoc nm bn tri ca l1 . T hp vi cc tr c xc nh ca tam gic ABC ko theo h qu l min b ph bi 2000 tam gic l sai vi hnh thang F c xc nh ca ng k qua mt song song ca 2000 tam gic. Do : [ ] [F ] 22 22 1 1 k = k < 9 9 9 9

hnh thang vi din tch 24 . 9 Bi v d (AB, l3 ) = 4 c , nhng im bao gm trng tm ca tam gic 3 ABC nm trn hay di l3 . Tng t d (l2 , B) = 4 b , d (l3 , C) = 4 c 3 3

cho bao li ca 4 tam gic cha F b chn bi l1 bn phi v l2 bn tri, l3 trn nh v di y AB. Quan st rng F l 1 nh ca

ng C, A v php tnh tin u1 v v2 ca 2 im tng ng C v B. Vi T1 , T2 l 2 php tnh tin vi T1 [u1 ] = v1 , T2 [u2 ] = v2 .

Php tnh tin bin nh tam gic ABC di mt phng cha C vi t 1 s 1 . Nh th [T ] = 9 . Php tnh tin T ca u1 v v1 ca hai im tng 3

.c

4 Tng t d (ZX, Y ) 4 b , d (XY, Z ) 3 c . 3

4 d (Y Z, X ) a 3

thi olympic Tip khc (pcm)

45

.c

Chng 6 thi olympic Estonia


6.37.Cho 5 s thc, chn 3 s bt k th hiu ca tng 3 s v tng hai s cn li s l mt s dng. Chng minh rng tch ca tt c 10 hiu s

thc ai bi

x2 vi 1 i

hoc x2 i

x2 ai bi = ( ai +bi )2 ai bi = ( ai bi )2 0 i 2 2 ai bi v ai bi 0 cho mi i, chng ta c th nhn 5 bt ng i


5 i=1

6.38.Chng minh rng khng th chia mt tp bt k gm 18 s nguyn dng lin tip thnh hai tp A v B, vi tch ca cc phn t trong A bng tch ca cc phn t trong B. Li gii: Chng minh phn chng. Gi s, chng ta chia c mt tp S = n, n + 1, , n + 17 ca 18 s nguyn dng lin tip thnh tp A, B sao cho aA a = bB b v tch ca cc phn t trong A bng tch ca cc phn t trong B, nu 1 tp cha mt bi s ca 19, th tp cn li cng phi nh vy. Do vy, S khng cha bi s no ca 19 hoc cha

bi = xi2 xi1 + xi xi+1 + xi+2 vi i = 1, 2, 3, 4, 5. Vi mi i, chng ta c

Li gii: Cho 5 s l x1 , x2 , x3 , x4 , x5 ta c 5 hiu s tng ng l a1 , a2 , a3 , a4 , a5 v b1 , b2 , b3 , b4 , b5 , y: ai = xi2 + xi1 + xi + xi+1 xi+2

5 th thu c 5 x2 i i=1 ai bi

.c

(ty theo s kh nng 3 s c chn) nh hn hoc bng tch ca bnh phng 5 s .

thi olympic Estonia

47

t nht hai bi s ca 19. V c duy nht 1 trong 18 s nguyn dng lin tip c th l bi s ca 19, S phi khng cha bi s no. Bi vy, n, n + 1, , n + 17 ln lt ng d vi 1, 2, 3, , 18 mod 19( chia ly d). Do vy, theo qui tc Wilson a bB b = n(n + 1) (n + 17) 18! 1 (mod 19) Tuy nhin, hai tch ca bn tri l bng nhau, iu ny khng c kh
aA

nng v -1 khng l bnh phng ca php mod 19. Bi vy, khng tn ti hai tp A v B. 6.39.Cho M, N v K l cc im tip xc ca ng trn ni tip tam gic ABC vi cc cnh ca tam gic, gi Q l tm ng trn i qua trung cc on thng MN, NK, KM. Chng minh rng tm ng trn ni tip v ngoi tip tam gic ABC thng hng vi Q. AB v X, Y, Z l trung im cc on thng NK, KM, MN. Theo bi, Q l tm ng trn ngoi tip tam gic XYZ. ng thng AX l Li gii: chng minh, gi s M, N v K nm trn cc cnh BC, CA,

tip XYZ, nn tm ca ca cc ng trn ny l thng hng vi I. Ni mt cch khc, Q, I v tm ng trn ngoi tip ca tam gic ABC l thng hng. 6.40.Tm tt c cc hm f: N N sao cho: f(f(f(n))) + (f(f(n)) + f(n) = 3n vi x N .

Li gii: Nhn xt rng nu f(a) = f(b), th vi n = a v n = b cho biu thc 3a = 3b hay a = b. Bi vy, f l duy nht. Chng ta chng minh bi qui np vi n Z th f(n) = n. Gi thit rng vi n < n0 , f(n) = n Chng ta chng minh rng f (n0 ) = n0 ( Mnh p dng vi n = 1). V f l duy nht, nun n0 > k th f (n) = f (k) = k, v vy

B v Z l hnh chiu ca C vi cng php nghch o. Dn n, nghch o nh x ng trn ngoi tip tam gic ABC n ng trn ngoi

v AXZ = . Do , Tam gic vung AXK v AKI l ng dng, v 2 IA.IX = IK 2 . Bi vy, X la hnh chiu ca A di php nghch o qua ng trn n tip ca tam gic ABC. Tng t, Y l hnh chiu ca

trung tuyn on thng KN ca tam gic cn AKN, va l ng phn gic v l ng cao ca tam gic. Do vy, A, X v I l thng hng,

.c

48

Nguyn Hu in, HKHTN H Ni f (n) n0 (*) vi n n0 .c bit, (*) p dng vi n = f (n0 v tng t vi f (f (n0 )) Thay n = n0 trong hm cho, chng ta thy: 3n0 = f (f (f (n0))) + f (f (n0 )) + f (n0 ) Du = xy ra khi f (n0 ) = n0 n0 + n0 + n0

6.41.Trong tam gic ABC chng ta c AC = BC . Ly im X bn trong tam gic v gi = A, = B, = C v = BCX. Chng t rng:
sinsin = sin() Nu v ch nu X nm trn trung tuyn ca tam gic ABC k t im C. sinsin sin()

Li gii: Ly M l trung im ca AB t = ACM v = MCB. Khng gim tnh tng qut, gi s > v BC > AC, theo quy tc

AMB = 2( + ) = ( ) + ( ) ng thc cui cng ng v + + + = v MAB > = MAD, D nm gia bn knh AM v AB v do nm trong tam gic AXB. Tng t MAD = =BCM = MCB = MCD dn n t gic MCAD l ngoi tip. Chng ta tnh cc gc cc on AD,MD v BD to vi cc cnh ca tam gic AMB. u tin chng ta c MAD = , DMA = DCA = v DB M = CB M = DB M = Kt hp cc biu thc ca cc gc tam gic AMB, ta c biu thc DAB = MAB MAD =

B AM = B AB = ABB = 2 MB A = +

v BD BCB l tam gic cn vi BC = BC v 2 = B CB nh vy CBB = . Do ABB = CBB CBA = 2 2 v MA = MB = MB, ABB = v tam gic MAB l tam gic cn, v 2 vy, cc gc ca tam gic ny l:

Chiu B qua on CX c im B v t im D trn on CB, trn pha i din on thng AB ti C, r rng = BAD. p dng inh l Ceva trong tam gic ABC i vi cc on thng ng qui AD, XD

.c

ng phn gic, phn gic gc ACB gp cnh AB ti im gn A hn B, v d ti mt im trn AM . Do > .

thi olympic Estonia v AB D = AB M DB M = p dng nh l Ceva v nh l sin, chng ta thy: 1=


sinM ADsinAB DsinB M D sinDAB sinDB M sinDM A sin() sin = sin sin sin( ) sin sin() = M B sin sin( ) MC sin sin() MA = M C sin sin( ) sin sin() = sin sin sin( ) sin sin

49

B MD = B MA DMA =

Nu X nm trn on CM, th (, ) = ( , ) v biu thc bng nhau trn dn n biu thc cho. Ngc li, gi s c biu thc cho. t = BCA v t f () = sin.sin() . Chng c cho rng sin(()) f () = f ( ), gi tri chung ny = 0 (nonzero) v 0 < , < v vy 1 sin.cos( ) cos.sin( ) = = cot cot( ) f () sin.sin( )

Li gii: a) Gi s chng minh phn chng tn ti m tp F c phn chia t cc s nguyn dng. t dy ith l F1 , F2 , ,, v (i) (i) (i) (i) Fn+2 , Fn+1 tng vi n 2 nn tn ti Ni sao cho Fn+2 , Fn+1 > m vi
(i) (i)

qu s hng N u tin ca mi dy s. Theo nguyn l Pigeonhole, hai trong s cc s k, k + 1, , k + m tn ti trong cng dy F cc s nguyn ny khc nhau ti mi m, y l mu thun. V vy cu tr li i vi phn a) l khng.

mi n > Ni . t N = max{N1 , N2 , , Ni }, v chn mt s nguyn dng k vt

b) V hn s lng ca F - sequence khng c cc phn t chung?

hng trc . C th tch tp tt c cc s nguyn dng thnh a) Hu hn

6.42.Chng ta gi mt tp v hn dy cc s nguyn dng l F - sequence nu mi s hng ca tp(Bt u t s hng th 3 ) bng tng ca 2 s

y l mt hm ng bin nghch vi 0 (0, ). Bi vy = gamma v X phi nm trn on thng CM. Mnh c chng minh.

.c

1 f ()

c nh ngha v nhn gi tr ti = v = . Tuy nhin:

50

Nguyn Hu in, HKHTN H Ni b) nh ngha dy Fibonasi {Fn } vi F0 = F1 = 0 v quy Fn+1 = Fn + Fn1 vi n > 1 n c th hin th bng cch quy np j c duy nht biu din Zeckendorp ak .ak1 .k1 vi cc tnh cht ak = 1, mi ak = i hoc 1 khng c 2 s lin tip bng 1 v j = k ak .Fk . C v i=1 hn cc s nguyn dng m c biu din Zeckendorpf kt thc vi a = 1. Vi mi m, nh ngha mt dy Fm nh sau: t s hng th n l s biu din Zeckendorpf l ak .ak1 .k1 tip theo n - 1 cc s 0. Th tng ca s hng th n v n+ 1 l n n k=1 ak .Fk+n k=1 ak .Fk+n1 +
n k=1 ak .Fk+n1

n k=1 ak .(Fk+n1 + F k = n ak .Fk+n+1 k=1

+ n)

L s hng th n+2. Do , Fm l mt dy F, bt k s nguyn dng j tn ti trong F ng vi mt s nguyn dng m, mt biu din

.c

Zeckendorpf biu din l ging j, tr bt k cc s 0. Do , cc dy s nguyn dng, dn n cu tr li cho phn b) l c.

Chng 7 thi olympic Hungary


7.43.Tm tt c cc gi tr dng ca p bit rng tn ti cc s nguyn dng n, x,y tho mn pn = x3 + y 3

7.44.C hay khng mt a thc f bc 1999 vi h s l f(n) , f(f(n)) , f(f(f(n))), . . . l cc cp gi tr tng i vi bt k s nguyn n no? Li gii: Cho g(x) l a thc bt k bc 1997 vi nhng h s nguyn v cho f(x) =x(x-1)g(x) +1 . Ta chng minh rng nu f tho mn yu cu bi ton th mnh trn ng. Tc la phi ch ra rng vi n l s nguyn bt k v p l 1 gi tr phn tch thnh f(n), vy th p f k (n) vi bt k s nguyn dng k>1. c bit hn, ta chng minh rng f k (n) 1(modp)vi mi k> 1. Ta

Vy ch c p=2 v p=3 tha mn yu cu bi ton.

ht ch l mt trong x v y v p/(x+y). Theo pn = x 3 + y 3 . Ti (n,x,y) +(n-3, x/3, y/3). Nhng n < n tri vi gi thit n nh nht.

tch thnh (x + y)2 (x2 xy + y 2) = 3xy.V p khng chia ht cho3, p chia ht cho t nht 1 s hoc x hoc y. Hn na p khng th chia

Chn n, x, y sao cho n nh nht V p = 2 ta c (x, y) = (1, 1) . Do x2 xy + y 2 = (x y)2 + xy > 1 nh l (x + y). V c 2 gi tr ny u chia ht cho x3+y3, chng phi l nhng bi s ca p. Do p phn

.c

Li gii: p=2 v p=3 ta c: 21 = 13 +13 v 32 = 13 +23 Gi s rng p>3 v khng xy ra mu thun trong phngtrnh gm 3 s thc dng.

52

Nguyn Hu in, HKHTN H Ni t k chng minh bi ton. Bit rng cho mt a thc h vi cc h s nguyn a b(modc) tc l h(a) h(b)(modc). Xt trng hp k=2,

7.45.Chn ng cc ng phn gic ca cc gc tam gic ABC l X, Y v Z. ng trn ngoi tip tam gic XYZ ct 3 on thng AB, BC v CA. Chng minh rng tng di ca hai trong nhng on thng ny ph thuc vo di cnh th 3. Li gii: Ta t cc k hiu khong cch trong sut bi ton ti a=BC ,b=CA , c=AB l dng. ng thi, cho ng trn ct BC ti X v P, CA ti Y v Q, v AB ti Z v R, t x = PX, y=QY, z=RZ.

f (n) 0(modp) suy ra f (f (n)) f (0) 1(modp). Vy th f (f k(n)) f (1) 1(modp). iu phi chng minh.

ca 1 im cho im A ta c

Sau khi nhn cc v vi a/bc v sp xp li ta c

Khi cng 3 phng trnh li ta nhn ra iu n gin lx+ y +z = 0. Theo hai trong cc s hng x,y,z phi cng du v s hng th 3 phi tri du hai s cn li. Do tng gi tr tuyt i ca hai s hng trc bng tr tuyt i ca s hng sau, l iu phi chng minh. 7.46.Cho kv t l cc s nguyn t cng nhau ln hn 1. Bt u t php hon v (1,2,. . . .. n) ca dy s 1,2 . . . . . . ..n, ta co th i 2 s nu chng khng l k hoc t . Chng minh rng ta c th thc hin bt k php han v no ca 1,2,. . . n khi v ch khi n k + t 1

Li gii: Dng th G m cc nh ca n l cc s nguyn 1,2,. . . .n vi hiu ng bin gia a v b khi v ch khi |a b| {k, t} Ta thy iu

b z b+a c x b+c

Tng t ta c

c y c+a

a z b+a

.c

bc ( bc c+a c+a

o
= = =

+ y) =

Theo nh l ng phn gic ta c: YA=bc/(c+a) , AZ=bc/(a+b). Do , QA=bc/(c+a)+y , AR=bc/(a+b)-z. p dng nh l v phng tch
bc ( bc b+a ab

z)
abc (a+c)2

abc (b+a)2

+ +

b x b+c c y a+c

abc (b+c)2 abc (a+c)2

abc (a+b)2 abc (c+b)2

thi olympic Hungary

53

i 2 s trong cng 1 nhnh ca G, Theo khng s no c th v v tr c c thay th bi 1 s mt b phn khc. Vy th , ta khng

kin trva nu tng ng vi (i) mi php hon v c th thc hin c, (ii) G l ng lin; (iii) n k + t 1 (i) (ii) V mi bi trao

th thc hin c mi php hon v tr khi tt c cc s u nm trn mt nhnh. (i) (ii): Ta chng minh bng php quy np trn m rng xc nh mt th lin vi cc s nguyn m., bt k php hon v no ca cc s nguyn ny c th c c t php hon v khc bt k tao bi s trao i lin tip ca cp (a,b), ti a v b l cc nh lin k ca th. Yu cu r rng khi m = 1. Mt khc, chn mt nh a sao cho phn th cn li l ng lin sau khi b a v du, ta c th cho im A l im cui ca cc nh. Mt s qu o ca cc nh phn bit

m ng d m un k s c ni vi nhau ( cc cnh bin ngang qua ca dng {a, a + k} ) do ch ra rng tt c cc s 1,2,. . . .,k

c ni ln nhau. V t l ga tr tng ng vi k nn 1, 2t, 3t, . . . , kt biu din bi tt c cc ng d mun k. Do , ta c th sp xp li 1, 2, . . . ., k theo th t b1 , b2 , . . . , bk ti bi it(modk). Ch rng k 0 kt(modk) do bk = k Vy khi k 1 i 1 ta c k 1 bi

v v th nn n k + t 1 bi + t Do tn ti nh bi + t v c ni bi mt cnh bin ti bi Hn na, bi + t bi+1 (modk) nn bi + t

). Do , ta phi c k <n; tng t , t<n . Vy c n k cnh bin ca dng thc {a, a + k} v n-t ca cnh bin ca dng thc{a, a + t} S lin thng yu cu ita nht n -1 cnh bin, do (a k) + (n t) n1 n k+t1 (iii) (ii): Tt nhin trong trng hp ny k,t < n. ch rng 2 s

(ii) (iii): Nu t nht k l n th mi cnh bin s ni 2 bin s ng d theo mun t. Vy s khng c ng no gia 1 v 2 (mu thun

chim ch bi 1 (a) Bng gi thit quy np, cc bin s thay th a c th hon v khi cn, do t c hon v . p dng b ny vi m=n v th G chng minh rng (ii) (i)

.c

a0 a1 . . . ar ni lin a0 = 1 (a) v ar = a Bng cch chuyn v tr lin tip (a0 a1 ), (a1 a2 ) . . . (ar1 ar ) ta c th di chuyn a ti v tr ban u

54

Nguyn Hu in, HKHTN H Ni c ni ti bi+t . V th cho nnbi , c ni ti bi+1 trong G vi mi i = 1, 2, . . . , k 1. Vy cc s ny c ni lin ln nhau v bi ton

c gii quyt.

7.47.Cho s nguyn dng bt k k, cho e(k) l s c dng chn ca k. v cho o(k) l s c dng l ca k. Vi n>1, chng minh rng : n e(k) k=1 v
n k=1

o(k) khc nhau vi hu ht n.

1,2. . . ,n bng vi tng ca |n/d| n khong tt c cc s nguyn dng n chn d. V [ n ] [ d+1 ] v s nguyn dng a v n, ta c d
n k=i

Li gii: S nguyn c th chia c bi d nm chnh gia dy s 1,2,. . . ,n l |n/d| Do , tng ca o(k) (rerp.e(k)) n khong k=

o(k)

n k=i

e(k) =

n k=i ([ 2i+1 ]

n [ 2i ])

t:

Ti tng hn c xc nh v cc s hng bng 0 khi i> [ n ]. Tng 2


n k=i n k=i

sin BAC. Tng t sin ZP X = sin CBA v sin XP Y = sin ACB. Tng ng vi cc iu sau P l trng tm tam gic XYZ Cc tam gic YPZ,ZPX, XPY c din tch bng nhau

c=AB. Gi P l mt im nm trong tam gic, v tr xc nh sau. Gi X, Y, Z l chn cc ng vung gc k ti cc cnh BC, CA, AB, theo th t , x =PX, y= PY, z=PZ. Chu rng sin Y P Z = sin( BAC) =

yz sin Y P Z = zx sin ZP X = xy sin XP Y sin<BAC/x = sin<CBA/y = sin<ACB/z a/x = b/y = c/z (nh l hm Sin)

Dng ng thng song song vi BC, cch BC mt khong l a nm

Li gii: a: Gi tam gic l ABC vi di cc cnh l a=BC, b= CA,

gc qua P ti cc nh ca tam gic. Chn ca cc ng vung gc to thnh mt tam gic nhn P l trng tm.

7.48.Cho mt tam gic trong khng gian, hy nu cch dng im P nm bn trong tam gic tho mn iu kin sau: nu h mt ng thng vung

.c

o(k)

e(k) = [ n ] 1

n k=i ([ 2i ]

n [ 2i+1 ])

thi olympic Hungary

55

cng pha vi A. Tip tc dng n thng song song vi CA, cch CA mt khong l b, nm cng pha vi B. t Q l giao im ca chng,ch rng tia CQ i qua min trong tam gic Ly P bt k trn CQ, xt t s khong cch ti BC vi khong cch ti AB. Nu P=Q t s ny bng a/b ;bi v mi im P l hnh nh ng dng ca nhau vi C,nn t s ny khng ph thuc vo P v lun bng a/b. Hn na ta c th dng 1 tia t A hng vo tam gic m vi mi P trn tia th t s k/c t AB vi k/c t CA bng c/b. Hai tia ny giao nhau ti im P no trong tam gic . Nu ta t P l giao im ca chng, ta thu c a/x = b/y v b/y = c/z ,v bi ton c li gii.

.c

Chng 8 thi olympic India


8.49.Cho ABC khng u. Gi P l mt im nm trong tam gic . Cc on thng qua P v ni nh vi cc cnh i din ca tam gic

cng phi nm trong ABC. By gi ta dng di hnh hc. Ch rng

2 2 2 BD.BD, = (BHa +Ha D)(BHa Ha D) = BHa = (AB 2 AHa )(AD 2 AHa )

gic th ta suy ra c l AB hoc AC phi nm trong ADD , . Th nhng iu ny khng xy ra v AD = AD , = < min(AB, AC). , , , ABC, tng t nh vy BE , v CF Do vy AD phi nm trong

Ly i xng cc on thng trn qua cc ng cao tng ng ta , , , , thu c cc on AD , BE , v CF . Nu AD khng nm trong tam

Li gii: Xt 3 on l AD, BE, v CF , gi cc ng cao ca tam gic l AHa , BHb , v CHc .

.c

c di bng nhau v bng tha mn < min(AB, BC, CA). Chng minh rng c im P , = P tha mn cc tnh cht ging nh P.

Tng t nh vy ta c EA.EA, = AB 2 2 Tht vy, EA.EA, = BD.B , D. Nh vy F B.F , B = CE.CE , v DC.D,C = AF.A, F

= AB 2 AD 2 = AB 2 2 (8.1)

thi olympic India

57

By gi p dng nh l Ceva cho 3 ng ng quy AD, BE, v CF AF ta c BD . CE . F B = 1 DC EA Do vy BD , CE , AF , BD, BD CE , CE AF , .AF . , . , =( , ).( ).( ) D,C E A F B D C.DC E , A.EA F , B.F B BD , .BD CE , CE AF , .AF =( , )( )( ) = 1 (8.2) E A.EA F , B.F B D , C.DC Nhng theo nh l Ceva AD , BE , v CF ng quy ti im P , nm ngoi ABC. Nu P trng vi P , th P s l trc tm ca tam gic, nhng nu vy th theo gi thit 3 ng cao ca ABC c di bng nhau v bng .. iu ny l v l, v ABC khng u. Do P , = P , v cc on thng qua P , c di bng nhau v bng . t ca m u nh hn hoc bng n. Hy chng minh rng m l c ca n! Li gii: Ta ch ch cn chng mnh rng vi mi c nguyn t p ca m v vi mi s nguyn k 1 m P k |m th pk |n! (1) 8.50.Cho m, n l cc s nguyn dng tha mn m
n2 4 , , ,

dng BT Bernoulli ta c p 2 = (1 + (p 1)) 2 1 + k2 .(p 1) k . 2 2 Cui cng, nu k = 3 th (*) lun ng ngoi tr p = 2. Nu k = 3 v


k2 k2

chng minh n kp ta i chng minh 2 p k p 2 k (*) (V 2 k kp) khi n 2 p i chng minh (*): Vi k = 2 th (*) 1 1 (ng). Nu k 4 th p

ca p, ko theo pk |n!.

Tht vy nu k = 1 thy ngay (1)ng v p n. v do vy p|n! . Mt 2 2 khc v m n nn ta c pk n . hay n 2 pk . 4 4 Ta thy, nu n kp th t nht k s thuc vo tp 1, 2, ....., n l bi

.c

m
pk

v mi c nguyn

p = 2 th m 8; n 5 v n! thc s chia ht cho 8 (8 = 23 ), do vy ta c pk l c ca n! trong mi trng hp, bi ton c chng minh. 8.51.Cho G l mt hnh vi n nh vi n 4 v m cnh. Chng minh rng nu m > n( 4n 3 + 1)/4 th G cha mt 4- Chu trnh. Li gii: Chng ta m s cc b 3 cc khong cch gia cc nh (v, a, b) sao cho v l cnh k c a v b. S cc b nh th vi mi v c nh l

k2

58

Nguyn Hu in, HKHTN H Ni deg(v).(deg(v)-1). V tng cc deg(v) ly theo tt c cc v l 2m, v x(x1) l hm li theo x, bt ng thc Jensen cho ta tng deg(v)(deg(v)-1) ly theo tt c cc v b nht l n.(2m/n)((2m/n)1) = 2m, (2m/n1). Nu G khng c 4-chu trnh no, th vi bt k a, b c nh no c nhiu nht mt nh k vi c a v b. Ko theo c nhiu nht n(n-1) b 3 ni trn. Do vy, c mt 4- chu trnh nu 2m, (2m/n1) > n(n1)

4m2 (2n)m n2 (n 1) > 0. Bt ng thc ny lun ng i vi m ln hn nghim ln ca phng trnh 4x2 (2n)x n2 (n 1) iu ny c ngha l G c 4-chu trnh nu nh m ln hn nghim ln ca phng trnh bc 2 trn, hay m > 2n+ 4n +16n 8 thit cho ta iu phi chng minh.
2

3 16n2

m > n.

4n3+1 . 4

T gi

8.52.Cho hm f : Q {0, 1} tha mn f(x) = f(y) th f (x) = f ( x+y ) = f (y) 2 vi x, y Q, Nu f(0)=0 v f(1)=1 hy chng minh f(q) = 1 vi q Q

gi thit quy np, f (x1 ) = f (y1 ). Ta cn cn chng minh f (x2 ) = f (y2 ). Tht vy gi swr f (x2 ) = f (y2) khi xt (x, y) = (x1 , y1 ) v (x, y) = (x2 , y2 ) vo trong iu kin bi, ta c f (b) = f ( x1+y1 ), 2 v f (a) = f ( x2 +y2 ) . Tuy nhin iu ny khng xy ra v x1 + y1 = 2 x2 + y2 . Do f (y2 ) = f (x2 ) hay f (y2 ) = f (a).

Vy f (y2 ) = f (b). chng minh xong b . p dng b trn vi a = 0 v b = 1 ta thy rng f(n)=1 vi mi n nguyn dng. Tip theo ta thy f (1 + r ) = 0 r, s N, s v nu ngc li, p dng b vi a = 1, b =1 + r v n = s th s f(1+r) = 0 mu thun vi f(n) = 1 trn. Do vy, f(q) = 1 vi mi s hu t q 1.

Vi n = 1 (1) hin nhin ng

Gi s mnh ng vi n k. t (x1 , y1 , x2 , y2 ) = (b, k(b a) + a, a, (k + 1)(b a) + a). Theo

(1) Tht vy, ta s chng minh b trn bng quy np.

.c

Li gii: Trc tin ra chng minh b sau: Cho a, b l cc s hu t. Nu f (a) = f (b) th f(n(b - a) + a) = f(b) vi mi n nguyn dng

Ti liu tham kho


[1] Titu Andreescu, Zuming Feng, and George Lee, Jr. Mathematical Olympiads 20002001, Problems and Solutions From Around the World, The Mathematical Association of America, 2002. [2] Nguyn Hu in, Phng php irichle v ng dng, NXBKHKT, 1999. [3] Nguyn Hu in, Phng php Quy np ton hc, NXBGD, 2000. [4] Nguyn Hu in, Nhng phng php in hnh trong gii ton ph thng, NXBGD, 2001. [5] Nguyn Hu in, Nhng phng php gii bi ton cc tr trong hnh hc, NXBKHKT, 2001. [6] Nguyn Hu in, Sng to trong gii ton ph thng, NXBGD, 2002.

NXBHQG, 2004.

[9] Nguyn Hu in, Gii ton bng phng php i lng bt bin, NXBGD, 2004.

[8] Nguyn Hu in, Gii phng trnh v nh nghim nguyn,

[7] Nguyn Hu in, a thc v ng dng, NXBGD, 2003.

.c

Nguyn Hu in

OLYMPIC TON NM 2000 (Tp 2)


t
NH XUT BN GIO DC

.c

49 THI V LI GII

v n m a t h .c o m

Li ni u
th gi lnh lamdethi.sty ti bin son mt s ton thi Olympic, A m cc hc tr ca ti lm bi tp khi hc tp L TEX. ph v cc bn ham hc ton ti thu thp v gom li thnh cc sch in t, cc bn c th tham kho. Mi tp ti s gom khong 50 bi vi li gii. Tp ny c s ng gp ca Trnh Quang Anh, Nguyn Th Bnh, Nguyn Th Thanh Bnh, o

c thi gian sa li, mong cc bn thng cm.

Rt nhiu on v mi hc TeX nn cu trc v b tr cn xu, ti khng

ti liu ting Vit v ch ny, ti c xem qua v ngi dch l chuyn v ngnh Ton ph thng. Bn c th tham kho li trong [1].

Rt nhiu bi ton dch khng c chun, nhiu im khng hon ton chnh xc vy mong bn c t ngm ngh v tm hiu ly. Nhng y l ngun

H Ni, ngy 2 thng 1 nm 2010 Nguyn Hu in

51 89/176-05 GD-05

.c

th Kim Cc, Nguyn Hong Cng, Gip Th Thy Dung, Mai Xun ng, Hong H, Nguyn Th Thanh H.

M s: 8I092M5

Mc lc
Li ni u . . . . . . . . . . . . . . . . . . . . . . . . . . . . . . . . . . . . . . . . . . . . Mc lc . . . . . . . . . . . . . . . . . . . . . . . . . . . . . . . . . . . . . . . . . . . . . . . 3 4 5 9 14 18 24 32 39 45 50

Chng 8. thi olympic i Loan . . . . . . . . . . . . . . . . . . . . . . . Chng 9. thi olympic Th Nh K . . . . . . . . . . . . . . . . . . . .

Chng 7. thi olympic Nc Nga . . . . . . . . . . . . . . . . . . . . . .

Chng 6. thi olympic Rumani . . . . . . . . . . . . . . . . . . . . . . . .

Chng 5. thi olympic Mng c . . . . . . . . . . . . . . . . . . . . . . .

Chng 4. thi olympic Korea . . . . . . . . . . . . . . . . . . . . . . . . . .

.c

Chng 3. thi olympic Nht Bn . . . . . . . . . . . . . . . . . . . . . .

Chng 2. thi olympic Italy . . . . . . . . . . . . . . . . . . . . . . . . . . . .

Chng 1. thi olympic Israel. . . . . . . . . . . . . . . . . . . . . . . . . . . .

Chng 1 thi olympic Israel


m m a
nn phn tun hon ca phn thp phn khng th ch bao gm ton s khng. Tuy nhin, n ln, s cc s 0 cha trong f(n) tin ti v cng, v vy phn tun hon ca phn thp phn phi cha ton s 0 mu thun. V vy a khng l s hu t. 1.2. . ABC nh l nhng im nguyn. Hai trong ba cnh c di thuc tp 17, 1999, 2000 . Tm gi tr ln nht c th ca din tch ABC. Li gii: Khng mt tng qut, gi s cnh AB, BC c di thuc 17, 1999, 2000 th SABC = 1 AB.BC sin BCA 1 2000 2000 sin = 1000. 2 2 2 1.1. nh ngha f (n) = n!. Cho

a = 0.f (1)f (2)f (3).... Ni cch khc, thu c s biu din phn thp phn ca a vit cc biu din thp phn ca f (1), f (2)., ... trong mt hng, a c phi l s hu t khng?

Li gii: Nu a l s hu t th cc con s trong phn thp phn phi xut hin mt cch tun hon. V f(n) lun bao gm mt s khc khng,

.c

Nguyn Hu in, HKHTN H Ni ng thc c th xy ra, chng hn trong m nh l (0,0); (44,8) v (-8, 44) chnh xc 2 cnh di 2000 v 442 + 82 = 2000 v gc gia 2 cnh l . T , din tch ln nht ca l 1000. 2 1.3. Bi ton 3.Cc im A, B, C, D, E, F nm trn 1 ng trn v cc ng thng AD, BE, CF ng quy. Ly P, Q, R l cc trung im cnh AD, BC, CF tng ng. 2 on (dy cung) AG, AH c v sao cho AG // BE v AH//CF chng minh rng PQR v DGH ng dng. Li gii: Cc gc nh hng mun . Gi s on thng AD, BE, CF ng quy (ct nhau) ti X v O l tm ng trn cho bi. Hin nhin OP X = OQX = ORX = , suy ra O, P, Q, R v X cng 2 thuc 1 ng trn. V vy DGH = DAH = DXC = CXP = RXP = P QR Tng t DGH = P RQ, t suy ra PQR DGH.

Li gii: Ta a ra 1 v d v php c tam gic tt vi 8 tam gic. t hng hnh vung sao cho on AB t nm ngang v A l nh trn bn tri. Ly M v N l cc trung im cnh AB v CD tng ng, v P l 1 im trung on MN khc trung im MN. Cc gc MP A, AP D v DP N v cc gc phn x ca chng qua MN - tt c u l cc gc nhn. Ta chn Q, R trn ng thng nm ngang qua P sao cho Q, P, R nm theo th t t tri qua phi v QP, PR c di rt nh (khng ng k) chia hnh vung thnh cc bng cch v on QA, QM, QN, QD, RB, RM, RN, RC v QR. Nu ta chn Q sao cho PQ nh th s o cc gc MQA, AQD, DQN s gn bng s o gc MP A,

a. Cho 1 v d v php c tam gic tt ca hnh vung. b. Tm s nh nht ca cc tam gic cn c mt php c tam gic tt?

gic khc). Mt php c tam gic tt ca 1 hnh vung l php c trong mi tam gic u nhn.

u c tch ri, ch chung 1 nh hoc chung nhau ch 1 cnh c th. Khng nh no ca 1 tam gic c th nm phn trong ca cnh tam

.c

1.4. Mt hnh vung ABCD cho trc, mt php c tam gic ca hnh vung l 1 s phn chia hnh v thnh cc tam gic sao cho bt k 2 tam gic

thi olympic Israel AP D, DP N, v vy nhng tam gic ny s nhn.

Tng t, nu chn R sao cho PR nh th MRB, BRC, CRN s cng nhn. D kim tra rng cc gc trong s phn chia trn l nhn nh yu cu. b.Ta s chng minh s nh nht l 8. Ta ch ra rng 8 l gi tr c th thc hin c. V vy, ch cn ch ra nhng php c tam gic tt no vi t hn 8 tam gic. Nhn xt rng trong 1 php c tam gic tt, mi nh ca ABCD l nh ca t nht 2 tam gic bi v gc vung phi c chia thnh cc gc nhn. Nh vy, bt k nh no nm trn cnh ABCD phi l nh ca t nht 3 tam gic v bt k nh nm phn trong phi l nh ca t nht 5 tam gic. Tm li, ta c th chng minh mt kt qu mnh hn v mi gc ca

nh trong cc nh trong php c tam gic m nm bn trong hnh vung ABCD. Theo trn i 1. Trc tin gi s rng c mt nh trong P. Kt qu ca on trc cho ta: on PA, PB, PC, PD phi l cc cnh ca cc tam gic trong php c tam gic. Mt trong gc AP B, BP C, CP D, DP A phi ln hn gi s l AP B. Gc ny phi c 2 chia ra trong php c tam gic ny bng cnh PQ no , vi Q l im thuc on AB. Nhng c AQP v BQP c s o t nht l nn 2 Q phi nm trong cnh ca tam gic no m khng nm trong on QA, QB hoc QP. Tuy nhin khng th to c mt cnh m khng ct AP hoc BP v cnh khng kt thc mt nh trong th hai.

: mu thun. Ta xt 1 php c tam gic tt bt k ca ABCD. Ly i l s ca cc

V vy, c 1 im Y trong ABX sao cho AXY l mt thnh phn C phn t ca php c tam gic tt. Nhng nu vy AY X ABX = 2

BC (khng trng B). Bng nh ngha ca php c tam gic : khng c nh khc ca mt tam gic trong php c tam gic nm trn on AX.

.c

mt tng qut, gi s gc (nh) l A. Cnh AX no ca tam gic chia gc vung ti A ra.Gi s phn chng rng X khng nm phn trong hnh vung ABCD, khng mt tng qut, gi s X thuc on

hnh vung ABCD. Phi c mt tam gic m cnh bt u t nh hnh vung v im cui nm trn phn trong hnh vung ABCD. Khng

Nguyn Hu in, HKHTN H Ni Gi s tip i 2. Vi mi mt n cc tam gic, ta c th m 3 cnh c tng 3n; mi cnh nm trn bin hnh vung c m 1 ln, cc cnh khc c m hai ln. Nu i = 2 th vi mi 2 im trong t nht 5 cnh tam gic nhn im lm im cui, nhiu nht 1 cnh tam gic cha c hai nh trong, nn t nht 9 cnh tam gic khng nm bin ca hnh vung. Nu i 3, ly bt k 3 nh trong. Mi nh thuc

t nht 5 cnh tam gic v nhiu nht 3 cnh tam gic cha 2 trong 3 nh . V vy t nht 3 x 5 3 = 12 cnh tam gic. Khng thuc bin hnh vung. Trong c hai trng hp u c t nht 9 cnh tam gic khng thuc bin hnh vung, v hn na li c 4 cnh tam gic thuc bin hnh vung. V vy 3n9 x 2 + 4 = 22 hay n 8. V vy trong

mi trng hp phi c t nht 8 tam gic tho mn yu cu.

.c

Chng 2 thi olympic Italy


m
2.5. Gi s ABCD l mt t gic li, vi = DAB; = ACB; = DBC; v = DBA. Gi thit rng < /2, + = /2 v + 2 = , chng minh rng

h
B

(DB + BC)2 = AD 2 + AC 2 .

.c

Li gii: Gi s D l im i xng ca D qua ng thng AB. Ta c D BA = DBA = , nn D BC = D BA + ABD + DBC = 2 + = . Vy, D , B, C l thng hng. Cng c AD C + ACD = ADB + ACB = + = /2, nn D AC = /2 v tam gic A AC vung. Theo nh l Pythagorean, D C = AD 2 + AC 2 , ko theo (DB + BC)2 = (D B + BC)2 = D C 2 = AD 2 + AC 2 = AD 2 + AC 2 ,

10 c iu phi chng minh.

Nguyn Hu in, HKHTN H Ni

2.6. Cho s nguyn c nh n > 1, Alberto v Barbara chi tr chi sau, bt u vi bc u tin v sau xen k gia ln th hai v ln th ba : Alberto chn mt s nguyn dng. Barbara chn mt s nguyn ln hn 1 l mt bi hoc c ca s

nguyn ca Alberto, c th chn ng l s nguyn ca Alberto. Alberto cng hoc tr 1 t s ca Barbara. Barbara chin thng nu c y chn ra n vi 50 ln chi. Vi gi tr no ca n c y l ngi thng cuc.

Li gii: Mc ch ca chng ta l Barbara l ngi thng cuc nu v ch nu t nht l mt iu kin sau c tha mn :

Trng hp 1 : (a) n = 2, trong trng hp ny Barbara thc s chin thng. (b) 4| n. Nu a1 = 1, th Barbara c th chn b1 = n v chin thng. Ni cch khc, a1 = 3, Barbara c th chn b1 = 3, a2 bng 2 hoc 4, v Barbara c th chn b2 = n. (c) C s nguyn m > 1, (m2 1)| n. Nh trng hp 2, Alberto phi chn a1 = 3 ngn Barbara thng cuc. Thc t, c ng mt s nguyn trong cc s m 1, m v m + 1 chia ht cho 3, ngha l hoc 3 chia ht m hoc 3 chia ht m2 1v v v 3 chia ht n. Trong trng hp u tin, Barbara c th chn b1 = m, bt buc a2 = m 1 v ko theo Barbara chn b2 = n. trong trng hp

s a1 , b1 , a2 , b2 , . . . l cc s c chn sau khi Barbara chn 2 cho ln chn u tin.

a = n, c y chin thng; ni cch khc, ln chn th hai ca Alberto phi l s chn, v Barbara c th chn s 2 trong ln chn th hai. Gi

mt s chn th Barbara c th chn 2 trong ln u tin. Nu thay a bng mt s l, th Barbara c th chn chnh l s a l tt nht. Nu

.c

c s nguyn m > 1, sao cho (m2 1)| n. u tin chng ta ch ra rng khi v ch khi ba iu kin ny l ng, th Barbara l ngi chin thng. Nu Barbara chn ln u tin a l

n = 2; 4| n ;

thi olympic Italy sau, Barbara c th chn b1 = n.

11

By gi chng ta thy rng Barbara c mt chin thut chin thng nu t nht mt trong cc iu kin l ng. By gi chng ta gi thit rng khng iu kin no l ng vi n > 1 v chng minh rng Alberto c th lun lun ngn cn Barbara tin n chin thng. Bi v iu kin th nht v th hai l khng ng v bi v iu kin th hai l sai vi m = 2, chng ta c n = 2, 3, 4,. Vy n > 4. Gi mt s nguyn dng n l s hy vng nu a|n v n|a. Chng ta chng minh rng vi s nguyn b > 1, tn ti a {b 1, b + 1} sao cho

a l s hy vng. u ny ko theo rng Alberto c th bt u chn mt vi s hy vng v cng chn mt vi s hy vng theo sau ngn

v p(b) = 2001. Xc nh cc gi tr c th ca p(1).

Li gii: Nu p(x) = 2000x2 x, th p(0) = 0, p(1) = 1999, v p(1) = 2001. Nu p(x) = 2000x2 +x, th p(0) = 0, p(1) = 2001, v p(1) = 1999. Do , c th p(1) = 1999 hoc 2001. By gi gi s rng p(1) = 1999, 2001. Th a, b = 1. Bi v p(0) = 0, chng ta vit c p(x) = xq(x) vi a thc q(x) c h s nguyn. Bi v q c h s nguyn, q(a) l mt s nguyn, v c th vit q(x) q(a) = (xa)r(x) vi a thc r(x) c h s nguyn. V bi v r c h s nguyn, r(b) l mt s nguyn, v chng ta c th vit r(x) r(b) = (x b)s(x)

2.7. Gi s p(x) l mt a thc vi h s nguyn sao cho p(0) = 0 v 0 p(1) 107 , v sao cho tn ti cc s nguyn a, b tha mn p(a) = 1999

v b + 1 l l. iu ny ko theo chng nguyn t cng nhau v tch ca chng b2 1 chia ht n, mu thun vi gi thit iu kin th ba l sai.

chng ta phi c (b 1)|n v (b + 1)|n. Nu b 1 v b + 1 l chn, th mt trong chng phi chia ht cho 4 - nhng 4| n, mu thun. Vy, b 1

.c

2, mu thun. Ni cch khc, b n. Bi v n khng chia ht n + 1 hoc n + 2 vi n > 2,

cn Barbara tin n chin thng sau 50 ln. Gi s v iu kin l mc ch trn l sai vi s nguyn b > 1. Nu b > n, th b 1 v b + 1 phi l bi ca n. Do n chia ht hiu, tc l

12

Nguyn Hu in, HKHTN H Ni vi a thc s c h s nguyn. Do : p(x) = xq(x) = xq(a) + x(x a)r(x) = xq(a) + x(x a)r(b) + x(x a)(x b)s(x). c bit, khi cho x = a v x = b, chng ta tm c 1999 = aq(a) 2001 = bq(a) + b(b a)r(b). Bi v p(0), p(a) v p(b) l cc s phn bit, v vy 0, a v b cng phn bit. Do , chng ta c th gii hai phng trnh trn tm c 1999 a 2001 bq(a) . r(b) = b(b a) (*)

C nh (a, b) l chung cho ba cp trn. T (*) chng ta bit rng q(a) 2001 b q 1999 v r(b) phi bng r = . Cho x = 1 v (*) phi bng q = a b(b a) tm p(1) : (a, b) (1999, 2001) (1, 3) (1999, 2001) q(a) 1 1 r(a) 0 p(1) 1 + (2000.2002)s(1) 3331 + 8s(1) 1 + (1998.2000)s(1).

(1999, 2001), (1, 3), (1999, 2001).

Vi hn ch ny, kt hp vi iu kin |a b| {1, 2}, b|2001, a = 1, v b = 1, ko theo rng (a, b) bng mt trong cc cp sau :

1 hoc 2. Cng vy, vi mi x Z, chng ta c p(x) = xq(x) v v vy x| p(x). c bit, a| 1999, cho nn

Bi v a = b, chng ta c |a b| chi ht p(a) p(b). V vy |a b| bng

1999 666 0

|a| {1, 1999}.

.c

q(a) =

(*)

thi olympic Italy

13

V vy, p(1) c dng m + ns(1) vi s nguyn c nh m, n. Tht vy, gi s rng c mt s dng m + n gia 0 v 107 , t s l s nguyn. s Chng ta c p(x) = q x + rx(x a) + sx(x a)(x b), chng ta c p(0) = 0, p(a) = 1999, p(b) = 2001, v p(1) = m + n. s Do , cc gi tr c th ca p(1) l 1999 v 2001, v cc s gia 0 v 107 ng d vi 1 (mod 2000.2002), 3331 5 (mod 8), hoc 1 (mod 1998.2000).

.c

Chng 3 thi olympic Nht Bn


m
3.8. Ta tro mt lot cc l bi nh s a1 , a2 , . . . , a3 n t tri qua phi bng

Bt u vi 192 qun bi dnh s 1, 2, . . . , 192 t tri qua phi, liu ta c c trt t 192, 191, . . . , 1 sau s ln tro hu hn? Li gii: Vi mi n, cho f (n) l v tr trong chui cc qun bi qun bi i v tr th n sau mi ln tro. Ta thy rng sau k ln tro, f k (n) v tr th n. Ta c bit rng f (1), . . . , f (192) bng 3, 6, . . . , 192, 2, 5, . . . , 191, 1, 4, . . . , 190. Trong trt t ny, s khc bit gia bt k s hng no vi s hng ng trc n l ng d t 3 ti modul 193. V f (1) 3 (mod 193) ta c f (n) 3n (mod 193) vi mi n. Trong trt t (33 )20 , (33 )21 , . . . , (33 )26 , vi mi s hng l bnh phng ca s hng trc n. t nht mt s hng trc n (s hng u tin 2t) khng ng d vi mt modul 193; gi s N = 3d ( d l s nguyn

1, 2, 3, 4, 5, 6 3, 6, 2, 5, 1, 4 2, 4, 6, 1, 3, 5

V d nu 6 l bi c nh s 1, 2, . . . , 6 t tri qua phi th vic tro chng 2 ln s thay i trt t ca chng nh sau:

.c

a3 , a6 , . . . , a3 n, a2 , a5 , . . . a3n1 , a1 , a4 , . . . , a3n2

vic sp xp cc l bi theo th t mi:

thi olympic Nht Bn

15

khng phi l s hng cui cng trong trt t ny. Do vy, N 2 s hng tip theo ca N trong trt t l ng d vi 1 modul 193. V 193 chia c cho N 2 1 nhng khng chia c cho N - 1, n phi chia ht cho (N 2 1)(N 1) = N + 1 = 3d + 1, c ngha l 3d 1 (mod 193) .

dng) l gi tr ln nht vi thuc tnh ca n, v 193 l s nguyn t, theo nh l Fermat c: (33 )26 (33 )192 1 (mod 193), do vy 3d

Vin = 1, 2, . . . , 193 ta c f d (n) 3d n n (mod 193). Do vy f d (n) = 193 n, c ngha l trt t 192, 191, . . . , 1 xut hin sau d ln tro. Ch : Gi tr d tm thy trn thc t l bng 24. S nguyn dng k nh nht tha mn 3k 1 (mod 193) l 8, c ngha l trt t 192, 191, . . . , 1 xut hin ln u tin sau 8 ln tro bi. im no thng hng. Chng minh rng: 3.9. Trong mt phng cho cc im phn bit A, B, C, P, Q, khng c 3

AB + BC + CA + P Q < AP + AQ + BP + BQ + CP + CQ.

Li gii: Trong bi gii ny, khi gi mt a gic V1 . . . Vn l li nu V1 , . . . , Vn to thnh mt a gic li trong trt t . (V d nu ta ni hnh vung ABCD l li th ta khng ni rng ACBD l li.) Ta ni rng iu kin (a) c nh nu t gic XYPQ l li vi X, Y

thc ny ta c:

rng t gic ABPQ l li. Nu AP giao vi BQ ti O, th bt ng thc tam gic cho ta AB AO + BO v P Q P O + QO. Cng 2 bt ng AB AO + BO + OP + OQ = AP + BQ

V khng c 3 trong s cc im cho no thng hng nn bt ng thc tam gic cng ch ra rng BC < BP + P C v CA < CQ + QA Cng 3 bt ng thc cui cng ny ta c kt qu cn chng minh. Tip n ta ni ti iu kin (b) c nh, nu xem X nm trong tam gic YZM vi s hon v (X, Y, Z) ca (A, B, C) v vi M {P, Q}. Ta chng

{A, B, C}. Trong trng hp ny ta chng minh bt ng thc cn chng minh l c nh. Khng mt tnh tng qut, ta c th gi thit

.c

16

Nguyn Hu in, HKHTN H Ni minh rng bt ng thc cn chng minh c nh trong trng hp ny. Khng mt tnh tng qut, gi s A nm trong tam gic BCQ. S chuyn im P ty ti mi cnh PB, PC l cc hm li ngt, c ngha l P P B + P C cng l mt hm li ngt. Do vy, trn tt c cc im

P hoc trong tam gic BCQ hm ny ch t cc i khi P trng vi B, C hoc Q. Vy nn: AB +AC < max{BB +BC, CB +CC, QB +QC} = QB + QC cng bt ng thc ny vi bt ng thc BC < BP + PC v PQ < PA + QA, c bt ng thc tam gic ta c kt qu cn chng minh. Do vic i tn cc im, bao li ca 5 im cho hoc phi l tam gicBC, hoc ABP, hoc APQ, hoc t gic li ABCD, hoc ABPQ, hoc APBQ, hoc ng gic li ABCPQ hoc ABPCQ. Nu tam gic ABC l bao li th Q phi nm pha trong mt trong cc

PQ so vi B m khng mt di tnh tng qut. Vy nn iu kin (b) c nh. Nu t gic ABCP l bao li th Q nm trong tam gic APB hoc trong CPB. Trong trng hp u t gic BCPQ l li v trong trng hp th hai t gic BAPQ l li. Vy nn iu kin (a) c nh. Nu t gic li ABPQ, ng gic li ABCPQ hay ng gic li ABPCQ l bao li th t gic ABPQ l li v iu kin (a) c nh. Cui cng, nu t gic li APBQ l bao li th C hoc nm trong tam gic ABP hoc ABQ; c trong 2 trng hp iu kin (b) c nh. Do vy, trong tt c cc trng hp, hoc iu kin (a) hoc iu kin (b) c nh, t suy ra bt ng thc cn chng minh l ng.

iu kin (b) c nh. Nu tam gic APQ l bao li th ta c th gi thit C khng gn hn

giao vi P B. Nu tam gic ABP l bao li th C phi nm trong tam gic ABP v

nhng nm cng pha ng AB so vi Q, do vy QC phi giao vi mt trong 2 on thng AP v P B. Nu QC giao vi AP , th t gic ACPQ l li v iu kin (a) c nh; tng t iu kin (a) c nh nu QC

.c

tam gic APB, BPC, CPA. Khng mt i tnh khi qut gi thit rng Q nm trong tam gic APB. V C khng nm bn trong tam gic APB

thi olympic Nht Bn

17

3.10.Cho 1 s t nhin n 3, chng minh rng tn ti 1 tp hp An vi 2 thuc tnh sau: (i) An bao gm n s t nhin ring bit. (j) Vi bt k a An , tch s ca tt c cc phn t khc trong An c

s d l 1 khi c chia bi a.

s nguyn a1 , a2 , . . . , ak+1 vn l cc s nguyn ring bit ln hn 1. Xem xt biu thc a1 a2 . . . ai1 ai+1 . . . ak+1 (mod ai )

mn a1 a2 . . . ak (mod ai ) khi 1 i k. Gi s {1, 1} v xc nh aa+1 = a1 a2 . . . ak . V ak+1 2ak 1 > ak vi tt c cc k, cc

Li gii: Gi s a1 , a2 , . . . , ak (vik 2) l cc s nguyn ring bit ln hn 1 tha

= 1 v mt ln tp hp = 1. Tp hp An bao gm cc s kt qu l a1 , a2 , . . . , an do tha mn iu kin u bi.

Bt u vi cc s a1 = 2, a2 = 3, ta p dng cch ny n-3 ln tp hp

.c

(a1 a2 . . . ai1 ai+1 . . . ak )ak+1 (1)( )

r rng n khng i vi i = k + 1. Vi i < k n khng i v (mod ai )

Chng 4 thi olympic Korea


m

Tip theo, gi s (1) khng l ng d bnh phng modunp. Ta phi tm mt s k no c k v p k 1 u l ng d bnh phng. Nu p1 l ng d bnh phng th chn k = p1 . Nu ngc li, th 2 2

l Pigeonhole Principle s c mt cp (k, p k 1) m c hai s k v (p k 1) u l ng d bnh phng nh ta nh tm.

mi ng d trong s p1 cc ng d bnh phng khc khng s ri 2 vo trong cc cp {1, p 2} , {2, p 3} , ..., p3 , p+1 . Theo nguyn 2 2

x2 + y 2 + z 2 < p2 . Gi tr s c xc nh nh trng hp trc.

V vy, ta c th chn x, y 0, 1, ..., p1 sao cho x2 k(modp) v 2 2 y p k 1(modp). Cho z = 1, ta c x2 + y 2 + z 2 chia ht cho p v

chia ht cho p nhng 1 + (p 1)2 < p2 nn tn ti {1, 2, ..., p 1} sao cho x2 + y 2 + z 2 .p = 0 .

Li gii: Vi trng hp p = 2, ta c th ly x = 0, y = z = = 1. By gi ta xt trng hp p > 2. Trc tin ta xt trng hp 1 l ng

d bnh phng modun p, khi tn ti mt s t nhin a, 0 < a < p1 sao cho a2 1(modp). B (x, y, z) = (0, 1, a). V x2 + y 2 + z 2 = a2 + 1

.c

4.11.Ch ra rng vi mi s nguyn t cho trc p th tn ti nhng s t nhin x, y, z, tho mn x2 + y 2 + z 2 .p = 0 v 0 < < p.

thi olympic Korea 4.12.Tm tt c cc hm f : R R tho mn f (x2 y 2) = (x y) [f (x) + f (y)] vi mi x, y R. Li gii: Cho x = y, ta c f (0) = 0. Cho x = 1, y = 0 ta c f (1) = f (1).

19

Cho x = a, y = 1, sau cho x = a, y = 1 ta c: f (a2 1) = (a 1) [f (a) + f (1)] f (a2 1) = (a + 1) [f (a) f (1)]

Cho cc v phi ca cc phng trnh bng nhau v gii phng trnh i vi f (a) ta c f (a) = f (1).a vi mi a.

giao im ca hai ng phn gic ngoi cc gc ABD v ADB, DAB v DBA, ACD v ADC, DAC v DCA tng ng. Chng minh rng bn im P, Q, R, S cng nm trn mt ng trn.

.c

4.13.Cho t gic li ABCD l t gic ni tip. Gi P, Q, R, S ln lt l cc

D C

o
P

f (x) = kx vi hng s k no r rng u tho mn yu cu bi ton.

m
B

Nh vy, mi hm s no tho mn rng buc cho phi c dng f (x) = kx vi hng s k no . Ngc li, bt k hm s no c dng

20

Nguyn Hu in, HKHTN H Ni Li gii: Cc gc xt n u l cc gc nh hng ngoi tr cc trng hp ni khc i. Gi s chng ta c mt tam gic tu XY Z vi tm ng trn ni tip l im I v tm ng trn bng tip IX i din vi gc X. Suy ra X, I, IX thng hng. Ta c IY IX = = IZIX v vy t gic IY IX Z l 2 ni tip c v XIX Y =IIX Y =IZY hay Y IX X=Y ZI. X

AQD=ABI1 , ARD=I2 CD, v ASD=ACI2 . Khi coi cc gc sau l khng nh hng, ta thy I1 BD, ABI1 , I2 CD v ACI2 u bng AQD = ACD . 2 2 Hn na, cc gc trn u cng mt hng, nn nu coi chng l nhng gc nh hng, chng s bng nhau. Nh vy (tr li vi nhng gc nh hng) ta c: AP D = AQD = ARD = ASD v bn im P, Q, R, S cng nm trn cung trn trng bi A, D. 4.14.Cho p l mt s nguyn t sao cho p 1 (mod4). Hy tnh
p1

p dng kt qu ca phn trn vi (X, Y, Z, IX ) l (A, D, B, P ), (D, A, B, Q), (A, D, C, R) v (D, A, C, S) ta c, AP D=I1 BD,

gic ABD i din vi gc A v gc D, tng t R, S l cc tm ng trn bng tip ca tam gic ACD i din vi gc A v gc D.

k=1

Gi I1 , I2 ln lt l tm ng trn ni tip cc tam gic ABD v tam gic ACD. T gi thit ta suy ra P , Q l cc tm ng trn bng tip ca tam

.c

2k 2 k2 2 p p

IX

thi olympic Korea

21

Li gii: Vi mi s thc x, t {x} = x [x] [0, 1). Ta c 2k 2 2k 2 = p p k2 k2 = p p k2 2k 2 2 =2 p p Nu {x} <


1 2 1 2

2k 2 p k2 p k2 p 2k 2 p

Ta c

ng d vi mt s no trong p1 , p 1 , v c tt c 2 nh th. T suy ra tng cho bng p1 . 2

thnh p1 cp dng {a, da} sao cho a2 (da)2 (modp). 2 V vy, c ng mt ng d trong mi cp m bnh phng ca n

V p l s nguyn t ng d vi 1 m un p, ta bit 1 d2 (modp), vi d l mt s no . Phn chia cc ng d m un p khc khng

k 2 l ng d m un p vi mt s no trong

Nu {x} th 2 {x} {2x} = 2{x} (2 {x} 1) = 1 Nh vy, tng cn tnh trong bi ra s bng l s cc phn t k trong 2 [1, p 1] sao cho kp 1 , hay bng vi s ng d k khc khng m 2

.c

th 2 {x} {2x} = {x} 2 {x} = 0

p+1 ,p 2

1 .

4.15.Xt nhng hnh L sau y, mi hnh c to bi bn hnh vung n v ghp li.

p1 2

ng d

22

Nguyn Hu in, HKHTN H Ni Cho m v n l cc s t nhin ln hn 1. Chng minh rng mt hnh ch nht kch thc mxn s c xp bi cc hnh cho khi v ch khi m.n l bi s ca 8. Li gii: Trc tin ta chng minh rng nu 8\mn, th hnh ch nht mxn c th c xp bi cc hnh cho. Trng hp 1: C m v n u l s chn. Khng mt tnh tng qut ta gi s rng 4\m, 2\n. Hai hnh cho c th ghp c mt hnh ch nht kch thc 4x2, v m.n/8 hnh ch nht nh vy s ghp thnh mt hnh ch nht kch thc mxn (gm n/2 hng v m/4 ct). Trng hp 2: Hoc m hoc n l. Khng mt tnh tng qut, ta gi s rng m l s l. Khi 8\n. V m > 1 nn m 3. Ta c th ghp c mt hnh kch thc 3x8 nh hnh v sau:

Nhng hnh 3x8 nh vy c th ghp thnh hnh ch nht (3xn).

mnh hnh ch L trong hnh ch nht c ghp s gm mt s l en hnh vung. V c tt c 1 s chn (nx m ) vung mu en, nn hnh 2 ch nht c ghp cha 1 s chn cc hnh ch L, m ta t s l 2k. Nh vy m.n = 8k, hay 8\mn. 4.16.Cho nhng s thc a, b, c, x, y, z tho mn a b c > 0 v x y z > 0. Chng minh rng: b2 y 2 c2 z 2 3 a2 x2 + + (by + cz)(bz + cy) (cz + ax)(cx + az) (ax + by)(ay + bx) 4

l 4 nn 4\(m.n). Khng mt tnh tng qut, gi s 2\n, v t m hng trong hnh ch nht mxn thnh hai mu en trng cnh nhau. Mi

By gi ta s chng minh rng: nu hnh ch nht c kch thc (mxn) c ghp bi cc hnh trn th 8\m.n. V mi mt hnh L c din tch

Nu m = 3, ta ghp xong. Trong trg hp ngc li, m > 3, th phn cn li (m3)xn c th ghp nh trong trng hp 1 v 2\(m3).

.c

thi olympic Korea

23

Li gii: t v tri ca bt ng thc l S. V a b c > 0 v x y z > 0 nn ta c bz + cy by + cz suy ra (by + cz)(bz + cy) (by + cz)2 2 (by)2 + (cz)2 t = (ax)2 ; = (by)2 ; = (cz)2 , khi ta c: a2 x2 a2 x2 = 2 + (cz)2 ] (by + cz)(bz + cy) 2 [(by) + p dng tng t cho hai bt ng thc, ta c 1 S ( + + ) 2 + + + S dng bt ng thc Cauchy-Schwarz, ta c

Vy bi ton c chng minh.

1 3 ( + + )2 1 + + ) S ( 2 + + + 2 (2 + 2 + 2) 4

Do ,

1 3 ( )2 + ( )2 + ( )2 + 3( + + ) (2+2+2) 2 2

m v phi bng

.c

+ + )(( + ) + ( + ) + ( + ))( + + )2 + + +

Chng 5 thi olympic Mng c


5.17.t rad (1) = 1, vi k > 1, t rad (k) l tch cc s nguyn t ca k. Mt dy cc s t nhin a1 , a2 , ... vi s hng u a1 c xc nh bi mi quan h: an+1 = an + rad (an ). Hy ch ra vi mi nguyn dng N, dy an+1 = an + rad (an ) gm N s hng lin tip trong mt cp s cng.

Chng minh: V rad (an ) l c ca c an v rad (an ) nn rad (an ) l c ca an + rad (an ) = an+1 mi tha s nguyn t ca rad (an ) l c ca an+1 V rad (an ) v rad (an+1 ) l tch ca cc c s nguyn t T cho ta kt qu rad (an ) l c ca rad (an+1 ) an *) Vi mi s nguyn dng n t bn = rad(an ) v zn =
rad(an+1 ) rad(an )

V rad (an ) l c ca rad (an+1 ) nn bn l mt s nguyn dng n Do b 1, ta cng c kt qu tng t vi zn Mt khc zn v rad (an ) l nguyn t cng nhau do rad (an+1 ) l tch ca cc c s nguyn t.

Li gii: *) B 1: Trong dy rad (a1 ) , rad (a2 ) , ... mi s hng l c ca s hng tip sau n.

.c

thi olympic Mng c V vy ta c: bn+1 an+1 = = rad (an+1 )


an +rad(an ) rad(an ) rad(an+1 ) rad(an )

25

bn+1 + 1 zn

*) B 2: Vi mi N, tn ti mt s nguyn dng M tha mn: zM = zM +1 = ... = zM +N 2 = 1 Chng minh: C vi s nguyn t p nh hn 2N tha mn iu kin tn ti mt s n sao cho p l c ca an p dng b 1, tn ti mt s nguyn m ln sao cho am chia ht cho mi s nguyn t. Gi M l s ln hn m sao cho bM l nh nht. Ta cn chng minh M tha mn iu kin ca b ny. Tht vy, gi s kt qu trn l khng ng, khi ta cn ch ra s k dng nh nht tha mn zM +k1 = 1

bM +k =

bM + k bM + k bM + N 1 bM +k1 + 1 = < bM zM +k1 zM +k1 2N 2N

iu ny mu thun vi gi thit M l s t nhin ln hn m v bM l s t nhin nh nht. Vy b 2 c chng minh. *) p dng b 2, vi mi N, tn ti mt s t nhin M tha mn: rad (aM ) = rad (aM +1 ) = ... = rad (aM +N 1 ) Vy aM , aM +1 , ..., aM +N 1 l cc s hng lin tip trong mt cp s cng.

Do chia ht cho mi an . T suy ra zM +k1 2N. V vy:

ht cho aM +k nhng khng chia ht cho aM +k1 v do aM +k1 chia ht cho rad (aM ) v rad (aM ) chia ht cho mi s nguyn t nh hn 2N.

Ta cn ch ra khng c s nguyn t no nh hn 2N c th chia ht cho zM +k1 . iu ny l ng v zM +k1 l tch cc s nguyn t chia

Mt khc k N 1 v zM = zM +1 = ... = zM +N 2 = 1 do bM +k1 = bM + k 1

.c

26

Nguyn Hu in, HKHTN H Ni

5.18.Trong mt phng, cho ba ng trn 1 , 2 , 3 i mt tip xc ngoi nhau. Gi P1 l tip im ca 1 , 3 , P2 l tip im ca 2 , 3 . A, B l hai im trn ng trn 3 khc P1 , P2 sao cho AB l ng knh ca ng trn 3 . ng thng AP1 ct li ng trn 1 ti X, ng thng BP2 ct li ng trn 2 ti Y . Cc ng thng AP2 , BP1 ct nhau ti Z. Chng minh rng X, Y, Z thng hng. Li gii: Xt cc gc l c hng theo modulo Gi P3 l tip im ca hai ng trn 1 , 2 v O1 , O2 , O3 l tm ca ba ng trn 1 , 2 , 3 tng ng. Gi 4 l ng trn ngoi tip tam gic P1 P2 P3 v O4 l tm ng phng ca ba ng trn 1 , 2 , 3 th ta c: O4 P1 = O4 P2 = O4 P3 . Do O4 l tm ca ng trn 4 . V O4 O1 O1 O3 nn O1 O3 l tip tuyn ca 4 .

O 3 P2 A = O 3 P2 Z = P2 P1 Z V Z v Z cng thuc AP2 , AP2 = BZ nn suy ra Z Z . Do Z 4 . ZP1 O3 = ZP1 O4 + O4 P1 O3 = XP1 Z + ZP1 O4 = XP1 O4 (1) V P1 O4 l tip tuyn ca 1 nn ta c: XP1 O4 = XP3 P1 (2) T (1) v (2) suy ra ZP1 O3 = XP3 P1 Gi l l ng thng ZP3 nu Z P3 hoc ng thng tip xc vi 4 ti P . Khi : (l, P3 P1 ) = ZP1 O3 (v O3 P1 l tip tuyn ca 4 . Kt hp iu ny vi kt qu trn, ta suy ra: (l, P3 P1 ) = XP3 P1 Xl Chng minh tng t ta c Y l . V Z l nn ta suy ra ba im X, Y, Z thng hng.

V O4 P1 O3 v XP1 Z cng vung (do AB l ng knh) nn ta c:

Do P2 P1 Z = P2 P1 Z v Z thuc ng thng BZ

.c

Chng minh tng t ta cng c O1 O2 , O2 O3 l tip tuyn ca 4 . V O3 AB nn ta c: P2 P1 Z = P2 AO3 = O3 P2 A.

Nu gi Z l giao im th hai ca AP2 v 4 th do O3 P2 l tip tuyn ca 4 nn ta c:

thi olympic Mng c 5.19.Mt hm s f : R R tha mn cc iu kin sau: 1) |f (a) f (b)| |a b| a, b R 2) f (f (f (0))) = 0 Chng minh rng f (0) = 0.

27

Li gii: Ta s dng nhn xt sau: f k (x) = f (f (...f (x) ...))


k ln f

T (1) ta c: |f (0)| = |f (0) 0| f 2 (0) f (0) f 3 (0) f 2 (0) = f 2 (0)

Li gii: Gi s ng trn ngoi tip t gic BA1 B1 C1 ct li ng thng AC ti X. Ta cn chng minh X phi thuc on AC. Trc ht, do A nm trn ng BC1 nhng khng thuc on BC1 nn A phi nm ngoi ng trn . Tng t, C nm ngoi ng trn . Ta c mi im nm trong on B1 X u nm trong ng trn , do B1 X khng cha A cng khng cha C. V B1 nm trn cnh AC do X nm trn cnh AC. t BC = a; CA = b; AB = c. p dng phng tch ca im A i vi ng trn ta c: AB.AC1 = AX.AB1

BC AC AB = = AC + AB BA + BC CA + CB

ca tam gic ti cc im A1 , B1 , C1 . Chng minh rng:

5.20.ng phn gic ca cc gc A, B, C ca tam gic ABC ct cc cnh

Vy f (0) = 0.

.c

*) Trng hp 2: f (0) = f 2 (0) Ta c: |f (0)| = |f (0) 0| |f 2 (0) f (0)| = 2 |f (0)| |f (0)| 0 f (0) = 0

*) Trng hp 1: f (0) = f 2 (0) Khi : f (0) = f 2 (0) = f 3 (0) = 0

v |f 2 (0)| = |f 2 (0) 0| |f 3 (0) f (0)| = |f (0)| ta suy ra |f (0)| = |f 2 (0)|

28

Nguyn Hu in, HKHTN H Ni T nh l v ng phn gic trong tam gic ta c: AC1 = Do ta c: AX = AC1 .AB bc a+c (a + c) c = .c. = AB1 a+b bc a+b
(a+c)a b+c

bc bc ; AB1 = a+b a+c

Chng minh tng t ta cng c: CX = Hn na, do X thuc cnh AC nn:

b = AC = AX + CX = (a + c) hay

c a + a+b b+c

(9 + 9 + 9)2 (4 + 4 + 8)2 (3 + 3 + 3)2 1= ; 2= ; 3= 9.9.9 4.4.8 3.3.3 4= (2 + 2 + 4)2 (1 + 4 + 5)2 (1 + 2 + 3)2 ; 5= ; 6= 2.2.4 1.4.5 1.2.3 (1 + 1 + 2)2 (1 + 1 + 1)2 8= ; 9= 1.1.2 1.1.1 Ta s chng minh khng c cc kt qu khc tha mn b sau: B : Nu n c th biu din c di dng (x + y + z)2 xyz

Li gii: *) Nhn xt: Ta c cc s sau tha mn yu cu bi ton:

vi x, y, z l cc s nguyn dng.

5.21.Nhng s nguyn no c th biu din c di dng (x + y + z)2 xyz

.c

BC AC AB = = AC + AB BA + BC CA + CB

c a a b c b = + = a+c a+b b+c b+c a+c a+b

thi olympic Mng c th n c th vit di dng: (x + y + z )2 xyz vi x y +z ; y Chng minh b : x +z ; z x +y


(x+y+z)2 xyz

29

Gi x, y, z l cc s nguyn dng tha mn n = l nh nht.

v x + y + z

V n l mt s nguyn nn x l c ca (x + y + z)2 do x l c ca (y + z)2 t x =


(y+z)2 , x

khi ta c
2

(y + z)2 y+z + 1 (x + y + z)2 x = (y+z)2 x yz yz


x

y+z x

+1 yz

(x + y + z)2 = =n xyz
2

(2x) 4x Khi n = x(x1) = x1 x 1 l c ca 4x x 1 l c ca 4 x {2; 3; 5} n {8; 6; 5} *) Trng hp 3: y + z x y z > 1

Do x y Do n=

Khi : yz (y + z) = (y 1) (z 1) 1 yz y+z x z ta c: xy z ; xz y

Khi n = (2x+1) x l c ca 2x + 1 x = 1 v n = 9 x2 *) Trng hp 2: x = y + 1 > z = 1

Ta xt cc trng hp sau: *) Trng hp 1: x = y z = 1

t
0

(x + y + z)2 =2 xyz

x y z 1 1 1 + + + + + x y z yz xz xy

Gi s n = (x+y+z) . p dng b , khng mt tnh tng qut, ta c xyz th gi s y + z x y z

.c

Do x+y+z l nh nht nn x+y+z x +y+z Suy ra x x = (y+z) x x y + z Chng minh tng t th ta cng c: y x + z ; z x+y (B c chng minh)

3 2. +1+1+1 = 6 2

Vy cc kt qu ca n cn tm l n {1; 2; 3; 4; 5; 6; 8; 9}

n {1; 2; 3; 4; 5; 6}

30

Nguyn Hu in, HKHTN H Ni

5.22.Mt t nc c n thnh ph. Tng chi ph ca chuyn i t thnh ph i n thnh ph j l xij . Gi s rng tng chi ph ca tuyn ng qua mi thnh ph ng mt ln v kt thc ti im bt u khng ph thuc vo vic chn tuyn ng. Chng minh rng tn ti cc s a1 , a2 , ..., an v b1 , b2 , ..., bn sao cho xij = ai + bj vi mi s nguyn dng i, j tha mn 1 i < j n. Li gii: t f (a, b) = xa1 + x1b xab vi a, b v 1 l ba s phn bit *) B : f (a, b) khng ph thuc vo a, b Chng minh b : +) Vi n c a v b 2, iu ny l tm thng v khi f c xc nh khng

a, b, 1, c, 2, 3, ..., a 1, a + 1, ..., b 1, b + 1, ..., c 1, c + 1, ..., n phi c tng ton b chi ph bng nhau. Cc tuyn ng ny gn ng nht, cho php ta dng tm c s khc nhau ca tng chi ph ca 2 tuyn ng l:

Do , f (a, b) = f (b, c) vi mi a, b, c i mt khc nhau v khc 1 Hn na, tng ca ton b chi ph ca 3 tuyn ng: 1, a, b, 2, ..., n ; b, 1, a, 2, ..., n ; a, 1, b, 2, ..., n phi bng tng ca ton b chi ph ca 3 tuyn ng: 1, b, a, 2, ..., n ; a, 1, b, 2, ..., n ; b, 1, a, 2, ..., n

(xa1 + x1b + xbc ) (xab + xb1 + x1c )

v tuyn ng

a, 1, b, c, 2, 3, ..., a 1, a + 1, ..., b 1, b + 1, ..., c 1, c + 1, ..., n

.c

Nhng nhng ng thc ny l tng cc chi ph ca 2 tuyn ng m mi tuyn u i qua mi thnh ph ng 1 ln, do chng bng nhau. +) Vi n 4, tuyn ng:

+) Vi n = 3 ta cn ch ra f (2, 3) = f (3, 2) hay x21 + x13 + x32 = x31 + x12 + x23

thi olympic Mng c Do 2 (x1a + xab + xb1 ) = 2 (x1b + xba + xa1 ) T suy ra f (a, b) = f (b, a) Vi c, d khng bng a, b ta c: f (a, b) = f (b, c) = f (c, d) ; f (a, b) = f (b, c) = f (c, b) ; f (a, b) = f (b, a) = f (a, c) = f (c, a) iu ny chng minh c kt qu ca b .

31

*) Vi mi a, b phn bit v khc 1 ta c: f (a, b) = F vi F l hng s Cho a1 = 0 ; b1 = F v t bk = x1k ; ak = xk1 F Vi mi i, j khng ng thi bng 1 ta c: xij = xi1 xi1 x1j + xij + x1j = xi1 F + xij = ai + bj tc l tn ti cc s a1 , a2 , ..., an v b1 , b2 , ..., bn sao cho xij = ai + bj vi mi s nguyn dng i, j tha mn 1 i < j n

.c

Chng 6 thi olympic Rumani


m
6.23.Hm f : R2 R c gi l olympic nu n tha mn tnh cht: vi

knh l

P (x) = a(z z0 )n vi a, z0 C v n N. Nu A1 , A2 , , An l cc im ri rc trong R2 sao cho f (A1 ) = f (A2 ) = = f (An ) th A1 , A2 , , An nm trn ng trn vi tm l (Re(z0 ), Im(z0 )) v bn
n

Ngc li, ta gi s rng khng phi tt c cc nghim ca P l bng nhau, khi P (x) c dng: P (x) = (z z1 )(z z2 )Q(z) vi z1 v z2 l 2 nghim phn bit ca P (x) sao cho |z1 z2 | l nh nht. Gi l l ng thng i qua hai

, |f (A1 )|

c r 0, bi v nu ngc li ta c z3 l mt nghim ca P sao cho:

im Z1 v Z2 vi Z1 = (Re(z1 ), Im(z2 )),Z2 = (Re(z2 ), Im(z2 )), v t z3 = 1 (z1 + z2 ) sao cho Z3 = (Re(z3 ), Im(z3 )) l trung im ca Z1 Z2 . 2 K hiu s1 , s2 ln lt l cc tia Z3 Z1 , Z3 Z2 , v r = f (Z3 ) 0. Ta phi

suy ra cc im l cc nh ca mt a gic li.

Li gii: Trc ht ta gi s rng tt c cc nghim ca P l bng nhau, khi ta vit c di dng:

Cho P C[X] khc a thc hng. Chng mnh rng hm f : R2 R c cho bi f (x, y) = |P (x + iy)|, l olympic khi v ch khi tt c cc nghim ca P l bng nhau.

.c

n 3 cc im ri rc A1 , A2 , , An Rn , nu f (A1 ) = f (A2 ) = = f (An ) th cc im A1 , A2 , , An c gi l cc nh ca a gic li.

thi olympic Rumani |z1 z3 | |z1 z2 |, iu ny l mu thu vi |z1 z2 | l nh nht. Do


ZZ3 Zs1

33

lim f (Z) = lim f (Z) = +.


ZZ3 Zs1

v f lin tc, tn ti Z4 s1 v Z5 s2 sao cho f (Z4 ) = f (Z5 ) = r. Do vy f (Z3 ) = f (Z4 ) = f (Z5 ) v Z3 , Z4 , Z5 khng phi l cc nh ca a gic li. Do vy, f khng phi l olympic. 6.24.Vi n 2 l s nguyn dng. Tm s cc hm f : {1, 2, , n} {1, 2, 3, 4, 5} tha mn tnh cht: |f (k + 1) f (k)| 3 vi k = 1, 2, , n 1 Li gii: Ta c n 2 bt k v tm s cc hm tng ng. Nu f : {1, 2, , n} {1, 2, 3, 4, 5} phi tha mn cho th f (n) = 3 bi nu ngc li th f (n 1) 0 hoc f (n 1) 6, v l. K

an+1 = en + dn , bn+1 = en

an+2 = en+1 + dn+1 = an+1 + bn+1 = an+1 + en = an+1 + an do vy, {an }n2 tha mn nh dy Fibonaci {Fn }n0 , vi cc ch s c chn sao cho: F1 = 0 v F1 = 1. Bi v a2 = 2 = F2 v a3 = e2 + d2 = 3 = F3 , vy suy ra an = Fn vi n. Do ,an +bn +dn +en = 2(an +bn ) =

6.25.Cho n 1 l mt s nguyn dng v x1 , x2 , , xn l cc s thc sao cho: |xk+1 xk | leq1 vi k = 1, 2, , n 1. Chng mnh rng:
n n

2en+1 = 2an+1 = 2Fn+1 vi n 2 v 2Fn+1 tha mn tnh cht cho.

bng quy np ta c an = en v bn = dn n 2. Do vy vi n, ta c:

Ta cn tm an + bn + dn + en vi n 2. Ta c: a2 = e2 v b2 = d2 ;

en+1 = an + bn , dn+1 = an

k=1

|xk |

k=1

xk

.c

a2 = e2 = 2 v b1 = d2 = 1, v do vy vi n 2 :

hiu an , bn , dn , en l s cc hm f : {1, 2, , n} {1, 2, 3, 4, 5} tha mn tnh cht cho sao cho f (n) tng ng bng 1, 2, 4, 5. Khi

n2 1 4

34

Nguyn Hu in, HKHTN H Ni

Li gii: Nu s cc s xk m ln hn s cc s xk dng th (a1 , , an ) l mt hon v ca (x1 , , xn )(tng ng l (x1 , , xn )) sao cho a1 , , an l mt dy khng gim. Do cch xy dng, lc lng P cc

xk xj 1. Do vy, ak0 +1 ak0 + 1 1,ak0 +2 ak0 +1 + 1 2. K hiu P v N ln lt l tng ca cc s trong P v N. Mt khc ta c:

s dng ak khng nhiu phn t hn lc lng N cc s mak , v do n1 . V N l khc rng v a1 , , an l khng gim, cc vy |P | 2 phn t ca P l ak0 +1 < ak0 +2 < < ak0 +l vi k0 > 0 Gi s rng 1 i n 1. Trong dy x1 , , xn phi c hai phn t k nhau xj v xk sao cho xj ai v xk ai+1 suy ra 0 ai+1 ai

|P N | |P N | |2P |

a 3

a(a 1)(a 2) 6

3 Li gii: Ta thy rng: n + Cm > 2m + 1 vi m ln hn gi tr N, bi v v tri l bc 3 vi h s cao nht dng trong khi v phi l tuyn

tnh vi m.
3 Nu m 0(mod4), th Cm = m(m1)(m2) l chn bi v t s chia ht 6 3 cho 4 cn mu s th khng.Nu m 3(mod4) th Cm = m(m1)(m2) l 6 l bi v c t s v mu s u chia ht cho 2 nhng khng chia ht 3 cho 4. Do vy ta chn m > maxk, N sao cho n + Cm l s l.

3 3 3 3 3 n = Ca1 Ca2 Ca3 Ca4 Ca5

nguyn dng a1 > a2 > a3 > a4 > a5 > ksao cho:

6.26.Cho n, k l cc s nguyn dng ty .Chng minh rng tn ti cc s

n1 n1 n2 1 .( + 1) = 2 2 4

.c

2(1 + 2 + +

n1 ) 2

thi olympic Rumani

35

3 3 3 3 Ta vit: 2a+1 = n+Cm > 2m+1. Ta thy rng:(Ca+3 Ca+2 )(Ca+1 3 2 2 Ca ) = Ca+2 Ca = 2a + 1. Do vy

n = (2a + 1)

m 3

a a+1 a+2 a+3 + 3 3 3 3

m 3

tha mn yu cu bi ton v a + 3 > a + 2 > a + 1 > a > m > k 6.27.Cho P1 P2 Pn l mt a gic li trong mt phng. Gi s rng vi cp nh Pi , Pj , tn ti nh V ca a gic sao cho Pi V Pj = minh rng n = 3
. 3

Chng

l li, nn mi hoc nm trong 1 phn hoc trng vi A, B, C. Nu 2 im bt k trong A1 , B1 , C1 , gi s l A1 , B1 nm trong min DX , th A1 XB1 < . Do vy, max{A1 X, XB1 } > A1 B1 , mu thun 3 vi A1 B1 l ln nht. Hn na, khng c hai im trong A1 , B1 , C1 trong cng 1 phn. By gi ta gi s rng mt trong cc im A1 , B1 , C1 ( gi s l A1 ) nm trn 1 phn(gi s l DA ). Bi v min{A1 B, A1 C} BC, ta c

BA1 C . Ta c B1 khng nm trong DA . Bi v a gic cho l 3

gic u. Ta ch ra rng

ABC A1 B1 C1 . Cc ng thng AB,BC, CA chia = mt phng thnh 7 phn. Gi DA gm cc phn do tam gic chia m nhn BC lm bin v cc phn c to ra ti phn to ra cc nh B v C. Tng t ta nh ngha cho DB v DC . Bi v a gic cho

l tam gic u. Tung t, ta chn A1 , B1 sao cho A1 B1 c di ln nht, v nh C1 sao cho A1 C1 B1 = , khi tam gic A1 B1 C1 l tam 3

l 2 nh phn bit sao cho AB c di nh nht, v C l nh sao cho ACB = . Khi max{AC, CB} AC, tam gic ABC phi 3

.c

Chng ta ch ra rng tn ti cc nh A, B, C v A1 , B1 , C1 sao cho:(i) tam gic ABC v A1 B1 C1 l tam gic u v (ii) AB(tng ng l A1 B1 ) l khong cch nh nht (ln nht) khc 0 gia 2 nh. Hn na, A, B

max{Y X, Y Z} > XZ. Tng t nu XY Z u hoc min{Y X, Y Z} < XZ

Li gii: Trong li gii ny ta s dng kt qu sau: Cho tam gic XYZ sao cho XY Z th tam gic l u hoc 3
3

th tam gic XYZ

36

Nguyn Hu in, HKHTN H Ni li, B khng nm trong tam gic AA1 B1 , v tng t C khng nm trong tam gic AA1 B1 . T c B1 nm trn min ng c bin l cc tia A1 B v A1 C. Tng t, vi C1 . Hn na, = B1 A1 C1 BA1 C = , du 3 3 bng xy ra khi B1 v C1 ln lt nm trn tia A1 B v A1 C . Bi v a gic cho l li,nn iu ny ch xy ra khi B1 v C1 ln lt bng B v C -trong trng hp BC = B1 C1 , ta c tam gic ABC v A1 B1 C1 l bng nhau. Mt khc, khng c im no trong A1 , B1 , C1 nm trn DA DB DC , do chng ln lt trng vi A, B, C. Trong trng hp ny, tam gic ABC v A1 B1 C1 l trng nhau. Do vy hai nh bt k ca a gic c khong cch ging nhau, nh AB = A1 B1 . iu ny l khng th xy ra nu c hn 3 im trong mt phng th hon ton khng c tnh cht ny. Do vy n=3

6.28.Chng minh rng tn ti v hn b gm 4 s nguyn dng(x, y, z, t)

x3 + y 3 + z 2 = t4

Li gii: t a = k 3 vi k l s chn v k > 0 ta c:

Bi v k 3 + 1 l s l,nn (2k 3 , k 3 + 1) = (k 3 , k 3 + 1) = 1. Do vy, ta c v hn b bn s dng (x, y, z, t) = (2k 3 , 2k, (k 3 1)2 , k 3 + 1) vi k > 0 l s chn, tha mn cc iu kin ca bi ton. 6.29.Biu din nh phn ca mt s nguyn dng l a, c xc nh bng thut ton n gin sau: xc nh mt s nguyn dng nh nht n sao cho 22000 l c ca an 1

Li gii: Bi v a l s l nn (a, 2k ) = 1 vi k 0. Do vy, theo nh k k1 a(2 ) 1(mod2k ) vi k. Do vy bc n ca a l Euler, ta c a2 modulo 22000 chia cho 22000 1 = 21999

(2k 3 )3 + (2k 3 )3 + [(k 3 1)2 ]2 = (k 3 + 1)4

tc l

(a + 1)4 (a 1)4 = 8a3 + 8a

.c

sao cho c chung ln nht ca 4 s l 1.v tha mn:

thi olympic Rumani

37

Nu a 1(mod22000 ) suy ra n=1. Ta gi s rng a = 1(mod22000 ) . Vi mi m 1 , ta vit: a2 1 = (a 1)(a + 1) (a2 + 1)(a2 + 1) (a2

m 2 m1

+ 1)

().

(*)vi m c nh (m 1), 21 l s m cao nht ca 2 m chia ht cho m 1 biu thc mi ngoc n pha trn . Nu a 1(mod4), khi a = 1, biu din nh phn ca a kt thc l: 1 00 01
s ch s

Biu din nh phn ca a c kt thc l 2 ch s 01 hoc 11. Ta c k a 1(mod4) v do vy a2 1(mod4) vi k 1. Do vy phn tch

Trong cc trng hp tng ng ta c n = 21999s hoc n = 2. Bi v ta c th d dng s dng biu din nh phn ca a suy ra hai trng hp v gi tr ca s l g, ta c th s dng biu din nh phn ca a tm n. 6.30.Cho tam gic nhn ABC v im M l trung im ca BC. Tn ti duy nht mt im trong N sao cho ABN = BAM v ACN = CAM. Chng minh rng BAN = CAM.

chia cho (a2 1 c 2s+m . T c m 1 nh nht sao cho a2 1 chia ht cho 22000 l 2000 s (nu s<2000) hoc 1 ( nu s 2000).
m m

s m cao nht ca 2 chia cho (a + 1) c 2s trong khi s m cao nht ca 2 chia cho (a 1) c 2. Trong mi trng hp, ta s dng (*) v kt qu s m cao nht ca 2

vi s l s ngueyen ln nht sao cho 2s |(a + 1). Trong trng hp ny,

1 011 1
ch s

Nu thay th l a 1(mod4) , khi bi v a = 1 , biu din nh phn ca a kt thc l:

.c

m cao nht ca 2 chia cho a 1 l 2s trong khi s m cao nht ca 2 chia ht cho a + 1 l 2

vi s l s nguyn ln nht sao cho 2s |(a 1). Trong trng hp ny, s

38

Nguyn Hu in, HKHTN H Ni Li gii: Cho B l im nm trn tia AC sao cho ABB = BAM, cho C l im nm trn tia AB sao cho ACC = CAM. Khi N l giao im ca hai ng thng BB v CC ng thng i xng vi AM qua ng phn gic ca gc BAC, ct BB ti P. Gi D l im i xng ca A qua M, t gic ABCD l hnh bnh hnh. Bi v P AB = CAM = CAD v ABP = MAB = DAB = ADC, tam gic ABP v ADC l ng dng. Do AP AB = . Bi v BAD = P AC, nn tam gic BAD v tam vy, AD AC gic PAC ng dng. Hn na, ACP = ADB = CAM. T suy ra P nm trn ng thng CC cng nh nm trn BB , v do vy N P . Hn na c BAN = BAP = CAM nh yu cu bi ton.

.c

Chng 7 thi olympic Nc Nga


m
7.31.Sasha th xc nh vi s nguyn dng x 100. Anh ta chn hai s

Sasha c th xc nh c gi tr ca x sau 7 cu hi. Li gii: +) Vi n = 0, 1, 2, ..., 6, t an l s nguyn duy nht trong [0; 2n ) tha mn 2n |(x an ). R rng a0 = 0. +) Vi n 5, an+1 bng an hoc an + 2n m khi kt qu c vn c nu v ch nu gcd (x + 2n an , 2n+1 ) = 2n V 2n an < 2n+1 < 100 ta suy ra nu Sasha bit c gi tr ca an ,

anh ta c th xc nh c an+1 vi mt cu hi iu kin bng cch t (M, N) = (2n an , 2n+1 ). Do sau 6 cu hi, Sasha c th xc

nh c a1 , a2 , ..., a6 v kt lun x bng a6 hoc a6 + 64 Bi v a6 = a6 + 64 (mod 3), Sasha c th xc nh c x nu anh ta pht hin ra liu c hay khng x a6 (mod 3) vi cc cu hi ca anh ta. Tht vy, anh ta c th nu t N = 3 v M {1, 2, 3} nn 3 |(a6 + M ) ,

7.32.Cho O l tm ng trn ngoi tip tam gic nhn ABC. ng trn 1 vi tm K i qua cc im A, O, C m ct cc cnh bn AB v BC ti M v N. t L l im i xng vi K qua ng thng MN.

anh ta s thu c cu tr li "3" nu v ch nu x a6 (mod 3).

.c

nguyn dng bt k M v N m nh hn 100 v c cu hi "S no ln nht trong cc c s chung ca x + M v N?" Chng minh rng

40 Chng minh rng BLAC.

Nguyn Hu in, HKHTN H Ni

Li gii: Gi , , l cc gc A, B, C ca tam gic ABC. V t gic ACNM ni tip, BNM = , BMN = nn MKC = 2 ; NKA = 2 V ng thng AC l trc ng phng ca ng trn v 1 nn n vung gc vi OK. Khi ta c: AOK = COK = OAK = OCK = Do AKC = 2 4 Kt hp cc iu trn, ta c: MKN = 2 + 2 (2 4) = 2

Li gii: Ta s dng s dch chuyn trong l thuyt th. Trong mt th, tt c mi nh u c bc t nht l 3. Ta chng minh tn ti mt chu trnh m di ca m khng chia ht cho 3. Thc hin thut ton sau: +) C nh mt im u v1 sau cho v1 , v2 , ..., vi nu tn ti mt im ring t i nh v ti nh gn k th vi+1 l mt nh. Bi v th c gii hn v tt c cc nh t c bng thut ton ny l r rng. Qu trnh ny kt thc ti vi nh vn . Ta bit mi nh c t nht bc

con ng tun hon (c ngha l ni kt thc l ni bt u) nh th s con ng trong nhng con ng khng th chia ht cho 3.

m mi con ng ni hai thnh ph v khng c hai con ng no ni hai thnh ph c tn ging nhau. N c hiu rng trong mi thnh ph u c ti thiu 3 con ng i ra. Chng minh rng tn ti mt

7.33.C vi thnh ph trong mt quc gia v mt cch t tn ng. Ni

ng trn ngoi tip tam gic MBN nn ta suy ra MLB = V khi : MBL = . Bi v BAC = , ta d dng suy ra BLAC. 2

.c

Mt khc ta cng c: MBN CBA bi v t gic ACNM ni tip. Do v O l tm ng trn ngoi tip tam gic ABC v L l tm

V L l im i xng ca K qua ng thng MN nn ta c: MLN = 2 v LM = LN. Do LMN OCA

thi olympic Nc Nga

41

Ta c 3 chu trnh:

3 v bi s tha nhn mi nh gn k vi vn lp thnh mt dy. Nh vy, vn l nh k vi va , vb , vn1 vi a < b < n 1.

va va+1 va+2 ... vn1 vn va

vb vb+1 vb+2 ... vn1 vn vb

va va+1 va+2 ... vb1 vb vn va Nhng chu trnh trn c di n a + 1 ; n b + 1 ; b a + 2 theo th t. Bi v (n a + 1) (n b + 1) (b a + 2) = 2 khng th

7.34.Cho x1 , x2 , ..., xn l cc s thc (n x2 < minh

0, ..., f (xn ) > 0. iu ny v l v f (x1 ) + f (x2 ) + ... + f (xn ) = 0 c cho bi phng trnh trn. Chng minh tng t ta cng suy ra xn > 0 Gi s rng 2 i n +) Nu xi 0 th khi ta c 0 v
n j=1 n i1

x1 < x2 < ... < xi1 < xi cng chng minh c

Li gii: Vi 1 < x < 1, t f (x) = x x13 Ta phi c x1 < 0 v nu khng f (x1 ) 0 v f (x2 ) > 0, f (x3 ) >

s y1 < y2 < ... < yn .

2) tha mn iu kin 1 < x1 < 13 13 ... < xn < 1 v x1 + x2 + ... + x13 = x1 + x2 + ... + xn . Chng n 13 13 13 rng: x1 y1 + x2 y2 + ... + xn yn < x1 y1 + x2 y2 + ... + xn yn vi mi

f (xj ) =

.c

chia ht cho 3 Do tn ti 1 trong 3 chu trnh trn c di khng chia ht cho 3.

f (xj ) > 0
j=1

+) Nu thay th xi > 0 th khi ta c: 0 < xi < xi+1 < ... < xn v ta f (xj ) > 0
j=1

+) S dng cng thc ly tng Abel v kt qu trn ta c:

42
n n

Nguyn Hu in, HKHTN H Ni


n

i=1 n n

xi yi

x13 yi i
i=1

=
i=1

yi f (xi )

= y1
n i=1

f (xi ) +
i=2

(yi yi1 ) (f (xi ) + f (xi+1 ) + ... + f (xn ))

=
i=2

(yi yi1 ) (f (xi ) + f (xi+1 ) + ... + f (xn )) > 0

T ta d dng suy ra iu phi chng minh. 7.35.Gi AA1 , CC1 l cc ng cao ca tam gic nhn ABC. ng phn gic ca gc nhn gia hai ng thng AA1 , CC1 ct cc cnh AB v BC ti P, Q tng ng. Gi H l trc tm tam gic ABC v M l trung im ca cnh AC, ng phn gic ca ABC ct on HM ti R. Chng minh rng t gic P BQR ni tip c mt ng trn. Li gii: H ng vung gc vi cnh AB v BC ti P , Q tng ng, chng ct nhau ti R . Gi S l giao im ca R P v HA, T l giao

Do :

HP HC 2MU MU HV HT = = = = = HS HQ HA 2MV MV HU

Khi , php v t tm H bin ng thng P S thnh ng thng MU v cng bin ng thng QT thnh ng thng MV . Do n bin R = P S QT thnh M = MU MV . V th 3 im H, R , M

thng hng Ta li s dng gi thit P HA QHC, ta c HP B, HQB l ng d v chng cng ph vi hai gc HP A, HQC. Nh vy, BP = BQ v BR P BR Q nn P BR = QBR .

Mt khc, v HAP = P HA QHC

Do P Q l ng phn gic ca gc nhn to bi hai ng thng AA1 , CC1 nn P HS = QHT . Do P HS QHT

a
2

gc t M ti BC ct HC ti V . V P SH, HT Q c cc cnh tng ng song song v P SH = HT Q

ABC = QCH v P HA = QHC nn

.c

im ca R Q v HC. H ng vung gc t M ti AB, ct HA ti U v h ng vung

thi olympic Nc Nga

43

Do R nm trn c hai ng thng HM v ng phn gic ca ABC, suy ra R R T d dng suy ra t gic P BQR ni tip v BP R =
2

= BQR

7.36.C 5 vin ngc c trng lng khc nhau. Oleg bit c trng lng ca tng vin. Vi mi vin ngc x k hiu m (x) l trng lng ca n. Dmitrii c gng xc nh trng lng ln nht ca cc vin ngc . Anh ta c php chn 3 vin A, B, C v hi Oleg rng: "C phi m (A) < m (B) < m (C) khng?". Oleg ch tr li "ng" hoc "Sai". Hi Dimitrii c th xc nh c trng lng ln nht sau 9 cu hi hay khng? Li gii: Chng ta s ch ra rng Dimitrii s khng th xc nh c khi lng vin ngc ln nht sau 9 cu hi.

hi u tin, mt phn s1 ca nhng kh nng c loi b nu Oleg tr li "ng" ti cu hi th i + 1; trong khi phn b ca mt phn ca s2 s c loi b nu Oleg tr li "Sai". Nu |S1 | xi v Oleg tr li "ng" th ta c: 2 xi xi+1 = xi |S1 | 2 Mt khc ta c: |S2 | xi 2 Nu Oleg tr li "Sai" ta li c xi+1 xi 2 Do , nu x1 = 120; x2 80; x3 60; x4 x6 10; x7 5; x8 3; x9 2

"Sai" th Dimitrii c th loi b ti a 20 trong s 120 kh nng c th. Trong trng hp xi+1 xi 20 vi mi i. Vi mi xi m ph hp i cu

m (A) < m (B) < m (C). Nh vy, nu Dimitrii hi liu m (A) < m (B) < m (C) v Oleg tr li

vi i = 1, 2, ..., 8 rng c 5! = 120 cch ly c vin ngc c trng lng ln nht. Sau vi 3 vin ngc A, B, C bt k, ng 1 vin ngc c th c 6

.c

cu hi th i ca Dimitrii c chnh xc xi vin ngc c trng lng ln nht tha mn cu hi th i. Ta ch ra rng xi+1 max xi 20, 1 xi 2

Gi s Dimitrii c mt phng php xc nh c vin ngc c khi lng ln nht sau 9 cu hi hoc t hn. Gi s rng sau khi Oleg tr li

40; x5

20;

44

Nguyn Hu in, HKHTN H Ni T , Dimitrii khng th chc chn rng anh ta tm thy c kt qu sau 9 cu hi.

.c

Chng 8 thi olympic i Loan


8.37.Cho tam gic nhn ABC, AC > BC v M l trung im AB. Cc ng cao AP v BQ gp nhau H, ng thng AB v BQ ct nhau R. CMR: RH CM. ng vung gc t H xung CM. V HP C = HQC =HXC = nn H, P, Q, X v C cng nm trn mt 2 ng trn. Tng t, v HXM = HSM = thuc mt ng trn.

trong cc trc i xng ca 3 ng trn ny l AB, PQ, HX phi trng nhau. V R = AB PQ nu R phi thng hng vi H v X. Do : RH CM. 8.38.Gi (h) l s cc s nguyn dng n tho mn UCLN ( n, k) = 1 v n k . Gi s (5m 1) = 5n 1 vi m, n nguyn dng no . CMR: UCLN (m, n) > 1. Li gii: Trong li gii ny, chng ta s dng nhng l thuyt v hm sau:

Hn na, P, Q, S v M thuc cng mt ng trn v chng nm trn ng trn 9 im ca tam gic ABC. Theo tnh cht i xng, hai

t
2

Li gii: Gi S l chn ng cao h t C xung AB v X l chn

nn cc im H, X, S v M cng

.c

46 (ab) = (a) (b)(1) (p ) = p p1 (2)

Nguyn Hu in, HKHTN H Ni

(1) Vi a, b nguyn t cng nhau (2)Nu p nguyn t v l s nguyn dng Gi s phn chng rng UCLN (m, n) = 1 u tin ta ch ra m l s l Ta c: 5n 1(mod8) nu x l chn . Nu m l chn th 5m 1. nhng 5m 1 = 8 .8 . .(16) = 8 V 5m 1 = 8, 5n 1 = (5m 1). hoc (8).(P ) = 8(P ) 0(mod8)

Vi P > 1 l lu tha ca s nguyn t l P Do n phi chn, UCLN ( m,n) = 1 theo nh gi thit phn chng. Tip theo ta gi s rng

Do ,5m 1 = 4

p vi S l tp cc s nguyn t l.
pS
m

Hn na, ta khng th c P 1(mod5) v 5 chia ht (5m 1) = 5n1 v P 1 = (p) , iu ny khng xy ra. Do P 4 (mod 5) Suy ra 1 5m 1 = 4 (p) = 2
pS pS

5 Li c, theo lut tng h : ( p )( p ) = (1) (51)(p1) = 1 5 4 p dn n( 5 ) = 1m P 1 hoc 4 (mod5).

5 Cho P l phn t bt k thuc S. V 1 = ( p = ( 5p ) = ( m )n v m l l, ) (5) = 1 p

Li c :d/(p2 ) v (5m 1) = 5n 1 do d/n. Nhng d>1 v 5 = 1(modp) vUCLN(m,n)=1-a (theo gi thit phn chng)

pS

(4).

(p 1) = 2 3|s| (mod5)

Do gt ca ta l sai, vy UCLN (m, n) > 1.

T phng trnh u ny ta c |S| phi chn. Nhng t phng trnh th hai ta c:|S| 3(mod4)iu ny mu thun

8.39.Cho A ={1, 2,......, n} . Vi n N . Mt tp hp con ca A c gi l

kt ni nu n l s nguyn ln nht sao cho A cha k tp i mt

.c

p 4 4|s| (mod5) v1 = 5n 1 =

P 2/(5m 1) vi P nguyn t l R rng PX5, v 5 c............... modulop, gi d l ..................... nn ta c d/m

thi olympic i Loan

47

khc nhau: Sao cho giao ca hai tp bt k Ai v Aj l mt tp kt ni Li gii: Gi A1 , A2 , ...Ak l cc tp con khc nhau ca A trong gi thit t m = max (min Ai )
1in

v gi thit rng min Ai0 = m Mi tp Ai c phn t b nht nh hoc bng m, do cch xc nh m v mi tp Ai c phn t ln nht ln hn hoc bng m hocAi Ai0 = l tp khng kt ni. Do , mi cp k(min Ai , max Ai ) bng mt trong m(n+1m)cp

8.40.Cho hm F : N N tho mn f (1) = 0


0j
2

cha m0 m m0 =

n 2

n 2

iu ch ra rng c hai tp u bng {r, r + 1, ..., s } iu ny mu thun. 2 2 Do k ln nht l bng k = m(n + 1 m) (n + 1) n n = 2 2


n2 +2n 4

.Gi tr ln nht ny t c nu Ai l tp con kt ni ca A

.c

Nu c hai tp khc nhau th giao ca chng l mt tp kt ni cha r v s v cha c r, r+1,. . . ,s

( r,s) m 1 n m s n vi mi cp (r,s) ta ch ra rng t nht mt tp Ai c (MinAi , MaxAi ) = (r, s)

Li gii: Vi mi n Z + ta ch n biu din ca n trong h nh phn Ch rng c s ca biu din l s thay i t nht 1 k t bn tri ca biu din , v th c s c bt u bi a1 Chng ta gi gi tr thp phn ca c s ny l gi tr ui ca n vi mi mt xut hin trong biu din nh phn ca n, nu n i din cho s 2k , 2k
k 2

f (n) = max {f (j) + f (n j) + j} n 2 n

Tnh f(2000).

l mt gi tr place ca n.

48

Nguyn Hu in, HKHTN H Ni t g(n) l tng gi tr tail v place ca n Ta CMR: F(n) = g(n) t g(0) = 0 r rng g(1) =0. . . u tin ta CMR: g(n) g(j) + g(n j) + jn, j tho mn 0 j iu ny l hin nhin vi j=0 v g(0) =0 By gi ta b sung s k t ca n-j
n 2

Vi trng hp c bn ( khi n-j c k t 1) ta ch c th thay n-j. Trong trng hp (n,j) = (2,1) hoc ( n, j ) = (1;0) Trng hp (1) d c l ng. TH1: n-j v j c cng s cc k t l k +1 t a v b ln lt l s cc ch s 1 ngoi cng bn tri ca n-j v j. ta s ch ra rng g(n) = g(a + b) 2k+1 g(2k + a) + g(2k + b) + (2k + b) rng: g(a + b) g(a) + g(b) + b( iu ny ng do gi thit)

cng2k+1 k+1 . 2 V th g(a + b + 2k+1 ) g(a + b) bng v phi, ta chng minh c (2) TH2: n-j c s k t nhiu hn j Gi s n-j c k +1 k t v a = n j 2k . Ta cn CM rng: g(a + j + 2k ) g(a + 2k ) + g(j) + j. Ta bit theo gi thit:g(a + j) g(a) + g(j) + min {a, j} .

Do ta cn chng minh iu kin l:g?(a + j + 2k ) g(a + j) g(a + 2k ) g(a) + j min {a, i} (3)

Theo TH1 v phi: g(a + 2k ) g(a) = 2k k + a 2 k k Do ,v phi bng: 2 2 + a + j min {a, j} = 2k k + max {a, j} 2

Biu din nh phn ca a + b + 2k+1 ging nh biu din nh phnca a+b vi vic thm 1 vo 2k+1 v tr Do g(a + b + 2k+1)bng g(a+b)cngvi gi tr ui ca a+b v

Do v phi bng: 2k k + a + 2k 2 Cn i vi v tri: v a < 2k , b < 2k

Do ta c: g(a + b + 2k+1) g(a + b) g(2k + a) g(a) + g(2k + b) g(b) + 2k (2) g(2k + a) = g(a) + 2k k + a 2 v phi ta c g(2k + b) = g(b) + 2k k + b 2

.c

m
k 2

+ b + 2k = 2k+1

k+1 2

+a+b

thi olympic i Loan Vi v tri ca (3): nu a + j < 2k chng hn nh 2k k t khng c trong tng a + j + 2k th g = (a + j + 2k ) = g(a + j) + 2k do VT(3) VP(3)
k 2

49

+a+j

V th, v tri = 2k+1 k+1 2k k = 2k 2 2 Do (3) ng. Gi thit c CM hon ton

Mt khc nu c2k k t khng c trong tng a + j + 2k th g(a + j + 2k ) = g(a + j) + 2k+1 k+1 2k k 2 2


k 2

+ 2k > 2 k

k 2

+ max {a, j}

Ta CMR: Du bng xy ra khi g(n) = g(j) + g(n j) + jvi 1 s gi tr j t 2k l lu tha ln nht ca 2 nh hn n v t j = n 2k th g(n) = g(n 2k) + g(2k ) = g(n j) + n 2k = j

.c

nh phn 11111010000) ta c:f (2000) = 10864.

iu cho thy f (n) = g(n)n Vy vi vic tm gi tr place v gi tr ui ca 2000 (vi biu din

Chng 9 thi olympic Th Nh K


9.41.Tm cc b 4 s xp theo th t (x, y, z, w) ca cc s nguyn vi 0 x, y, z, w 36 x2 + y 2 z 3 + w 3 (mod 37) Li gii: Tt c cc ng d s l mod 37. Vi mi k trong khong 0 n 36 ta tm c cc cp s nguyn (x, y) vi 0 x, y 36 tha mn x2 + y 2 k. Ch rng iu ny tng ng vi (x 6y)(x + 6y) k.

Gi ta xt trng hp k = 0. a x + 6y, b x 6y. Vi bt k gi tr a {1, 2, . . . , 36} c chnh xc mt gi tr b {1, 2, . . . , 36}

y) vi y = 0 x2 + y 2 0 ( l (x, y) = (0, 0)), v vi bt k y no khc c 2 cp (x, y) nh vy. Do vy c tng cng 2.36 + 1 = 73 cp (x, y) x2 + y 2 0.

trnh (x 6y)(x + 6y) k c chnh xc l 36 nghim (x, y) khi k = 0. Ta xem xt s cp 4 (x, y, z, w) x2 + y 2 z 3 + w 3 0. C 3 cn bc 3 r1 , r2 , r3 ca 1 mod 37, l: Nu ta g l mt phn t nguyn thy mod 37 th cn bc 3 l 1, g 12 , g 24 . Vi bt k z no, ta

ab k. Vi mi cp (a, b) trong 36 cp tng ng vi mt nghim (x, y) duy nht v ta phi c x (a + b)21 , y (a + b)121 . Do vy phng

Trc ht ta xem xt trng hp k = 0. Vi mi y {0, 1, . . . , 36} ta c (x 6y)(x + 6y) 0 nu v ch nu x 6y.. V vy c mt cp (x,

.c

thi olympic Th Nh K

51

c z 3 + w 3 0 nu v ch nu w bng r1 z, r2 z hay r3 z. Do vy c 109 cp (z, w) z 3 + w 3 0, mi cp z = 0 v 3 cp z = z0

i vi miz0 {1, 2, . . . , 36}. trn ta tm ra rng c chnh xc 73 cp (x, y) x2 + y 2 0.. Do vy c 109.73 b 4 (x, y, z, w)

cp (x, y) x2 + y 2 z 3 + w 3. Vy nn c (372 109).36 b 4 (x, y, z, w) x2 + y 2 z 3 + w 3 0. V vy, c 109.73 + (372 109).36 = 53317 b 4 (x, y, z, w) x2 + y 2 z 3 + w 3 . 9.42.Cho mt vng trn tm O, 2 ng tim cn xut pht t im S nm bn ngoi ng trn c tip im l P, Q. ng thng SO giao vi ng trn ti A, B vi B gn S hn A. Cho X l mt im nm trong cung nh PB v ng SO giao vi cc ng QX v PX ln lt ti C, D. Chng minh rng: 1 2 1 + = AC AD AB

x2 + y 2 z 3 + w 3 0. Vi mi cp trong 372 109 cp (z, w) z 3 + w 3 0 c chnh xc 36

Thay BC = AB - AC, BD = AD - AB v chia na bn tri, bn phi bi ng AB, ta c AD AB AB AC = AB.AC AD.AB iu ny c ngha l 1 1 1 1 = AC AB AB AD iu ny tng ng vi ng thc cn chng minh.

PC AC BC = = BD PD AD

Do vy, P B l phn gic bn trong ca CPD. p dng nh l ng phn gic trong v phn gic ngoi ta tm ra c:

Li gii: Ko di tia PC cho ct vi cung QB ti Y. Bng php i xng cung BX v BY, n ch ra rng CP B = Y P B = BP X = BP D.

.c

52

Nguyn Hu in, HKHTN H Ni

9.43.Vi 2 s nguyn dng bt k n, p. Hy chng minh rng c chnh xc (p + 1)n+1 pn+1 hm s f : {1, 2, . . . , n} {p, p + 1, . . . , p} Li gii: Vi m {p, p + 1, . . . , p}, c (min{p + 1, p m + 1})n l hm s tha mn iu kin cho bao gm nhng gi tr ch nm trong {m, . . . , m + p}. D nhin, (min{p + 1, p m + 1})n l hm s c cc gi tr ch nm trong {m + 1, . . . , m + p}. Do vy, chnh xc (min{p + 1, p m + 1})n (min{p, p m})n l hm s tha mn iu kin cho vi gi tr m nh nht. Biu thc ny bng vi (p + 1)n pn i vi mi gi tr ca p+1, m 0 v bng (p + 1 m)n (p m)n khi m > 0. Do , tng ca biu thc (p + 1)((p + 1)n p)n
p

|f (i) f (j)| p vi tt c i, j {1, 2, . . . , n}.

trong khi tng biu thc khi m > 0 l tng

9.44.Trong tam gic nhn ABC c bn knh ng trong ngoi tip R. ng cao AD, BE, CF ln lt c di l h1 , h2 , h3 . Nu t1 , t2 , t3 ln lt l chiu di cc tip tuyn t A, B, C ti ng trn ngoi tip ca tam gic DEF. Hy chng minh: 3 ti ( )2 R 2 hi i=1 . Li gii: Cho H l trc tm ca tam gic ABC v X, Y, Z ln lt l trung im ca AH, BH, CH. Bi ng trn ngoi tip ca tam gic
3

Cng 2 tng ny li ta c tng cc hm s tha mn iu kin cho l (p + 1)n+1 pn+1 .

m=1

((p + 1 m)n (p m)n ) = pn

.c

khi m 0 l

thi olympic Th Nh K

53

DEF l ng trn 9 im ca tam gic ABC, n qua cc im X, Y t1 v Z. Do t2 = AX.AD = AX.h1 hoc ( h1 )2 = AX. Ta c th tm 1 c cc biu thc tng t vi BX v CX. Vy nn bt dbgr thc cn chng minh tng ng vi 3 AX + BX + CX R 2 (nhn mi v vi 2): AH + BH + CH 3R. Cho A = , B = , C = ta c: AH = AF AC. cos = = 2R cos sin sin

Tng t, BH = 2R cos v CH = 2R cos , bt ng thc cn chng minh tng ng vi 3 cos + cos + cos . 2 trong khong (0, ). Do , theo bt ng thc Jensen ta c v tri ca 2 bt ng thc cui cng ny t cc i khi 3 gc u bng , trong 3 3 trng hp v tri bng 2 . Vy bt dng thc cui cng ny l ng ng thi bt ng thc cn chng minh cng l ng. 9.45.(a) Chng minh rng vi mi s nguyn dng n, s cp ca s nguyn xp theo th t tha mn x2 xy + y 2 = n l hu hn v chia ht cho 6.] [(b) Tm tt c cc cp s nguyn (x, y) xp theo th t tha mn x2 xy + y 2 = 727. Li gii: (a) Bt k nghim (x, y) u phi tha mn bt ng thc: n = x2 xy + y 2 = c rt nhiu nghim hu hn. (x y)2 x2 + y 2 x2 + y 2 + 2 2 2

v rt nhiu cp hu hn (x, y) tha mn c iu kin ny. Do vy

Nh rng tam gic ABC l tam gic nhn v hm t cos t l lm

.c

54

Nguyn Hu in, HKHTN H Ni Tip n ta chng minh rng s nghim chia ht cho 6. Nu (x, y) l nghim th (y, y x) cng l nghim. Php bin i tuyn tnh ny l kh nghch, do n hon v tt c cc nghim v ta c th chia cc nghim ra thnh cc lp vi mi lp di dng: (x, y), (y, y x), (y x, x), (x, y), (y, x y), (x y, x) i vi mt s nghim(x, y) ban u. R rng chng minh khng c 2 trong 6 nghim mi lp bng nhau tr khi x = y = 0 l khng th. Do vy mi lp c 6 phn t ring bit v kt qu l c chng minh. (b) Bt k nghim no vi x2 xy+y 2 = 727 ta c th p dng c php

Do bt k nghim no cng c th bin i thnh (13, 18) bng vic p dng 2 s m t trn. Nh vy bt k nghim no trong

(13, 18) hoc (18, 13) di bin i (x, y) (y, y x), c ngha tt c cc nghim ti x2 xy + y 2 = 727 l: (13, 18), (18, 31), (31, 13), (13, 18), (18, 31), (31, 13), (18, 13), (13, 31), (31, 18), (18, 13), (13, 31), (31, 18). 9.46.Cho tam gic ABC, cc ng phn gic trong v ngoi ca gc A ln lt ct ng thng BC ti D v E. Cho F l mt im ( khc im A) ng thng AC tip xc vi ng trn c ng knh DE.

Do vy, 2908 3x2 phi l s chnh phng, v n khng th chia ht cho 3. Do 3x2 x2 xy + y 2 = 727 ta bit c thm rng 2181 2908 3x2 2908 vi 46 < 2908 3x2 < 54. Kim chng nhng kh nng ta thy rng ch 2908 3x2 = 49 c nghim nguyn x, kt qu l ta c nghim duy nht(13, 18) ca phng trnh.

cc nghim nh vy vi y 0 x |y|. Sp xp li ng thc cn tm ta c x2 xy + y 2 727 = 0. Xem xt ng thc ny l mt ton phng trong y, ta c th p dng phng trnh bc 2 tm ra rng: x 2908 3x2 y= 2

bin i (x, y) (y, y x) nh phn (a) v c th (x, y) (y, x), c c nghim (x, y) khc vi y 0 x |y|. Gi ta tm tt c

.c

thi olympic Th Nh K

55

V tip tuyn ti A vi ng trn ngoi tip ca tam gic ABF v giao vi ng trn ti A v G. Chng minh rng AF = AG. Li gii: Ta chng minh cho trng hp C, B v E thng hng. Theo th t ta chng minh cho cc trng hp khc tng t. Gi O l tm ca . Theo nh l v ng phn gic trong v ngoi ca tam gic, ta c: CD CA CE = = DB AB BE Do vy CD(CE CB) = CD.BE = CE.DB = CE(CB CD) hoc ( cng CD (CE + CB) cho c 2 v). 2CD.CE = CB(CD + CE) Bi v CD + CE = 2CO, ta c: CD.CE = CB.CO Mt khc, theo

AG. Suy ra php nghch o a F, giao gia v 1 , ti giao F ca l1 v l2 ; hn na php nghch o (php bin i ngc) a G, giao ca 1 v AG, ti giao ca l1 v AG. Php di xng qua trung im ca AO a l1 ti chnh n v l2 ti AG; Do vy, s phn chiu ny a OF ti AG , c ngha l OF = AG. Bi F nm trn trung trc ca AO, ta cng c OF = AG. Vy nn AF = AG, iu ny ng ngha vi AF = AG. Kt qu c chng minh. 9.47.Hy ch ra rng c th ct bt k lng tr tam gic c chiu di v hn bi 1 mt phng cho ra kt qu thit din l mt tam gic u.

trc ca AO. K tip, php nghch o a 1 (vng trn i qua A, qua O v tip xc vi AG) ti l2 khng i qua A, qua O v song song vi

nghch o l mt ng trc giao vi AO v bao gm 1 im P trn tia AO vi AP = AO/2. Ni cch khc, nh l1 ca l ng trung

tia AO. l mt vng trn qua A, im trc giao vi AO v bao gm 1 im P nm trn AO vowis AP = 2AO. Do vy, nh ca n di php

.c

nh l tch im p dng vi C v ta c: CD.CE = CA.CF , suy ra CB.CO = CA.CF . Do vy, theo nh l tch im vi cc im A, B, O, F nm trn ng trn 1 no . Ta thc hin php nghch o A vi

56

Nguyn Hu in, HKHTN H Ni Li gii: Gi s rng mt mt phng trc giao vi cc cnh ca lng tr ti A, B, C v t a = BC, b = CA, c = AB v khng mt tnh tng qut gi s a b c. Vi t 0, xc nh: f (t) = Khi : Mt khc, ta c f (b) = a2 + (t + c2 b2 + t2 )2

c2 + t2

f (0) =

nn tn ti t0 sao cho f (t0 ) = 0. By g ta cho B nm trn 1 cnh vi B, cch B 1 khong l t. C nm cng cnh vi C v cch C 1 khong l c2 b2 + t2 v nm trn gi i din ca mt phng (ABC) tnh t 0 B. Theo nh l Pitago ta c: AB = c2 + t2 ; AC = 0 a2 + (t0 + b2 + (c2 b2 + t2 ) = 0 c2 b2 + t2 )2 = 0

a2 + c2 b2 c2 0 a2 + (b + c)2 c2 + b2 > 0. Bi f l lin tc

c2 + t2 ; 0

Li gii: Hng t gic sao cho AB nm ngang v nm pha trn CD, A nm pha ty so vi B. Trc ht ta cho l AL > BN, ni cch khc N nm pha bc (mc d khng nht thit phi chnh bc) so vi L. Gi s ngc li th c 1 on nm ngang vi im cui tri L v im cui phi trn MN vi di AB. Mt khc, di ca on ny ln hn khong cch gia LK v MN , iu ny c gi thit l AB. Do vy ta c s tri ngc nhau v AL > BN. Tng t ta cng c th rt ra c AM > DK. Dng P v Q sao cho cc t gic BMPN v DKQL l cc hnh ch nht. Ta bit t phn trn rng P nm ng bc so vi Q. Dng R v S sao

gic ALM v NCK l giao nhau cn cc ng trn ngoi tip ca tam gic LDK v MBN th khng giao nhau.

9.48.Cho hnh vung ABCD, cc im M, N, K, L ln lt nm trn cc cnh AB, BC, CD, DA sao cho MN song song vi LK v khong cch gia MN v LK bng AB. Hy ch ra rng cc ng trn ngoi tip ca tam

V vy mt phng (ABC) p ng c yu cu bi.

.c

BC =

c2 + t2 0

thi olympic Th Nh K

57

cho R pha ng nam so vi Q v sao cho t gic PRQS l hnh ch nht c cc cch song song vi cc cch ca hnh vung ABCD. ch ra rng cc ng trn ngoi tip ca tam gic ALM v NCK l giao nhau, ta thy rng cc ng trn b chn bi cc ng trn ngoi tip tam gic ALM v NCK ln lt bao hm cc hnh ch nht ALRM v CKSN. V vy cc ng trn ny u cha hnh ch nht PRQS. Do cc min trong ca cc ng trn ngoi tip tam gic ALM v NCK l giao nhau vy nn cc ng trn ngoi tip cng phi giao nhau. Gi ta chng minh rng ng trn ngoi tip 1 ca tam gic MBN v ng trn ngoi tip 2 ca tam gic LDK l khng giao nhau. Ch rng chng cng ln lt l ng trn ngoi tip ca hnh ch nht BMPN v DKQL. Cho l1 l tip tuyn vi 1 ti P, l2 l tip tuyn vi 2 ti Q. V MN song song vi LK do vy BP song song vi QD. Do

x, y R.

Li gii: Ta cho hm f (2n x) g(x) = lim n 2n tha mn iu kin u bi. Vic trc tin cn lm l ch ra rng tn ti vi mi x. Thc t ta c th chng minh c iu ny v ng thi cng chng minh c |f (x) g(x)| 1

vittcx, y R. Chng minh rng tn ti mt hm g :R R tha mn |f (x) g(x)| 1 vi mi x R, v g(x + y) = g(x) + g(y) vi mi

|f (x + y) f (x) f (y)| 1

9.49.Cho f : R R l mt hm tha mn :

.c

nm pha bn phi ca l2 ; mt khc mi im trn 2 nm trn hoc bn tri ca l2 . Vy nn 1 v 2 khng th giao nhau.

l1 vung gc vi BP v l2 vung gc vi QD nn ta c l1 song song vi l2 . Vy nn mi im ca 1 nm trn hoc gi bn phi ca l1 cng

58

Nguyn Hu in, HKHTN H Ni i vi tt c x. Trc ht thy rng t x = y = 2m x0 trong bt ng thc cho ca f ta c: f (2m+1 x0 ) 2f (2m x0 ) 1 Chia cho 2m+1 ta c: 1 f (2m+1 x0 ) f (2m x0 ) m+1 2m+1 2m 2 Vi bt k x xc nh no, xem xt tng v hn:
m=0

f (2m+1 x) f (2m x) 2m+1 2m

V cc tr tuyt i ca cc s hng b chn bi mt dy 1 , 1 , . . ., tng 2 4 ca chng tin ti 1, tng ny hi t tuyt i v cng b chn bi 1.
n n

lim

m=0

.c

f (2m+1 x) f (2m x) 2m+1 2m

Gi ta c th ly ra hng s f(x) t

Suy ra gii hn trong biu thc cui cng ny hi t v ngu nhin n cng chnh l gii hn ta mun dng xc nh g(x). Hn na, ta thy phn trn rng lng cui cng ln nht bng 1, v vy ta c: |g(x) f (x)| 1 Tip tc thy rng g(x + y) = g(x) + g(y)vi mi x, y. Thy rng: g(x + y) g(x) g(y) = lim f (2n x) 2n y f (2n (x + y)) lim lim n n n 2 n 2n 2n n n n f (2 (x + y)) f (2 x) f (2 y) = lim n 2n

lim

t
lim

f (2n+1x) f (x) n 2n+1

Tng v hn trong gii hn bng ( f (2n+1 x) f (x)), c ngha l 2 f (2n+1 x) f (x) 2n+1

Mt khc, theo nh ngha tng v hn bng

n+1

thi olympic Th Nh K Theo iu kin cho, |f (2n (x + y)) f (2n x) f (2n y)| 1

59

vi n bt k, hng s trong gi hn ca biu thc cui cng nm trong khong 21 v 21 . V n n 1 =0 lim n 2n suy ra gii hn trong biu thc cui l bng 0. Do vy g(x + y) = g(x) + g(y).

.c

Nguyn Hu in

OLYMPIC TON NM 2000 (Tp 3)


t
NH XUT BN GIO DC

.c

33 THI V LI GII

v n m a t h .c o m

Li ni u
th gi lnh lamdethi.sty ti bin son mt s ton thi Olympic, m A cc hc tr ca ti lm bi tp khi hc tp L TEX. ph v cc bn ham hc ton ti thu thp v gom li thnh cc sch in t, cc bn c th tham kho. Mi tp ti s gom khong 30 bi vi li gii. Tp ny c s ng gp ca Nguyn Vn Hu, L Th Thu Hin, Nguyn Trung Hiu, Nguyn Th Mai Hoa, Nguyn Vn Huy, Nguyn Thng Huyn Rt nhiu bi ton dch khng c chun, nhiu im khng hon ton chnh xc vy mong bn c t ngm ngh v tm hiu ly. Nhng y l ngun ti liu ting Vit v ch ny, ti c xem qua v ngi dch l chuyn v Rt nhiu on v mi hc TeX nn cu trc v b tr cn xu, ti khng c thi gian sa li, mong cc bn thng cm.

ngnh Ton ph thng. Bn c th tham kho li trong [1].

H Ni, ngy 2 thng 1 nm 2010 Nguyn Hu in

51 89/176-05 GD-05

.c

M s: 8I092M5

Mc lc
Li ni u . . . . . . . . . . . . . . . . . . . . . . . . . . . . . . . . . . . . . . . . . . . . Mc lc . . . . . . . . . . . . . . . . . . . . . . . . . . . . . . . . . . . . . . . . . . . . . . . 3 4 5 11 20 26 30 34 40 42

Ti liu tham kho. . . . . . . . . . . . . . . . . . . . . . . . . . . . . . . . . . . .

Chng 7. thi olympic Anh . . . . . . . . . . . . . . . . . . . . . . . . . . . .

Chng 6. thi olympic Petecbua . . . . . . . . . . . . . . . . . . . . . . .

Chng 5. thi olympic a Trung Hi . . . . . . . . . . . . . . . . .

Chng 4. thi olympic o - Balan . . . . . . . . . . . . . . . . . . . . .

.c

Chng 3. thi olympic Chu Thi Bnh Dng . . . . . .

Chng 2. thi olympic Vit Nam . . . . . . . . . . . . . . . . . . . . . .

Chng 1. thi olympic Hoa K . . . . . . . . . . . . . . . . . . . . . . . . .

Chng 1 thi olympic Hoa K


1.1. Mt b bi c R qun , W qun trng v B qun xanh. Mt ngi chi thc hin vic rt cc qun bi ra khi b bi. Vi mi lt, anh ta ch

B) v tm tt c cc cch chi c th t c s tin pht Li gii: Ta s chng minh s tin pht ti thiu phi tr l min(BW,2 WR,3RB) D nhin s tin pht ny l t c, tng ng vi 1 trong 3 cch rt bi sau:(bb..bbrr..rr ww.. ww);(rr..rr ww.. wbb. . . )( ww.. wwbb..rr)Vi mi mt cch rt bi, ta nh ngha chui xanh l mt on lin tip cc qun bi mu xanh c rt ra khi b bi (tc trong mt s lt lien tip, ta ch rt qun xanh ra). Tng t, ta c nh ngha chui , chui trng. By gi ta s chng minh 3 b : B 1: Vi mi cch rt bi cho trc, ta c th thc hin 1 cch rt bi khc, trong 2 chui cng mu c gp vo nhau m khng lm tng s tin pht

tin pht bng ba ln s qun xanh cn li trong b bi Hy xc nh tng s tin pht ti thiu m ngi chi phi tr (ph thuc vo R, W,

cn li trong b bi - Nu l bi c rt c mu trng, tin pht bng hai ln s qun cn li trong b bi - Nu l bi c rt c mu ,

.c

c php rt ng 1 l bi, v phi chu mt s tin pht cho lt rt bi : - Nu l bi c rt c mu xanh, tin pht bng s qun trng

Nguyn Hu in, HKHTN H Ni Ta s chng minh trong trng hp gp 2 chui , cc trng hp khc hon ton tng t. Gi s gia 2 chui c w qun trng v b qun xanh. By gi, nu ta chuyn mt qun t chui th nht sang chui th 2, s tin pht s tng them 2w 3b (do mi lt rt qun trng phi tng them tin pht l 2 bi s xut hin ca 1 qun mi, v qun c chuyn i nm sau b qun xanh nn khng phi chu 3b tin pht). Nu , ta ch vic chuyn tt c cc qun t chui 1 sang chui 2. Ngc li, ta s chuyn tt c cc qun t chui 2 sang chui 1. Trong c 2 trng hp, 2 chui c gp vo nhau v s tin pht khng b tng thm. B 2: Cch chi ti u khng tn ti chui (tc khng xy ra trng hp rt 1 qun ngay sau 1 qun trng)

m khng lm tng s tin pht. Nhng cch chi hin ti l ti u, do vy ta phi c: (1) Gp 2 chui trng li v ta c 1 cch rt bi cng c s tin pht ti thiu: . Theo b 1, ta c th gp 2 chui vi nhau m s tin pht khng tng them. Nhng do cch chi ny l ti u, nn ta phi c , mu thun vi (1). Vy iu gi s l sai, b 2 c chng minh. Tc l bt k cch chi ti u no cng ch c ti a 4 chui. Kt hp vi b 2, cch chi ti u nu phi rt qun u tin l Cch chi ny l ti u khi v ch khi v , Tng t, ta cng c cch chi ging nh trn nu qun u tin uc rt l qun trng hoc qun xanh./.

chng minh tng t trong cc trng hp khc). Theo b 1, ta c th gp 2 chui , hoc 2 chui trng li vi nhau

bi u tin c rt c mu (cc trng hp khc chng minh tng t). By gi, ta gi s rng cc chui c rt c gi tr (theo th t,

hn B 3: Cch chi ti u (t tin pht nht) s c t hn 5 chui Gi s tn ti 1 cch chi ti u c 5 chui tr ln. Gi thit rng qun

.c

iu ny l hin nhin, v nu xut hin lt rt bi nh vy ta thay i chui bng chui , ta thu c 1 cch chi mi c s tin pht nh

thi olympic Hoa K c cc s thc m phng trnh x2 2mx 4(m2 + 1) c ng ba nghim phn bit. x2 4x 2m(m2 + 1) = 0

Li gii: p n: m = 3. Cho hai tha s v tri ca phng trnh bng 0 ta nhn c hai phng trnh a thc. t nht mt trong cc phng trnh ny phi nghim ng vi gi tr x no x l nghim ca phng trnh ban u. Nhng phng trnh ny c th vit di dng (x m)2 = 5m2 + 4

(1)v (x 2)2 = 2(m3 + m + 2) (2). Ta c ba trng hp m phng trnh ban u c th c 3 nghim phn bit: Phng trnh (1) c nghim

vy m = 2 hoc r = m2 +1. Tuy nhin, trong trng hp th nht th c hai phng trnh bc hai ca ta tr thnh (x 2)2 = 24, v v vy, ta ch

tha s ca c hai biu thc x2 2mx4(m2 +1) v x2 4x2m(m2 +1). Tr hai biu thc ny cho nhau ta nhn c x r l mt tha s ca (2m 4)x (2m3 4m2 + 2m 4), hay (2m 4)r = (2m 4)(m2 + 1). V

c cc nghim l x=-6, -4, 10, tha mn yu cu ca bi ton.

hay (m + 1)(m 3)(m2 + 1) = 0. Do m = 1 hoc 3. Trng hp m=-1 c ch ra khng tha mn. V vy, ta ch c m=3. Khi cc phng trnh ca ta tr thnh (x 3)2 = 49 v (x 2)2 = 64, chng

thu c hai nghim phn bit. Vy ta phi c r = m2 + 1. Khi , thay vo ng thc (r2)2 = 2(m3 +m+2), ta c (m2 1)2 = 2(m3 +m+2)

ca bi ton. Xt trng hp th ba, gi r l nghim ca phng trnh th xr l mt

(1) tr thnh (x + 1)2 = 9, tc l x=2, -4. Nhng iu ny c ngha l phng trnh ban u ca ta ch c nghim l 2 v -4, tri vi yu cu

mi gi tr thc m. V vy ta phi c m=-1 trng hp ny xy ra. Khi nghim duy nht ca phng trnh ny l x=2 v phng trnh

.c

nhin 5m2 + 4 = 0 khng th tha mn vi mi gi tr thc m. Trong trng hp th hai, ta phi c 2(m3 + m + 2) = 0; m3 + m + 2 phn tch thnh (m + 1)(m2 m + 2) v tha s th hai lun dng vi

kp hoc phng trnh (2) c nghim kp hoc hai phng trnh c mt nghim chung. Tuy nhin, trng hp th nht khng xy ra v hin

Nguyn Hu in, HKHTN H Ni 1.2. Cho ABC l tam gic u c din tch bng 7. Gi M, N tng ng l cc im trn cnh AB, AC sao cho AN=BM. Gi O l giao im ca BN v CM. Bit tam gic BOC c din tch bng 2. 2 (a) Chng minh rng M B hoc bng 1 hoc bng 3 . AB 3 (b) Tnh gc AOB. Li gii: (a) Ly im L trn BC sao cho CL=AN v gi P, Q ln lt l giao im ca CM v AL, AL v BN. Php quay vi gc quay 120o quanh tm ca tam gic ABC bin A thnh B, B thnh C, C thnh A; php quay ny cng bin M thnh L, L thnh N, N thnh M v bin O thnh P, P thnh Q, Q thnh O. Do OPQ v MLN l cc tam gic u ng tm vi tam gic ABC. Suy ra BOC=-MOC= 2 . V vy, O nm trn 3 ng trn i xng vi ng trn ngoi tip tam gic ABC qua BC. C nhiu nht hai im O trn ng trn ny v nm trong tam gic ABC t l khong cch t O ti BC v t A ti BC bng 2 , t l ny cng 7 l t l din tch ca cc tam gic OBC v ABC. V vy ta ch ra rng MB 1 = 3 hoc 2 tng ng vi cc v tr ca im O, v khng c t l no AB 3 khc (tc l khng c hai im M cho cng mt im O. Nu M B = 1 AB 3

2 c iu phi chng minh. Tng t, nu M B = 3 , theo nh l Menelaus AB [BOC] BO 2 1 BO ta c BN = 6 , do [BN C] = BN = 6 . Suy ra [BOC] = 6 CN = 7 . (b) M B = 3 th 7 7 [ABC] 7 CA AB MONA l mt t gic ni tip do A= pi v O = P OQ = 2 . Do

1 th AN = 3 , p dng nh l Menelaus cho tam gic ABN v ng thng AC [BOC] BO 2 BO CM, ta c ON = 3 , do [BN C] = BN = 3 . Suy ra [BOC] = 3 CN = 7 v ta 4 7 [ABC] 7 CA

.c

AOB=AOM+MOB=ANM +P OQ = ANM + . Nhng M B = 1 v 3 AB 3 AN 1 = 3 nn d dng thy c N l hnh chiu ca M trn AC. V vy AC

ANM = v AOB = 5 . Lp lun tng t i vi trng hp cn li, 2 6 ta c ANM = 6 v AOB = . 2


3 1.3. Cho f (x) = x2 2ax a2 4 . Tm tt c cc gi tr ca a |f (x)| 1

vi mi x [0; 1].

1 Li gii: p n: 2 a 42 . th ca f(x) l mt parabol c im cc tiu (c ngha l h s a 1 3 m) v nh l (a;f(a)). T f (0) = a2 4 ta c |a| 2 f (0) 1.

Gi s a 0 th parabol ca ta tng nghim ngt trong khong t 0

thi olympic Hoa K

phm vi theo yu cu ca bi ton, tc l f(1) nm trong gii hn ny. 9 T a 1 ta c 1 < (a + 1)2 4 v v vy f (x) = 1 5 (a + 1)2 < 1 . 2 4 4

1 ca bi ra khi 2 a 0. Vi a > 0, f gim vi 0 x a v tng vi a x 1. V vy ta cn ch ra gi tr nh nht ca f(a) nm trong

1 n 1, do f (1) 1. Nhng ta c 1 a + 1 1, 4 (a + 1)2 1, 2 1 5 5 (a + 1)2 1. T 4 (a + 1)2 = f (1), ta c f tha mn iu kin 4 4

1.4. K hiu u(k) l c l ln nht ca s t nhin k. Chng minh rng 1 2n


2n

3 Mt khc, f (a) = 2a2 4 nn ta phi c a 42 f (a) 1. Ngc li, nh gi f(0), f(a), f(1) ta ch ra c f tha mn iu kin ca bi ra khi 0 < a 42 .

k=1

u(k) 2 . k 3

k=1

1.5. Tm tt c cc s thc tha mn h x3 = 2y 1 y 3 = 2z 1 z 3 = 2x 1.

Li gii: Trc ht ta ch ra rng x = y = z. Gi s tri li rng x = y. Nu x > y, th y =


(x3 +1) 2

1 2n

2n

u(k) 2 2 = 4n + (1 4n ) > . k 3 3

T tng ca chui hnh hc ta c

>

(y 3 +1) 2

u(k) 1 1 = n+ n 2 k=1 k 4

2n

vi i n 1, v mt gi tr sao cho v(k) = 2n . Do , v tri bng


n1

.c

Li gii: t v(k) l c ln nht ca k c dng ly tha ca 2, nn u(k)v(k) = k. Trong {1, 2, ..., 2n} c 2ni1 gi tr ca k sao cho v(k) = 2i

= z, nn y > z, v tng t z > x,

o
i=0

2ni1 . 2n+i

10

Nguyn Hu in, HKHTN H Ni mu thun. Tng t, nu x < y th y < z v z < x, mu thun. Nn cc nghim ca h phng trnh c dng x = y = z = t vi t l nghim ca phng trnh t3 = 2t 1. Vy, nghim ca h phng trnh l 1 + 5 1 5 x = y = z = t, t 1, . , 2 2

1.6. Tm s t nhin a nh nht phng trnh sau c mt nghim thc: cos2 (a x) 2 cos (a x) + cos x 3x +2=0 cos + 2a 2a 3

Li gii: Li gii: Gi tr nh nht ca a l 6. Phng trnh tha mn

(cos(a-x)-1)2 + cos

Trong c hai trng hp ta u c 3x chia ht cho 2, v vy x phi chia ht cho 2 v a cng phi tha mn iu . Hn na, c hai trng hp ta cng u c -2a v 4a cng phi chia ht cho 3, v th a phi chia

ht cho 3. Tm li ta c 6 phi l c ca a v a=6 l gi tr nh nht cn tm.

vi

6a ta c 3x 4a (mod12a). x + 3 = (2k + 1) v nhn hai 2a

bng 0, ta thy cc gi tr cosin phi nhn gi tr bng 1 v -1. Nu cos x + = 1 th x + = 2k vi gi tr k nguyn v nhn hai v 2a 3 2a 3 Khi th nu cos x + = 1 th 2a 3 6a v vi ta c 3x 4a (mod12a).

Do c hai s hng v tri u khng m nn ng thc xy ra th chng phi cng bng 0. T cos(a-x)-1 = 0 ta c x phi l mt s nguyn ng d vi a trong php chia cho 2. T s hng th hai

.c

x 3x cos +1 + 2a 2a 3

khi a=6, x=8. chng minh a l gi tr nh nht, ta vit phng trnh di dng =0

Chng 2 thi olympic Vit Nam


m
(a) Xc nh qu tch trung im ca on thng M1 M2 (b) Gi P l giao im ca cc ng thng O1 M1 v O2 M2 . ng trn ngoi tip M1 P M2 ct ng trn ngoi tip O1 P O2 ti P v mt im khc l Q.Chng minh rng Q l im c nh khng ph thuc vo v tr ca M1 v M2 . Li gii:

cho khi quay theo chiu kim ng h s o ca gc M1 OM1 v M2 OM2 l bng nhau.

2.7. Trn mt phng cho hai ng trn 1 , 1 theo th t c tm l O1 v O2 . Cho M1 v M2 l hai im ln lt nm trn 1 , 1 sao cho O1 M1 v O2 M2 ct nhau. Cho M1 v M2 ln lt l hai im trn 1 , 1 sao

.c

12 Q M1 M O1 O M1

Nguyn Hu in, HKHTN H Ni

M2 M2 M O2

quanh 1 im qua mt s gc. Khi :

vi tam gic QO1 O2 . Do : q o1 q m1 = q o2 q m2 Hay tng ng: q o1 (q m1 ) (q o1 ) o1 m1 o1 m1 = = = q o2 (q m2 ) (q o2 ) o2 m2 o2 m2

V hai ng thng O1 M1 vO2 M2 ct nhau, o1 m1 = o2 m2 v ta c th gii phng trnh ny tm c gi tr duy nht ca q, suy ra Q l im c nh khng ph thuc vo v tr ca M1 v M2 .

Tng t: QM2 M1 = QO2 O1 , suy ra tam gic QM1 M2 ng dng

QM1 M2 = QP M2 = QP O2 = QO1 O2

t suy ra qu tch im M l ng trn tm O bn knh OM . (b)Chng ta s dng trc tip cc gc c modun . Ch rng:

m=

1 1 m1 + m2 = (o1 + z(m1 o1 )) + (o2 + z(m2 o2 )) = o + z(m o) 2 2 2

.c

O ln lt l trung im ca cc on thng: M1 M2 , M1 M2 v O1 O2 . m o2 Ta cng t z = m1 o1 = m2 o2 sao cho php nhn bi z l php quay m o1

(a)Chng ta dng cc s phc, mi im c k hiu l ch in hoa ta t tng ng vi mt s phc c k hiu l ch in thng. Gi M , M v

thi olympic Vit Nam

13

2.8. Gi s rng tt c ng trn ngoi tip ca bn mt ca mt t din c bn knh bng nhau. Hy ch ra rng hai cnh i bt k ca mt t din l bng nhau. Li gii: Trc ht ta chng minh rng vi bn im khng ng phng bt k X, Y , Z, W ta c: XY Z + Y ZW + ZW X + W XY < 2 X

o v n m a t
AC = sinADC 2R

m h .c
W

Z Tht vy: p dng bt ng thc tam gic cho cc gc ta thy rng: XY Z + Y ZW + ZW X + W XY < (ZY W +W Y X)+Y ZW +(XW Y +Y W Z)+W XY = (ZY W +Y W Z+Y ZW )+(XW Y +W Y X+W XY ) = + = 2. Gi R l bn knh chung ca cc ng trn ngoi tip bn mt t din. Ta nh ngha rng hai gc ca t din ABCD gi l i din vi cng mt cnh l hai gc v d nh ABCv ADC, ta c: sinABC =

( do nh l hm sin). Do hai gc bt k i din vi cng mt cnh hoc bng nhau hoc b nhau. Hn na, rng nu XZ v Y W l hai cnh i ca t din XYZW th suy ra (XY Z + ZW X + (Y ZW W XY < 2)) nn khng th c trng hp hai gc i din vi cng mt cnh XZ v vi cnh

14

Nguyn Hu in, HKHTN H Ni Y W u l b nhau. Ni cch khc, nu cc gc i din ca cnh XZ l b nhau th hai gc i din vi cng mt cnh Y W l bng nhau.

By gi ta gi s ngc li rng c hai gc i din vi cng mt cnh v d nh ABCv CDA l b nhau cn tt c cc cp gc i din vi cng mt cnh khc l bng nhau, ta c:

iu ny tri vi gi s. Do bn cnh cc gc i din vi cnh AC th cn mt s cp gc i din vi cng mt cnh khc l b nhau.Theo phn l lun trn th cp gc i din vi cnh BD khng th b nhau nn chng phi bng nhau. Do cp gc i din vi cng mt cnh trong s cc cnh AB, AD, CB v CD l b nhau. Khng gim tnh tng qut, gi s cp gc i din vi cnh AB l b nhau suy ra cp

= ( pi CAB DAC) + ( ACB DCA)

= ( CAB ACB) + ( DAC DCA)

= ABC + CDA =

BCD + DAB = ( CDB DBC) + ( ADB DBA)

.c

thi olympic Vit Nam gc i din vi cnh CD l bng nhau. Hn na: CDB = DCB DBC = DAB DAC = ( ABD ADB) ( ACD ADC) = ABD + ACD + ADB + ADC = ABD + ACD + ( ACB) + ( ABC) = ABD + ACD + ( ACB ABC) = ABD + ACD + CAB

15

BCD + DAB = ABC + CDA Suy ra 2DAB = 2ABC hay DAB = ABC. Cho nn, DB = 2RsinDAB = 2RsinABC = AC. Tng t ta cng c: DA = BC, DC = BA. l iu phi chng minh. 2.9. Cho hai ng trn (C1 ), (C2 ) ct nhau ti P v Q. Tip tuyn chung ca hai ng trn( gn P hn Q) tip xc vi (C1 ), (C2 ) ln lt ti A v B. Tip tuyn ca(C1 ) ti P ct (C2 ) ti E ( khc P ), tip tuyn ca (C2 ) ti P ct (C1 ) ti F ( khc P ). Gi H, K ln lt l

iu ny v l. Do cc gc i din vi cng mt cnh ca t din l bng nhau. Nh chng ta l lun trn th trong trng hp ny ta c:

= 2.

= 3

ADB + BDC + CDA = ( ACB) + ( BCA) + ( CBA)

.c

ADB, BDC v CDA l cc gc nh ca mt t din nn (ADB + BDC) + CDA < ADC + CDA < 2. Nhng:

Suy ra CDB < CAB. V cc CDB v CAB l khng bng nhau nn chng phi b nhau. By gi ta li c cc gc

16

Nguyn Hu in, HKHTN H Ni hai im nm trn hai tia AF v BE sao cho AH = AP , BK = BP . Chng minh rng nm im A, H, Q, K, B nm trn cng mt ng trn. Li gii: B A T R P H Q E F V iu kin c tnh cht i xng nn ta ch cn chng minh ABKQ

Nn t gic ABRQ l ni tip ng trn. Chng ta s chng minh

BP R = BAP + P BA = AQP + P QB = AP T + P EB = RP E + P ER = P RB Do tam gic BP R l tam gic cn vi BP = P R suy ra R = K. l iu chng ta cn chng minh. 2.10.Cho a, b, c l cc s nguyn dng i mt ngyn t cng nhau. Mt s nguyn n c gi l stubborn nu n khng c biu din di

P T ; t gic ni tip ng trn ta c:

rng K = R. Tht vy: S dng gc ngoi tam gic ABP v CP R; tip tuyn AB v

QAR = QAP = QP C = QBC = QBR

ng thng AP v tia BE. ng thng AB v BE ct nhau ti T . S dng tnh cht ca tip tuyn v t gic ni tip ta c:

.c

l ni tip ng trn. Chng ta s dng gc c hng modulo . Gi R l giao im ca

thi olympic Vit Nam dng: n = bcx + cay + abz

17

vi x, y, z l cc s ngyun dng bt k. Hy xc nh qua hm s ca a, b v c s lng cc s nguyn stubborn . Li gii: Chng ta c th khng nh c rng bt k s ngyun n no u c th biu din di dng n = bcx + cay + abz trong x, y, z l cc s nguyn v 0 < y b; 0 < z c cn x c th m. Tht vy,v a v bc nguyn t cng nhau nn ta c th vit: n = an + bcx0 vi n , x0 l cc s nguyn. v b v c l nguyn t cng nhau nn n = cy0 + bz0 vi y0 , z0 l cc s nguyn. Do n = bcx0 = cay0 + abz0 . Chn s nguyn , sao cho 0 < y0 + b b v 0 < z0 + c c, khi ta c :

Tip theo chng ta chng minh rng ng mt na cc s nguyn dng trong S = [bc + ca + ab; 2abc]. lm c iu ny ta i chng minh n S l stubborn khi v ch khi f (n) = (2abc + bc + ca + ab) n

khng l stubborn. iu kin cn: Gi s rng n l stubborn v biu din f (n) =

b + 1 y0 v c + 1 z0 l cc s nguyn dng, nhng iu ny l khng th v n l stubborn. Do , x > 0 v f (n) khng l stubborn. iu kin : Gi s ngc li f (n) khng l stubborn v n cng khng l stubborn. Biu din f (n) = bcx0 + cay0 + abz0 v n =

bcx + cay + abz vi 0 < y b, 0 < z c. Nu x khng dng th chng ta c th vit n = bc(1 x) + ca(b + 1 y) + ab(c + 1 z), vi 1 x0 ,

Suy ra x > 0 nh vy n khng l stubborn khi n > 2abc.

2abc < bcx + cay + abz bcx + cab + abc = bcx + 2abc

s nguyn v 0 < y b, 0 < z c. Khi :

stubborn. Mt khc ta cng khng nh rng mi s nguyn n > 2abc khng l stubborn. Chng han, biu din n = bcx + cay + abz vi cc

.c

y chnh l dng biu din ca n m ta mun. Ch rng bt k s nguyn dng no nh hn bc + ca + ab u l

n = bc(x0 a a) + ca(y0 + b) + ab(z0 + c)

18

Nguyn Hu in, HKHTN H Ni bcx1 + cay1 + abz1 vi xi , yi, zi l cc s nguyn dng. Khi : 2abc = bc(x0 + x1 1) + ca(y0 + y1 1) + ab(z0 + z1 1) t x = x0 + x1 1 v cng ty,z tng t. T ng thc trn chng t 0 bcx( mod a). V bc nguyn t cng nhau vi a nn x phi chia ht cho a suy ra x a. Tng t y b, z c, khi 2abc = bcx + cay + abz 3abc ( v l).

Tm li: c bc+ca+ab1 cc s nguyn dng nh hn bc+ca+ab l stubborn, mi s nguyn ln hn 2abc khng l stubborn, v mt na ca 2abc(bc+ca+ab)+1 cc s nguyn dng cn li l stubborn. Kt qu ca bi l tng ca: bc + ca + ab 1 + 2.11. 2abc + bc + ca + ab 1 2abc (bc + ca + ab) + 1 = 2 2

cc s nguyn dng stubborn.

Gi R+ l tp cc s thc dng v a, r > 1 l cc s thc. Gi

kin trong cu (a)) sao chof (x) > xr a1r vi mi x > 0.

Li gii: Ch rng ta c th vit li bt ng thc di dng:

1 (a) Nu f (x) < 22000 vi mi x < 22000 , chng minh rng f (x) xr a1r vi mi x > 0. (b) Xy dng mt hm f : R+ R ( khng cn tho mn iu

s rng f : R+ R l mt hm s sao cho: (f (x))2 axr f ( x ) vi mi a x > 0.

.c
2

f (x) xr a1r

m
f (x/a) (x/a)r a1r

(*)
x0 an

Gi s ngc li tc l tn ti x0 sao cho f (x0 ) > xo r a1r . t xn =

mt s dng v do f (xn ) cng l s dng. Chng ta gn x = xn vo bt ng thc v sp xp cc bt ng thc li ta c: n xn a1r n f (xn ) f (xn+1 ) r = = r f (xn ) axr axr a n n

n) v n = xf (x1r vi n 0, suy ra 0 > 1. T (*) ta c:n+1 2 vi ra n n n n 0, v bng quy np ta chng minh c rng: n 2 vi n 0. 0 n Chng ta s s dng iu ny ngay sau y, rng mi n 2 l 0

thi olympic Vit Nam

19

(b) Vi mi s thc xlun tn ti duy nht mt gi tr x0 (1; a] n sao cho x0 = an vi n l s nguyn. Gi (x) = x2 v t f (x) = 0 x (x)xr a1r . Bng php co li, chng ta cng c (x)2 = (x/a) vi mi x, ni cch khc (*) cng ng vi mi x. Ta cng c (x) > 1 vi mi x hay ni cch khc f (x) > xr a1r vi mi x > 0. l iu cn chng minh.

x0 1 ln), nhng cng vi n ln th ta li c xn = an < 22000 . iu ny tri vi gi thit tc iu gi s l sai. Do f (x) xr a1r vi mi x > 0.

(xn+1 Vi tt c cc gi tr n nh vy th ta c: f f (xn ) ) 2 hay tng ng ( do f (xn ) l dng) f (xn+1 ) 2f (xn ). Cho nn f (x) 22000 ( vi n

Vi mi n 0. n Ta thy lun tn ti N sao cho 2ar < 2 n vi mi n > N. 0

.c

Chng 3 thi olympic Chu Thi Bnh Dng


vi xi =

.c

3.12.Tnh tng S =

100 i=0

x3 i 13xi +3x2 i

o
i 100

m
i = 1, 101
101

x x Ta c th t f (x) = 13x+3x2 = x3 +(1x)3 x Cho x = xi , x = 1 xi = x101i v thm 2 phng trnh h qu ta tm ra:f (xi ) = f (x101i ) = 1. V th

Li gii: V 1 3x + 3x2 = x3 (x 1)3 = 0 x


3

50

S=

f (xi ) =
i=0

(f (xi ) + f (x101i )) = 51

3.13.Cho mt s b tr vng trn quanh ba cnh mt tam gic, mt vng mi gc, hai vng mi cnh, mi s t 1 n 9 c vit vo mt trong nhng vng trn ny sao cho i. Tng ca 4 s mi cnh tam gic l bng nhau. ii. Tng ca bnh phng ca 4 s trn mi cnh ca tam gic l bng nhau. Tm tt c cc cch tho mn yu cu ny. Li gii: Ly bt k mt s b tr cc con s, gi x, y, z l s trong gc v S1 , S2 ln lt l tng ca bn s, tng ca bnh phng bn s

i=0

thi olympic Chu Thi Bnh Dng trn mt cnh bt k. Do iu kin cho ta c:
9

21

3S1 = x + y + z +
k=1 9

k = x + y + z + 45

3S2 = x + y + z +
k=1

k 2 = x2 + y 2 + z 2 + 285

T ng thc th hai ta suy ra x, y, z hoc tt c chia ht cho 3 hoc khng c s no chia ht cho 3. Bi nguyn l Pigeouhole c hai s l ng d mod3. Ly phng trnh th nht theo mod3 ta cng suy ra 3| (x + y + z). Do x y z(mod3)

th nm cng cnh vi 8, tc l n nm trn cnh cha 2 hoc 5. V Min 72 + 92 , 72 + 52 + 82 > 126 nn s 7 phi nm trn cnh cha s 2 hoc 8. Nh vy 4 ln cc s trn 3 cnh phi l (2, 4, 9, 5); (5, 1, 6, 8); (8, 7, 3, 2) cho tng bnh phng

3.14.Cho tam gic ABC, trung tuyn AM v phn gic AN . V ng vung gc qua N ct MA, BA ti P, Q. Gi O l im m ng vung gc qua P vi BA ct AN. Chng minh QOBC Li gii: Cch 1. Nu AB = AC th QO l trung trc ca BC v yu cu phi chng minh c tho mn. Gi s AB = AC, s dng to Castesian:A(0; 0), N(1; 0). t dc ca AB l m th ca AC l 1. Vit B = (b; mb), C = (c; mc), b = c v c b, c l dng. dc

ca BC l m(b+c) . bc V P NAN v x-to ca P l 1, v P thuc ng thng AB(ng thng y = mx) nn ta c P = (1, m). Do phng trnh ca OP

l y = (x 1)/m + m, suy ra x interceptO l (m2 + 1, 0), M l

cc s trn mi cnh l 126. Cui cng, ta thy cc b s trn u tho mn.

.c

khng th v 5 > 3 s l c vit trong mi khe V th, (x, y, z) = (2, 5, 8) v S2 = 126. V 92 + 82 > 126 nn 9 khng

Nu (x, y, z) = (3, 6, 9) hay (1, 4, 7) th S2 = 137 hoc 17. Nu S2 = 137 th S2 1(mod3) suy ra ch c mt s trn ba cnh l l. iu ny

22

Nguyn Hu in, HKHTN H Ni ((b + c)/2; m(b c)/2) v n l trung im ca BC. Do phng trnh ca ng thng AM l y = m(bc) x. Bi v Q l giao im ca AM v b+c P N, ta c Q 1; m(bc) . V th dc ca P Q l b+c
m(bc) b+c m2 +11

bc . m(b+c)

1 l dc ca BC nn QOBC. Cch 2. Gi , , l s o cc gc CAB, ABC, BCA v y = BAM, z = MAC, x = MAN = |yz| 2 Nu = th QO l trung trc ca BC. Ngc li, gi s = th y = z , vi cch lm ny, ta bin i phng trnh bng cch nhn v chia vi biu thc lng gic khc 0 v = , y = z; , , y, z (0; ) S dng quan h lng gic vo cc ABC, BNO, ONQ ta c tan OQN. tan QAN = ON/QN ON/ QN/ AN = BN

BN/ AN = tan OBN

p dng lut hm sin cho ABC, ACM, ta c: sin .BM/AM sin y sin = = sin z sin sin .CM/AM Suy ra:
sin y+sin z sin ysin z

sin sin . sin +sin

tan /2 + /2 tan (/2) tan (y/2 + z/2) cot (/2) = = = tan (y/2 z/2) tan (y/2 z/2) tan /2 /2 tan /2 /2 Nu > th x = y/2 z/2 so snh biu thc cui cng vi (*), ta c: tan OQN = tan /2 /2 /2 . ta c: OQN, /2 / / 2 2 (0; /2) v t tan t l n nh, v th OQN = / / / 2 2 2 OQAB . Chng minh tng t nu >

Cho (u, v) = (y/2, z/2) ; (u, v) = /2, /2 ra:

sin(u + v)cos(u v) sin(2u) + sin(2v) tan(u + v) = = tan(u v) sin(u v)cos(u + v) sin(2u) sin(2v)

.c

Cho u, v trong khong (0; /2), ch rng

Suy ra rng BAN = /2, OBN = /2, QAN = x Do : tan OQN.tanx = tan (/2) tan (/2) ()

trong ng thc ny ta tm

thi olympic Chu Thi Bnh Dng 3.15.Cho n, k l cc s nguyn dng, n > k. Chng minh rng: 1 nn nn n! . < < n + 1 k k (n k)nk k! (n k)! k k (n k)nk

23

Li gii: S dng khai trin nh thc,ta vit: nn = (k + (n k))n n n k m (n k)nm > 0 vi mi m dng am , vi am = m=0 m Bt ng thc cho tng ng vi: nn < ak < nn n+1 Bt ng thc bn phi tho mn do nn =
n m=0

am > ak . chng minh

m=0

k=0

Do ta c iu phi chng minh.

Biu thc ny nh hn 1 khi m < k v ln hn hay bng 1 khi m k.

am = am+1

m+1

3.16.Cho mt hon v (a0 , a1 , .....an ) ca dy 0, 1, ....., n. Mt chuyn v ca ai , aj gi l hp php nu ai = 0, i > 0 v ai1 + 1 = aj . Hon v (a0 , a1 , .....an ) gi l chnh quy nu sau hu hn cc bc chuyn v hp php n tr thnh (1, 2, ...., n, 0). Vi n no th (1, n, n 1, .....3, 2, 0) l

chnh quy ?.

k m (n k)nm

Tht vy, ta chng minh rng am tng vi m k, gim vi m k, ch n n = m+1 . V th: rng nm m m+1

.c

nn =

am <

ak = (n + 1) ak

bt ng thc cn li ta ch cn ch ra ak > a0 , .....am1 , am+1 , .....am . Bi v n n

k m+1 (n k)nm1

nk m+1 . nm k

24

Nguyn Hu in, HKHTN H Ni Li gii: Vi n c nh, 0 v 1 l ci hon v (1, n, n 1, .....3, 2, 0) , v (1, 2, .....n, 0). Ta ni 0 l chm dt trong hon v 1 nu sau mt
, s chuyn v hp php ca 0 ta thu c 1 v nu khng mt chuyn , v hp php no c th p dng cho chuyn v 1 . V khng c chuyn

v hp php no c th c p dng cho 1 , nu 0 chnh quy th n chm dt trong 1 .Nh khi p dng chuyn v hp php cho 0 nhiu nht mt chuyn v hp php c th p dng cho mi hon v thu c. V th 0 chm dt trong nhiu nht mt hon v. Nu n = 1, 2, d dng kim tra (1, n, n 1, .....3, 2, 0) l chnh quy. Nu n > 2, chn, ta i hi rng 0 khng chm dt trong 1 v v th khng chnh quy. Cho k 0; n2 p dng k chuyn v hp php cho 0 suy 2

ra hon v m bt u vi 1, n, n 1, ....2k + 2, 0. Do 0 chm dt trong mi hon v bt u bi 1, n, 0 thu c sau n2 chuyn v hp 2

vo mc no, sp xp theo trt t tng. Nu s + 1| n + 1 v t > 0 th p dng n/(s + t) chuyn v hp php cho (s, t)_bc thang, qu trnh ta m ch nh s la chn bc thang Tip theo ta gi s rng s|(n + 1). Nu 2s khng chia ht cho n + 1 th p dng n/(s 2) chuyn v hp php cho (s, 0)_bc thang suy ra mt php chuyn v khc 1 m khng c thm chuyn v hp php no c th c p dng. Nu thay v 2s|(n + 1) th (s, 0)_bc thang thc s l (s, s)_bc thang ci m c th chn t (2s, 0)_bc thang Gi ta chng minh rng nu n > 2 v n l th 0 chnh quy nu v ch nu n + 1 l lu tha ca 2. V n + 1 chn nn ta vit n + 1 = 2q r, q l s t nhin v r l s t nhin l. p dng (n 1)/2 chuyn v hp

vo mc no, sp xp theo trt t tng. (3) Cho s s tip theo l s s ln nht trong 1, 2, ..., n cha phn nh

(1) Cho s s u tin 1, 2, ....s 1, 0 (2) Cho t s tip theo l t s ln nht trong 1, 2, ..., n cha phn nh

hiu. Vi mi s nguyn s > 0, t 0 sao cho s + t chia ht cho n + 1, ta xy dng php hon v gi l(s, t) _bc thang ti mt thi im nh sau: p dng (1) mt ln v lp li (2) v (3) mt cch xen k:

.c

php. By gi ta gi s rng n > 2, xt trng hp ny ta s a ra vi k

thi olympic Chu Thi Bnh Dng

25

php cho 0 dn n (2, 0)_bc thang Nu 2q > 2 th 2s|(n + 1) vi s = 21 , ......2q1 ta c th lp li dn n (2q , 0)_bc thang Nu r = 1 suy ra ta thu c 1 v 0 chnh quy. Ngc li, p dng r 2 chuyn v hp php dn n mt hon v 0 l ln cn tri ca n v th khng c mt chuyn v hp php no l c th. Tuy nhin hon v cui bt u bi 1, 2, ....2q hn 1, n. Suy ra 0 khng chm dt trong 1 v th 0 khng chnh quy Vy 0 l chnh quy khi v ch khi n = 2 hoc n + 1 l lu tha ca 2.

.c

Chng 4 thi olympic o - Balan


4.17.Tm tt c cc s nguyn dng N sao cho s ch chia ht cho 2 v 5 v N + 25 l s chnh phng.

Trng hp 2: b 1 .Trong trng hp ny, x2 chia ht cho 5, v vy x2 cng chia ht cho 25. iu chng t b 2. Ly x = 5y, cho y > 1 v (y 1)(y + 1) = 2a .5b2 . Ta c y 1 v y + 1 l cc s chn, v

1 p = 2 .(y 1) v q = 1 .(y +1) l cc s nguyn dng m p.q = 2a2 .5b2 . 2 m V vy p v q bng 2 v 5n vi m, n l cc s nguyn. Ta xt hai trng

2(mod3). V vy n l s t nhin v 5n 5(mod8). Ta c 2m (5m 1) 4(mod8),t suy ra m = 2. T ta c N = 2000. 2) 2m 5n = 1. Ta c 2m = 5n + 1 2(mod4) t suy ra m = 1 v

hp nh nh sau: 1) 5n 2m = 1; v 5n , 2m = 0(mod3),ta c : 2m 1(mod3) v 5n

n = 0. T ta c N = 200. V vy, tt c cc s tho mn bi ton l

l c ca 2. Nhng khng c hai s l c ca 2 m hai s li hn km nhau 10, do khng c s tho mn trng hp ny.

hai s t nhin hn km nhau 10. Ta xt hai trng hp: Trng hp 1: b = 0. Khi vi 2a = (x + 5)(x 5) ta c x + 5 v x 5

s m nguyn. Vi s nguyn x > 5,ta c x2 = N + 25,iu ny tng ng vi (x + 5)(x 5) = N. V vy N c biu din bng tch ca

.c

Li gii: Ta c th biu din N di dng l 2a .5b ,vi a v b l cc

thi olympic o - Balan N = 200 ; 2000.

27

4.18.Tm cc s nguyn n 5,sao cho ta c th s dng mu t nh a gic n-nh bi 6 mu m 5 nh lin tip nhau c mu khc nhau ? Li gii: Ta gi cc mu l a, b, c, d, e, f . Biu th S1 bi dy a, b, c, d, e v S2 bi dy a, b, c, d, e, f . Nu n > 0, ta c th biu din di dng 5x + 6y vi x; y 0, khi n tho mn cc iu kin sau: x lin vi

dy S1 ko theo y lin vi dy S2 trn hnh a gic. Ta c: y c th bng 0,1,2,3 hoc 4. Khi n c th bng 1 trong cc s c dng 5x ;

5x + 6; 5x + 12; 5x + 18 hoc 5x + 24. Tt c cc s ln hn 4 khng tho mn cc dng trn l 7,8,9,13,14,19. Xt tt c cc s n, tr cc trng hp bng 7,8,9,13,14,19. Ta c tn ti s k sao cho 6k < n < 6(k + 1). Theo nh l Pigeonhole, c t nht k + 1 nh ca n cnh c mu ging nhau. Gia 2 hoc 3 nh c t nht 4 nh khc na, bi v 5 nh lin tip c mu khc nhau. V vy, c t nht 5k + 5 nh, v n 5k + 5. Do vy, ngoi tr n = 7, 8, 9, 13, 14, 19,

Li gii: Ta c th chia khng gian thnh cc khng gian 3 chiu, m trng tm ca cc hnh lp phng n v trng vi cc li im. Xt cc im c to (x, y, z). Xt x + 2y + 3z. Ta gi 2 li im gn k nhau nu v ch nu chng khc nhau duy nht 1 to . Ta xem xt khng gian 3 chiu cha 7 hnh lp phng n v m trng tm ca chng ng vi cc s t 0 n 6. Do vy, khng gian c th chia thnh cc khng gian 3 chiu tho mn bi ton. 4.20.Trong mt phng, cho tam gic Ao Bo Co . Xt tt c cc tam gic ABC tho mn cc iu kin sau:(i)Co ; Ao vBo ln lt nm trn AB, BC v

trong.

phn chng(trong khng gian 3 chiu) khng gian c th chia thnh 3 chiu sao cho khng c 2 trong chng c th chia cc im nm bn

lp phng n v khc, to thnh 7 hnh lp phng n v vi cc trng tm l (0, 0, 0), (1, 0, 0), (0, 1, 0), (0, 0, 1). Chng minh hoc

4.19.Trong khng gian 3 chiu, cho hnh lp phng n v cng vi 6 hnh

cc trng hp cn li u tho mn bi ton. Vy tt c cc s n 5 tr 7,8,9,13,14,19 tho mn bi ton.

.c

28

Nguyn Hu in, HKHTN H Ni CA.(ii)ABC = Ao Bo Co , BCA = Bo Co Ao v CAB = Co Ao Bo . Tm qu tch tm ng trn ngoi tip tam gic ABC. Li gii: Ta c t nht mt tam gic ABC tn ti, v d ta xt tam gic Ao Bo Co gia tam gic ABC. Gi tam gic Ao Bo Co c ng trn ngoi tip w, tm O. Gi s tam gic ABC tho mn cc iu kin ca bi ton. V A v Ao nm 2 pha ca cnh BC, v Co Ao Bo = CAB. V vy, ng trn ngoi tip tm O1 ca tam gic Bo Co A phi i xng vi O qua Bo Co . Tng t ta xt vi ng trn ngoi tip tm O2 v O3 ca tam gic Ao Co B v Ao Bo C. Ta c t gic OBo O1 Co l hnh thoi sao cho O1 = bo + co.Tng t,ta c:O2 = ao + co

nht mt im l trc tm ca tam gic Ao Bo Co . 4.21.Cho 27 im phn bit trn mt phng khng c 3 im no thng hng. 4 im trong chng lp thnh cc nh ca hnh vung n v; 23 nh cn li nm trong hnh vung trn. Chng minh rng tn ti 3 im

.c

1 c honh 1 (a + b) + co . V vy, ly im H c honh 2 (a + b) th 2 c H nm trn AB. Ch rng h = m + (m co ) = ao + bo + co . Ta

c H nm trn OG vi G l trng tm Ao Bo Co , m OH = 3OG. Do vy H l trc tm H ca tam gic Ao Bo Co . Qu tch cn tm l duy

Li gii: Ta chng minh bng quy np. Ly n 1 im nm trong hnh vung (khng k 3 ng thng), hnh vung c th chia thnh 2n + 2 tam gic m cc nh ca cc tam gic cng l 1 trong n im hoc cc tam gic nm bn trong hnh vung. Vi n = 1, vi hnh vung l hnh li nn ta c th chia hnh vung thnh 4 tam gic bi cc ng nm trong hnh vung, do cc tam gic u nm trong hnh vung. Gi s bi ton ng vi n = k 1. Ta s chng minh bi ton ng vi n = k + 1. Tht vy, vi n = k, ta c th chia hnh vung thnh 2k + 2 tam

ring bit X; Y ; Z sao cho [XY Z]

Ly M l trung im ca O1 O2 . Xt A(a, 0), Co(co , 0), B(b, 0). Ta c 1 honh ca im O1 v O2 l 1 (a + co ) v 2 (b + co ). V vy im M 2

1 . 48

thi olympic o - Balan

29

gic m cc nh ca cc tam gic cng l mt trong k im hoc cc tam gic nm bn trong hnh vung. Vi n = k + 1, ta xt thm im P1 . V khng c 3 im no thng hng nn P nm trong s 2n+2 tam gic, v d ABC. Nh vy ABC chia thnh cc tam gic nh l AP B; BP CvCP A. Nh vy hnh vung c chia thnh 2(n + 1) + 2 = 2n + 4 tam gic.Vy bi ton c chng minh bng quy np. Trng hp c bit n = 23, hnh vung c th chia thnh 48 tam gic vi tng din tch bng 1. Vy 1 trong cc tam gic trn c din tch ti a l
1 48

tho mn yu cu bi ton.

.c

Chng 5 thi olympic a Trung Hi


5.22.Cho n s dng a1 , a2 , ..., an v tp hp {1 , 2 , ..., n }, vi mi i {1, 1}. Chng minh rng tn ti mt php hon v (b1 , b2 , ..., bn ) ca

ca chng xen k nhau iii) Vi 1 i n, du hiu ca cc s trong x1 , x2 , ..., xn c gi tr tuyt i ln nht bng i . lm c nh vy, ta ch n gin xy dng x1 , x2 , ..., xn trong trt t, ti mi bc chn xi0 theo quy tc du hiu ng vi thuc tnh (ii), vi i = i0 v t hoc |xi0 | > max {|x1 | , |x2 | , ..., |xi0 1 |} hoc |xi0 | < min {|x1 | , |x2 | , ..., |xi0 1 |}. V vy thuc tnh (iii) ng vi i = i0 .

Chn bi v i sao cho bj1 < bj2 |xj1 | < |xj2 | v j xj > 0, j, j1 , j2 . Gi thit rng 1 i n. Sp xp b1 , b2 , ..., bi theo bc tng dn thu c bk1 , bk2 , ..., bki . Bng cch xy dng, dy cc du hiu k1 , k2 , ..., ki xen k nhau, v ki = i . V vy :

i) Vi 1 i n, x1 , x2 , ..., xn c gi tr tuyt i khc nhau ii) Khi c sp xp theo th t gi tr tuyt i tng dn, du hiu

Li gii: Ta xy dng mt dy cc s khc khng x1 , x2 , ..., xn vi nhng thuc tnh sau y:

a1 , a2 , ..., an v tp hp {1 , 2 , ..., n } vi mi i {1, 1} sao cho du hiu ca i j bj bng vi du hiu ca i , 1 i n. j=1

.c

thi olympic a Trung Hi


i

31

j=1

j bj = i bki bki1 + bki2 bki3 + ... bk1 .

Biu thc trong ngoc l tng ca k/2 biu thc dng ca cc hnh thc bkj+1 bkj v c th cng thm vo mt s hng bk1 . V vy,
i j=1 j bj

c cng du hiu vi i vi mi i, pcm.

5.23.Cho t gic li ABCD. Dng ra pha ngoi cc cnh ca t gic cc tam gic u W AB, XBC, Y CD, ZDA vi S1 , S2 , S3 , S4 ln lt l trng tm. Chng minh rng S1 S3 S2 S4 AC = BD. Li gii: Chn O l mt im bt k. Gi a, b, c, d theo th t biu th cho cc vect t O n A, B, C, v D. M1 , M2 , M3 , M4 theo th t l trung im ca AB, BC, CD, DA, v si biu th cho cc vect t Mi ti Si vi i = 1, 2, 3, 4. C 2 vect x v y, cho (x, y) l gc gia chng thun theo chiu kim

(x) (y) = | (x)| | (y)| ( (x) , (y))

bin i m quay bt k vect no /2 ngc chiu kim ng h v 1 nhn ln ca n ln bi 23 . Khi :

1 xy 12 Tch v hng ca vect S3 S1 vi vect S4 S2 bng : (b d) + (a c) + s1 s3 2

|x| 2 3

.c

ng h. (Tt c cc gc u mod 2). Khng mt tnh tng qut, gi s rng ABCD l nh hng theo chiu kim ng h, v cho l php

|y| 2 3

(b d) (a c) + s2 s4 , 2

v bng tng ca 4 biu thc sau y : |b d|2 |a c|2 4 , s1 s3 s2 s4 ,

(x, y)

32

Nguyn Hu in, HKHTN H Ni 1 s (b a) s3 (c d) + (b c) s2 (a d) s4 , 2 1 1 s (c d) s3 (b a) + (a d) s2 (b c) s4 . 2 1 Biu thc u tin bng


1 4

th 3 u bng 0: MS1 AB ng rng s1 (b a) = 0, v. v. . . Trong biu thc th 2, ta thy : s1 s3 = ((b a) (d c)) = ((c a) + (b d)) v

(BD2 AC 2 ). Bn s hng trong biu thc

Tng ca cc argument ca 2 csin l : + ( (a b, c d) + (c d, a b)) = 3, Vi ng rng gi tr ca mi csin l ph nh ca ci khc. Do , s1 v s3 trong biu thc 4 trit tiu ln nhau bn ngoi. Tng t, cng lm nh vy vi s2 v s4 . 1 V th, ton b tch v hng bng 1 12 (BD2 AC 2 ). V 4 S1 S3 S2 S4 khi v ch khi tch v hng ny bng 0, S1 S3 S2 S4 BD = AC, pcm.

s3 (b a) = s3 (a b) = CD 2 3 (AB) cos (/2 + (c d, a b)) .

trong khi

s1 (c d) = AB 2 3 (CD) cos (/2 + (a b, c d))

1 hay 12 (CA2 BD2 ). i vi biu thc th t,

.c

Do , tch v hng ca chng bng 1/12 ca : ((c a) + (b d)) ((c a) (b d)) = |c a|2 + |b d|2 ,

s2 s4 = ((c b) (a d)) = ((c a) (b d))

thi olympic a Trung Hi

33

5.24.P, Q, R, S theo th t l trung im ca cc cnh BC, CD, DA, AB ca t gic li ABCD. Chng minh rng : 4 AP 2 + BQ2 + CR2 + DS 2 5 AB 2 + BC 2 + CD2 + DA2 Li gii: Ta bit cng thc : XM l trung tuyn ca tam gic 1 XY Z, th XM 2 = 1 XY 2 + 1 XZ 2 4 Y Z 2 . Ta thay (X, Y, Z, M) bng 2 2

(A, B, C, P ), (B, C, D, Q), (C, D, A, R) v (D, A, B, S) vo trong cng thc ny v cng 4 cng thc li vi nhau thu c 1 cng thc th 5. Nhn c 2 v ca cng thc th 5 vi 4, ta tm thy v tri ca bt ng thc s bng

vi du = xy ra khi v ch khi a + c = b + d(hay khi v ch khi ABCD l hnh bnh hnh). iu phi chng minh.

= |a + c b d|2 0

|a|2 + |b|2 + |c|2 + |d|2 + 2 (a b + b c + c d + d a a c b d)

biu th vect t O ti X. Ta c th khai trin ton b cc s hng trong AB 2 + BC 2 + CD2 + DA2 AC 2 BD 2 , v d vit AB 2 = |a b|2 = |a|2 2a b + |b|2 , thy rng biu thc ny bng

.c

y l cng thc bt ng thc hnh bnh hnh. chng minh n, gi O l mt im bt k trong mt phng v vi mi im X, gi x

Do , ta ch cn chng minh AC 2 + BD 2 AB 2 + BC 2 + CD2 + DA2 .

AB 2 + BC 2 + CD2 + DA2 + 4 AC 2 + BD2

Chng 6 thi olympic Petecbua


m
6.25.Cho AA1 , BB1 , CC1 l ng cao ca tam gic nhn ABC. Hai im A2 v C2 nm trn ng thng A1 C1 sao cho ng thng CC1 chia i on thng A2 B1 v ng thng AA1 chia i on thng C2 B1 . ng thng A2 B1 v AA1 gp nhau ti im K, v ng thng C2 B1 v CC1 gp nhau ti im L. Chng minh rng ng thng KL v AC song song vi nhau. Li gii: Gi im K1 v L1 l im gia ca C2 B1 v A2 B1 do K1 nm trn ng AA1 v L1 nm trn ng thng CC1 . D dng chng minh c ng cao AA1 ca tam gic ABC l ng phn gic ca

A1 K1 cng l ng cao ca tam gic A1 C2 B1 do A1 K1 vung gc vi B1 C2 tng t C1 L1 vung gc vi A2 B1 . T suy ra ng thng KK1 v LL1 l ng cao ca tam gic KLB1 a n chng ng quy ti vi ng cao l t B1 trong tam gic ny. T ng thng KK1 v LL1 gp nhau ti trc tm H ca tam gic ABC, l phi i qua im B1 v H. Do l vung gc vi AC. Bi v l l ng cao trong tam gic KLB1 i qua B1 , n cng vung gc vi KL. Chng ta kt lun rng KL//AC, nh bi. 6.26.Mt trm im c chn trong mt phng to . Hy ch ra rng ti

tam gic A1 B1 C1 t cho thy rng A1 K1 va l ng phn gic va l ng trung bnh ca tam gic A1 C2 B1 , v vy A1 C2 = A1 B1 v

.c

thi olympic Petecbua

35

a 2025 = 452 cc hnh ch nht vi cc nh trong s cc im ny c cnh song song vi cc trc. Li gii: Li gii th nht: Gi O l mt trong 100 im, v gi hnh ch nht c gi tr nu cc nh ca n l O v 03 im khc c la chn. Chng ta khng nh rng c t nht 81 hnh ch nht c gi tr. V qua O ng thng I1 v I2 song song vi trc to , ti m c chn l cc im nm trn I2 {O}. Cho im c nh bt k chn

P khng nm trn I1 hoc I2 , ti a mt hnh ch nht c gi tr c P nh l mt nh, hn th na, tt c cc hnh ch nht c gi tr ca dng ny cho im P no . Do c 99 m n nh im P, c ti a nhiu tam gic c gi tr kiu nh vy. Nu m + n > 17, chng ta lm

Li gii th hai: Gi mt hnh ch nht thch hp nu 4 nh ca n c la chn l cc im. V tt c cc ng thng ng I1 ....., In i qua t nht mt trong nhng im la chn. Gi thit rng Ii cha xi l cc im la chn, s := n xi = 100. S tam gic thch hp i=1 2 2 vi cc cnh bn trn ng thng ith v j th l ti a min{Cxi , Cxj }.

2xyxxy 2 2 i vi cc s nguyn Quan st thy rng min{Cxi , Cxj } 4 dng l x v y, bi v nu x y th cnh bn tay tri ti a l x(x1) 2 1 [x(y 1) + y(x 1)]. Do 4 x +x 2xi xj ) 1 i j n i4 j 1 i j n 4 1 ( n x2 + 100n) i=1 i 4

n c la chn cc im l 4 ln. V vy, c t nht 8100/4 = 2025 hnh ch nht, nh bi yu cu.

i O trn tt c 100 im, m s hnh ch nht cho mi O. Tng cng s im t nht l 8100, v chng ta tam gic bt k m cc nh ca

l 2525

p dng bt ng thc bnh phng trung bnh nghim v bt ng thc

Chng ta kt lun rng trong bt k trng hp no, c t nht 81 hnh ch nht m gi tr ca n l O v c 03 im khc c la chn. Bin

vy s tam gic thch hp ti a 1 = 1 (s2 n x2 ) 4 (n 1)s = i=1 i 4

.c

P v Q nh l cc nh; hn na tt c cc hnh ch nht c gi tr ca dng ny i vi cp (P, Q) nh vy. V c mn < m(17 m) < 8.9 = 72 cc cp nh vy, c t nht 72 < 8 hnh ch nht c gi tr.

c. Mt khc, cho mt cp (P, Q) l cc im chn, y P thuc I1 {O} v Q thuc I2 {O}, t nht c mt hnh ch nht gi tr c

36

Nguyn Hu in, HKHTN H Ni trung bnh hnh s hc, ta thy rng biu thc t cui cng ny ti a 2 l bng 2525 1 ( s +100n) = 252525( 100 +n) 252525.2 ( 100 n) = 4 n n n 2025 Nh bi yu cu

6.27.a, Tm tt c cc cp ca cc s nguyn khc bit a, b l lu tha ca mt s nguyn t.

b2 +a a2 +b
2

v b2 + a

b b, Cho a v b l cc s nguyn dng ln hn 1tho mn a2+a1 . Chng +b1 minh rng b2 + a 1 c t nht hai tha s nguyn t khc nhau.

Li gii: a, Chng ta chng minh rng ch c cp nh vy l (a, b) = (5, 2). Nu

a+1 a+1 (b = 1), th a2 +1 dn n a(a+1)(a2 +1) = a 1. Do vy a=1 khng a ra li gii khi a, b c a ra yu cu khc bit. V vy tng b>1, v

b2 b + 1 chia ht cho 3 v chia ht cho 3m1 nhng b2 < b2 + a = 3m dn n rng b+1 nhiu hn 3m/2 + 1 < 3m1 v khng th chia ht cho 3m1 . Chng ta kt lun rng 3m1 chia ht cho b2 b + 1 v do vy 9

chia ht cho 4.(b2 b + 1) = (2b 1).2 + 3. iu ny khng th v khng c bnh phng ng dng vi 6 modun 9. V vy khng c li gii ngoi (5,2) b, Gi thit, v s mu thun, rng b2 1+a l lu tha ca mt nguyn

t. V (b2 1)2 a2 chia ht cho b2 1 + a, khi a2 + b 1 l gi thit tng ca chng l (b2 1)2 + b + 1 = b(b 1)(b2 + b 1). Quan st thy rng b, b-1, v b2 + b 1 = b(b + 1) 1 = (b + 2)(b 1) + 1 l nguyn t cng nhau tng cp. V vy, mt trong b, b-1, v b2 + b 1

chng ta kt lun rng p chia ht cho c b+1 v b2 b + 1, th n cng phi chia ht cho (b2 b + 1) (b + 1)(b 2) dn n p=3.

Khng c li gii cho m=1. Nu m=2, th chng ta c b2 + a = 9, li gi d dng l (a,b)=(5,2). Mt khc, gi s rng m 3, mt trong b+1 v

b 2 2 chng ta c (b+1)(b+a 2 b+1) . Mi b+1 v b b + 1 t hn b + a, v vy c hai u khng chia ht cho b2 + a. V b2 + a = pm l lu tha ca p,

.c

chia cho b(b3 + 1) nhng gcd(b, b3 + 1)v b2 + a l lu tha ca mt nguyn t, v vy mt trong s b hoc b3 + 1 c th chia ht cho b2 + a. Trng hp u r rng l khng th c, trong trng hp th 2,

vit b2 + a = pm khi p l nguyn t v m 1. Quan st thy b(b3 + 1) (b2 )2 + b a2 + b 0(mod(b2 + a)), b2 + a

thi olympic Petecbua

37

a 0

phi chia ht cho lu tha nguyn t b2 1 + a. V b v b-1 nh hn b2 + a 1 chng ta phi c rng b2 + a 1 chia cho b2 + b 1 v v vy b v a = b l gi thit a<b. Mt khc, v b 2+a1 chng ta phi c a +b1 (a2 + b 1) (b2 + a 1) = (a b)(a + b 1). V vy a b nh l
2

mt mu thun .

6.28.Trong mt t nc c 2000sn bay, ban u khng c mt chuyn bay no ca hng hng khng. Hai hng hng khng ln lt gii thiu nhng chuyn bay thng kh hi mi (gia hai thnh ph bt k, ch c mt chuyn bay thng c gii thiu). C quan vn ti mun t c mc tiu l nu sn bay no b ng ca th mi ngi vn c th du lch gia hai sn bay bt k khc, c th bng chuyn ti. Hng hng khng to ra mc ch t c s b thua l. Hng hng khng no s chin thng cuc chi mt cch hon ho? Li gii: Cng ty hng khng th hai chin thng. Xem xt tnh hung khi

khng c thnh ph trong G1 c th gia nhp vi bt k thnh ph no ca G2. Do nu h s k thnh ph trong G1, th c k1 chuyn bay 2 gia hai thnh ph trong G1, (1999k)(1998k) chuyn bay gia hai thnh 2 ph trong G2, v 1999 chuyn bay gia A v thnh ph khc. Nh vy tng s chuyn bay l k(k-1999)+1999000 l u nhau. Ni c th, cha bao gi ti lt ca hng hng khng th hai b sung thm mt chuyn bay mi. Do vy, cng ty hng khng th hai lun trnh c s thua l. 6.29.Chng ta c phng trnh bc hai a thc li, tt c c cng bit s. Tng ca hai a thc bt k c nghim thc ring bit. Ch ra rng tng

trong G1 hoc trong G2 th phi gia nhp, bi v nu khng b sung chuyn bay gia hai thnh ph th mc tiu a ra khng t c. Tng t nh vy, tt c cc thnh ph phi gia nhp vo A, nhng

sn bay A bt k khi ng ca chia cc thnh ph ra lm hai nhm khng lin lc c l G1 v G2. Khi hai thnh ph bt k nm

m mc tiu khng t c, nhng vic b sung thm chuyn bay n bt k t c mc ch nu ra. Do mc tiu khng t c, c

.c

38

Nguyn Hu in, HKHTN H Ni s ca tt c cc a thc cng u c nghm thc ring bit. Li gii: Bit s chung phi l dng, bi v nu khng mi a thc ch c gi tr dng, v vy tng ca hai a thc bt kk s khng c nghim thc. Cho bit s chung l 4D, mi a thc l dng (x c)2 D i c bt k. mi a thc, c xem l khong cch trong a thc c cha gi tr m, khong cch c di l 2 D . Nu hai trong s khong cch ny (c1 D, c1 + D v (c2 D, c2 + D khng giao nhau, th |c2 c1| > D v 1 (c1 + c2) khng nm trong mt trong hai khong 2 1 cch. Do vy c hai a thc u khng m ti 2 (c1 + c2) nhng im ny khi tng s a thc p t c gi tr nh nht ca n - mu thun vi gi thit l c p nghim thc phn bit Do vy, hai khong cch giao nhau bt k. Chn mt khong cch (c D, c + D c l cc

Li gii: Nu 2.3 hnh ch nht bt k c bao ph bi hai gc, th chng ta c th d chuyn tt c cc gc ngoi tr hai gc . Do vy, chng ta c th tng kt rng khng c hnh ch nht nh vy tn ti. Chng ta xy dng mt th trc tip m cc nh ca n l cc gc, nh sau: i vi mi gc, v 2.2 hnh vung c cha gc , v thm mt ng ni t gc ny ti gc kia bao ph s d ca 2.2 hnh vung. Nu mt gc khng c im ni v phi n, chng ta c th dch chuyn gc , v vy chng ta c th tng kt rng, khng c gc nh vy tn ti. Do , mi ng ni ca th nm trong chu trnh no . Nu c hn mt chu trnh, th chng ta c th dch chuyn tt c cc gc

i vi gc bt k, cha 2.2 hnh vung bao ph ton b cc gc. Chng minh rng mt gc c th dch chuyn gia mt v 110 ca cc gc tnh cht c bo v.

6.30.Trn mt bn c am v hn t 111 gc khng chng nhau. Cc hnh c ch L c t lm 3 n v hnh vung. Tp hp c tnh cht sau:

thuc bt k. Ti im ny, tng s ca tt c cc a thc cha gi tr m a, do vy tng ny phi c nghim thc ring bit.

.c

tiu. V tt c cc khong cch giao nhau bt k khc iu ny, chng ta thy rng tt c cc khong cch u cha (c + D thuc i vi

thi olympic Petecbua

39

ngoi tr cc gc nm trong chu trnh ca chiu di di cc tiu, v yu cu c tnh c tn ti. V vy, n tho mn ch ra rng khng th tn ti mt chu trnh n bao gm tt c 111 nh. Theo im gia ca mt gc chng ta hng theo im ti im gia ca 2.3 hnh vung cha gc . Nh li rng chng ta tng kt khng c hai gc bao ph 2.3 hnh vung, mt iu d dng kim tra l nu c mt im ni t mt gc ti mt gc khc, th im gia ca nhng gc ny khc 1 trong c hai to ca chng l x v y. Do , trong chu trnh bt k, to x ca cc nh nn trong s bin i ca chu trnh, do s ca cc nh nm trong chu trnh l chn. Do vy, khng c mt chu trnh cha tt c 111nh, nh bi a ra.

.c

Chng 7 thi olympic Anh


7.31.Cho hai ng trn ct nhau (C1 ) v (C2 )c mt tip tuyn chung tip xc (C1 ) ti P, tip xc (C2 ) ti Q. hai ng trn ny ct nhau ti M

nht ca x2 + 4xy + 4y 2 + 2z 2 . Li gii: p dng bt ng thc trung bnh hai ln ta c: x2 + 4xy + 4y 2 + 2z 2 = (x2 + 4y 2)2 + 4xy + 2z 2 2 x2 4y 2 + 4xy + 2z 2 = 4xy + 4xy + 2z 2 3 3 4xy4xy2z 2 = 3 3 32(xyz)2 = 96. Du "=" xy ra khi v ch khi x2 = 4y 2 v 4xy = 2z 2 , tc l (x,y,z)=(4,2,4)

7.33.a, Tm mt tp A ca 10 s thc nguyn m khng c 6 phn t phn bit no ca A c tng chia ht cho 6 b, C th tm c mt tp nu 10 c thay bi 7 hay khng?

7.32.Cho x, y, z l nhng s thc dng tho mn xyz = 32. Tm gi tr nh

1 MN(XQsinMXQ) 2

trn vy XP 2 = XQ2 hay XP = XQ Cng v P XM + MXQ = ta c sinP XM = sinMXQ cho nn [MNP ] = 1 MN(XP sinP XM ) = 2 = MNQ. ta c iu phi chng minh.

Li gii: Gi X l giao im ca MN v PQ. V MN l trc ng phng ca (C1 ) v (C2 ), X c cng mi lin quan ny vi hai ng

.c

v N. Chng minh rng tam gic MNP v tam gic MNQ c cng din tch.

thi olympic Anh

41

Li gii: a, Mt v d ca tp A l A = {6j + k|1 j 5, 1 k 2} Trong bt k 6 phn t trong tp con ca A nu c t s ng d 1 modun 6 th t {1, 2, ..., 5}. nhng phn t khc trong tp con l ng d 0 mdun 6. V vy tng ca nhng phn t trong tp con l ng d t = 0 (moun6) b, Khng th cho bt k tp no c 7 s thc nguyn, chng ta c 6 phn t phn bit ca tp ny c tng chia ht cho 6. Bi v c hn 2 s nguyn trong tp ny, chng ta c th chn hn 2 m l chn. Cng cch ny lm nh th ta c th tm thy 5 tp con c 2 phn t ri rc ci m c tng l ng d ca 0, 2, 4 moun 6. Nu tt c tng xut hin, 6 phn t trong tp con tng ng c tng ng d 0 + 2 + 4 = 6 (moun 6). Cch khc, ch tng xut hin . Bi Pizenhle principle, 3 tp con s c tng nh nhau. Do nhng phn t trong 3 cp s c tng chia ht cho 6.

.c

Ti liu tham kho


[1] Titu Andreescu, Zuming Feng, and George Lee, Jr. Mathematical Olympiads 20002001, Problems and Solutions From Around the World, The Mathematical Association of America, 2002.

[6] Nguyn Hu in, Sng to trong gii ton ph thng, NXBGD, 2002. [7] Nguyn Hu in, a thc v ng dng, NXBGD, 2003. [8] Nguyn Hu in, Gii phng trnh v nh nghim nguyn, NXBHQG, 2004. [9] Nguyn Hu in, Gii ton bng phng php i lng bt bin, NXBGD, 2004.

hc, NXBKHKT, 2001.

[5] Nguyn Hu in, Nhng phng php gii bi ton cc tr trong hnh

thng, NXBGD, 2001.

.c

[4] Nguyn Hu in, Nhng phng php in hnh trong gii ton ph

[3] Nguyn Hu in, Phng php Quy np ton hc, NXBGD, 2000.

[2] Nguyn Hu in, Phng php irichle v ng dng, NXBKHKT, 1999.

Nguyn Hu in

OLYMPIC TON NM 1997-1998 (Tp 4)


t
NH XUT BN GIO DC

.c

51 THI V LI GII

v n m a t h .c o m

Li ni u
th gi lnh lamdethi.sty ti bin son mt s ton thi Olympic, m A cc hc tr ca ti lm bi tp khi hc tp L TEX. ph v cc bn ham hc ton ti thu thp v gom li thnh cc sch in t, cc bn c th tham kho. Mi tp ti s gom khong 51 bi vi li gii. chnh xc vy mong bn c t ngm ngh v tm hiu ly. Nhng y l ngun ti liu ting Vit v ch ny, ti c xem qua v ngi dch l chuyn v ngnh Ton ph thng. Bn c th tham kho li trong [1]. Rt nhiu on v mi hc TeX nn cu trc v b tr cn xu, ti khng c thi gian sa li, mong cc bn thng cm.

H Ni, ngy 2 thng 1 nm 2010 Nguyn Hu in

51 89/176-05 GD-05

.c

Rt nhiu bi ton dch khng c chun, nhiu im khng hon ton

M s: 8I092M5

Mc lc
Li ni u . . . . . . . . . . . . . . . . . . . . . . . . . . . . . . . . . . . . . . . . . . . . Mc lc . . . . . . . . . . . . . . . . . . . . . . . . . . . . . . . . . . . . . . . . . . . . . . . 3 4 5 9 13 17 21 24 28 31 37

Chng 8. thi olympic c . . . . . . . . . . . . . . . . . . . . . . . . . . . . Chng 9. thi olympic Irland . . . . . . . . . . . . . . . . . . . . . . . . . .

Chng 7. thi olympic Php . . . . . . . . . . . . . . . . . . . . . . . . . . .

Chng 6. thi olympic Czech v Slovak Repubulick . . . .

Chng 5. thi olympic Colombia . . . . . . . . . . . . . . . . . . . . . .

Chng 4. thi olympic Chine . . . . . . . . . . . . . . . . . . . . . . . . . .

.c

Chng 3. thi olympic Canada . . . . . . . . . . . . . . . . . . . . . . . .

Chng 2. thi olympic Bungari . . . . . . . . . . . . . . . . . . . . . . . . .

Chng 1. thi olympic Austria. . . . . . . . . . . . . . . . . . . . . . . . . .

Chng 1 thi olympic Austria


m o
xy(x y) + (y x) 0 0 1.1. Gii h phng trnh vi x, y l s thc

xy(y x) + 6(x y) + (x + y)(x y) = (x y)(xy + 6 + (x + y) xy + 1) = (x y)(x + y 2xy + 7) =

trnh ny l tng ng vi phng trnh sau(c suy ra t cch sp xp

l vi x = y = 2 hoc x = y = 3 (tm c bng cch gii phng trnh (1)) vi php th x = y By gi, ta xt trng hp x = y s c gii x + y 2xy + 7 = 0. Phng

Do vy, hoc xy = 0 hoc x+ y 2xy + 7 = 0. Cch duy nht c xy = 0

Chng ta li tr hai phng trnh cho nhau, tr phng trnh th hai cho phng trnh th nht v nhm li, ta c:

bnh phng ca mt hiu ta c phng trnh sau 5 5 1 (x 2 )2 + (y 2 )2 = 2

Li gii: Ta cng hai phng trnh trn cho nhau. Sau khi rt gn v a v

(y 1)(x2 + 6) = x(y 2 + 1)

.c

(x 1)(y 2 + 6) = y(x2 + 1),

6 li cc s hng v tha s) 1 (x 1 )(y 2 ) = 15 2 4

Nguyn Hu in, HKHTN H Ni

Gi s, chng ta c th gii phng trnh (1) v (2) mt cch ng thi. t a = x 5 v b = y 5 . Do , phng trnh (1)) tng ng vi 2 2
1 a2 + b2 = 2 v phng trnh (2) tng ng vi: 15 1 1 (a + 2)(b + 2) = ab + 2(a + b) = 2ab + 4(a + b) = 4 4 2 Cng phng trnh (4) v (3) chng ta thy:

(a + b)2 + 4(a + b) = 0 a + b = 0, 4 Ly phng trnh (4) tr (3) ta thy: (a b)2 4(a + b) = 1

By gi, (vi) w = x2 + yx2 + z 2 . V x2 + y 2 61 v z 2 0, x2 + y 2 + z 2 61. Nhng w 44. Do , chng ta c mu thun v gi thit ca chng ta l khng ng. Vy, nu ak = 1997 th k 3. 1.3. Cho k l mt s nguyn dng. Dy a n c xc nh bi a 1 = 1 v an l n s nguyn dng ln hn an1 l ng d n modulo k. Tm an trong dy trn.

c t nht mt s trong 4 s ak1 , ak2 , ak3 vak4 phi tn ti. t w = ak1 , x = ak2 , y = ak3 v z = ak4 .By gi, iu kin ca chng ta l: 1997 = w 2 + x2 + y 2. Do , w 1997 < 45 v v w l mt s nguyn dng nn w 44. Nhng do x2 + y 2 1997 4462 = 61.

Li gii: Chng ta gii trc tip: Gi s vi k > 3, ak = 1997. Khi ,

1.2. Cho dy s nguyn dng tha mn an = a2 + a2 + a2 vi n 3. n1 n2 n3 Chng minh rng nu ak = 1997 th k 3.

(3, 2).

1 1 ta c th tm c tt c cc cp c th t (a, b). Chng l ( 1 , 2 ) v ( 1 , 2 ) 2 2 . Do vy, cc nghim (x, y)ca h phng trnh cho l(2, 2),(3, 3),(2, 3) v

.c

(a b)2 = 1 a b = 1 V t phng trnh (5) chng ta c a+b = 0,v cng vi phng trnh (7)by gi

Nhng by gi chng ta thy rng ,nu a + b = 4 th phng trnh (6) s b sai; Do , a + b = 0. Th a + b = 0 vo phng trnh (6) chng ta thu c:

thi olympic Austria

u tin ln hn an1 m l ng d n modulo k phi l an1 + 1. n - th s nguyn dng ln hn an1 l ng d n modul k l n gin (n - 1)k hn s nguyn dng u tin ln hn an1 m tha mn iu kin . Do vy, an = an1 + 1 + (n 1)k. Li gii bng php quy ny a ra cu tr li ca bi ton trn. 1.4. Cho hnh bnh hnh ABCD, mt ng trn ni tip trong gc BAD v nm hon ton trong hnh bnh hnh. Tng t, mt ng trn ni tip trong gc BCD nm hon ton trong hnh bnh hnh sao cho 2 ng trn tip xc. Hy tm qu tch cc tip im ca 2 ng trn khi chng thay i. Li gii: Gi s K1 l ng trn ln nht ni tip trong gc BAD sao cho n nm hon ton trong hnh bnh hnh. N ct ng thng AC ti 2 im v gi s im xa A hn l P1 . Tng t Gi s K2 l ng trn ln nht

Li gii: Chng ta c an = n(2+(n1)k) . Nu k = 2 th an = n2 . Trc tin, 2 ch rng a1 1(modk). Do , vi tt c n, an n(modk), v s nguyn

ngang. Gi s T1 l chn ng vung gc h t I1 ti AB v T2 l chn ng vung gc h t I2 ti CD. Ch rng: I2 T2 I1 T1 = sin I1 AB = sin I2 CD = AI1 CI2 Nhng I1 X = I1 T1 v I2 X = I2 T2 . Do vy I1 X I2 X = AI1 CT2 V th tam gic CI2 X v tam gic AI1 X l ng dng v cc gc vung I1 XA, I2 XC l bng nhau. V cc gc ny bng nhau nn cc im A, X v C

gic ca cc gc. Mt khc v AI1 v CI2 l cc ng phn gic ca cc gc i hnh bnh hnhneen chng song sonh vi nhau. Do vy I1 I2 l ng nm

BCD. Gi s X l im tip xc ca 2 ng trn. V cc ng trn tm I1 v I2 l ni tip trong cc gc nn cc tm ny phi nm trn cc ng phn

tch l giao ca 2 on AP1 v AP2 . Chng ta bt u chng minh im tip xc phi nm trn ng AC. Gi s I1 l tm ng trn ni tip gc BAD v I2 l tm ng trn ni tip gc

.c

ni tip trong gc BCD sao cho n nm hon ton trong hnh bnh hnh. N ct ng thng AC ti 2 im v gi s im xa C hn l P2 . Khi , qu

Nguyn Hu in, HKHTN H Ni

phi cng tuyn. Do vy, im tip xc X phi nm trn ng cho AC ( l iu phi chng minh). Nh vy, chng ta bit rng X s lun nm trn AC, by gi ta s chng minh bt k im no thuc qu tch u l im tip xc. Cho X bt k nm trn qu tch , gi s I1 l ng trn b hn ng trn qua X, ni tip trong gc BAD. N s nm hon ton bn trong hnh bnh hnh bi v X l im gia A v P1 . Tng t, ta v mt ng trn tip xc vi ng trn I1 v ni tip trong gc BCD, t chng minh trn ta bit rng n phi tip xc vi ng trn I1 ti X, hn na n s hon ton xc nh bn trong hnh bnh hnh bi v X l im gia ca C v P2 . V vy, bt c im no thuc qu tch s chy qua X. chng minh rng bt k im no khc s khng chy qua. Ch rng bt k im no s hoc

.c

ca cc on AP1 v CP2 .

khng nm trn ng thng AC hoc s khng cho 1 trong 2 ng trn I1 hoc I2 c cha bn trong hnh bnh hnh. Do vy, qu tch thc s l giao

Chng 2 thi olympic Bungari


m
2.5. Tm tt c cc s thc m phng trnh x2 2mx 4(m2 + 1) c ng ba nghim phn bit. Li gii: p n: m = 3.

(2). Ta c ba trng hp m phng trnh ban u c th c 3 nghim phn bit: Phng trnh (1) c nghim kp hoc phng trnh (2) c nghim kp hoc hai phng trnh c mt nghim chung. Tuy nhin, trng hp th nht

khng xy ra v hin nhin 5m2 + 4 = 0 khng th tha mn vi mi gi tr thc m. Trong trng hp th hai, ta phi c 2(m3 + m + 2) = 0; m3 + m + 2 phn tch thnh (m + 1)(m2 m + 2) v tha s th hai lun dng vi mi gi tr

thc m. V vy ta phi c m=-1 trng hp ny xy ra. Khi nghim duy nht ca phng trnh ny l x=2 v phng trnh (1) tr thnh (x + 1)2 = 9, tc l x=2, -4. Nhng iu ny c ngha l phng trnh ban u ca ta ch c nghim l 2 v -4, tri vi yu cu ca bi ton.

gi tr x no x l nghim ca phng trnh ban u. Nhng phng trnh ny c th vit di dng (x m)2 = 5m2 + 4 (1)v (x 2)2 = 2(m3 + m + 2)

Cho hai tha s v tri ca phng trnh bng 0 ta nhn c hai phng trnh a thc. t nht mt trong cc phng trnh ny phi nghim ng vi

.c

x2 4x 2m(m2 + 1) = 0

10

Nguyn Hu in, HKHTN H Ni

Xt trng hp th ba, gi r l nghim ca phng trnh th x r l mt tha s ca c hai biu thc x2 2mx 4(m2 + 1) v x2 4x 2m(m2 + 1). Tr hai biu thc ny cho nhau ta nhn c x r l mt tha s ca (2m 4)x (2m3 4m2 + 2m 4), hay (2m 4)r = (2m 4)(m2 + 1). V

vy m = 2 hoc r = m2 + 1. Tuy nhin, trong trng hp th nht th c hai phng trnh bc hai ca ta tr thnh (x 2)2 = 24, v v vy, ta ch

c ch ra khng tha mn. V vy, ta ch c m=3. Khi cc phng trnh ca ta tr thnh (x 3)2 = 49 v (x 2)2 = 64, chng c cc nghim l x=-6, -4, 10, tha mn yu cu ca bi ton. 2.6. Cho ABC l tam gic u c din tch bng 7. Gi M, N tng ng l cc im trn cnh AB, AC sao cho AN=BM. Gi O l giao im ca BN v

thu c hai nghim phn bit. Vy ta phi c r = m2 + 1. Khi , thay vo ng thc (r 2)2 = 2(m3 + m + 2), ta c (m2 1)2 = 2(m3 + m + 2) hay (m + 1)(m 3)(m2 + 1) = 0. Do m = 1 hoc 3. Trng hp m=-1

ABC. Suy ra BOC=-MOC= 2 . V vy, O nm trn ng trn i xng 3 vi ng trn ngoi tip tam gic ABC qua BC. C nhiu nht hai im O trn ng trn ny v nm trong tam gic ABC t l khong cch t O ti BC v t A ti BC bng 2 , t l ny cng l t l din tch ca cc tam 7
2 gic OBC v ABC. V vy ta ch ra rng M B = 1 hoc 3 tng ng vi cc AB 3 v tr ca im O, v khng c t l no khc (tc l khng c hai im M cho 1 cng mt im O. Nu M B = 3 th AN = 1 , p dng nh l Menelaus cho AB AC 3 [BOC] 3 BO 3 BO tam gic ABN v ng thng CM, ta c ON = 4 , do [BN C] = BN = 7 . Suy 2 2 ra [BOC] = 3 CN = 7 v ta c iu phi chng minh. Tng t, nu M B = 3 , theo [ABC] 7 CA AB

ca tam gic ABC bin A thnh B, B thnh C, C thnh A; php quay ny cng bin M thnh L, L thnh N, N thnh M v bin O thnh P, P thnh Q, Q thnh O. Do OPQ v MLN l cc tam gic u ng tm vi tam gic

Li gii: (a) Ly im L trn BC sao cho CL=AN v gi P, Q ln lt l giao im ca CM v AL, AL v BN. Php quay vi gc quay 120o quanh tm

(b) Tnh gc AOB

.c

CM. Bit tam gic BOC c din tch bng 2. 1 (a) Chng minh rng M B hoc bng 3 hoc bng 2 . AB 3

nh l Menelaus ta c

BO =6, BN 7

do

[BOC] = BO = 6 . [BN C] BN 7

Suy ra

[BOC] 6 CN 2 = =7. [ABC] 7 CA

(b)

thi olympic Bungari


MB 1 =3 AB

11

2 . Do 3 MB 1 AN 1 AOB=AOM+MOB=ANM +P OQ = ANM + 3 . Nhng AB = 3 v AC = 3 nn d dng thy c N l hnh chiu ca M trn AC. V vy ANM = v 2 5 AOB = 6 . Lp lun tng t i vi trng hp cn li, ta c ANM = 6 v AOB = . 2

th MONA l mt t gic ni tip do A= pi v O = P OQ = 3

vi mi x [0; 1].

2.7. Cho f (x) = x2 2ax a2 3 . Tm tt c cc gi tr ca a |f (x)| 1 4

Li gii: p n: 1 a 42 . 2 th ca f(x) l mt parabol c im cc tiu (c ngha l h s a m) v

2.8. K hiu u(k) l c l ln nht ca s t nhin k. Chng minh rng

Li gii: t v(k) l c ln nht ca k c dng ly tha ca 2, nn u(k)v(k) = k. Trong {1, 2, ..., 2n} c 2ni1 gi tr ca k sao cho v(k) = 2i vi i n 1, v mt gi tr sao cho v(k) = 2n . Do , v tri bng 1 u(k) 1 = n+ n 2 k=1 k 4 T tng ca chui hnh hc ta c 1 2n
2n 2n n1

1 2n

k=1

2n

mn iu kin ca bi ra khi 0 < a

u(k) 2 . k 3

1 3 f (x) = 1 5 (a + 1)2 < 4 . Mt khc, f (a) = 2a2 4 nn ta phi c 4 a 42 f (a) 1. Ngc li, nh gi f(0), f(a), f(1) ta ch ra c f tha 2 . 4

k=1

u(k) 2 2 = 4n + (1 4n ) > . k 3 3

.c
i=0

gi tr nh nht ca f(a) nm trong phm vi theo yu cu ca bi ton, tc 9 l f(1) nm trong gii hn ny. T a 1 ta c 1 < (a + 1)2 4 v v vy 2

1 Nhng ta c 1 a + 1 1, 4 (a + 1)2 1, 1 5 (a + 1)2 1. T 2 4 4 5 (a + 1)2 = f (1), ta c f tha mn iu kin ca bi ra khi 1 a 0. 4 2 Vi a > 0, f gim vi 0 x a v tng vi a x 1. V vy ta cn ch ra

1 nh l (a;f(a)). T f (0) = a2 3 ta c |a| 2 f (0) 1. Gi s a 0 4 th parabol ca ta tng nghim ngt trong khong t 0 n 1, do f (1) 1.

2ni1 . 2n+i

12

Nguyn Hu in, HKHTN H Ni

2.9. Tm tt c cc s thc tha mn h x3 = 2y 1 y 3 = 2z 1 z 3 = 2x 1.

Li gii: Trc ht ta ch ra rng x = y = z. Gi s tri li rng x = y. 3 +1) 3 +1) Nu x > y, th y = (x 2 > (y 2 = z, nn y > z, v tng t z > x, mu thun. Tng t, nu x < y th y < z v z < x, mu thun. Nn cc nghim ca h phng trnh c dng x = y = z = t vi t l nghim ca phng trnh t3 = 2t 1. Vy, nghim ca h phng trnh l 1 + 5 1 5 . , x = y = z = t, t 1, 2 2

2.10.Tm s t nhin a nh nht phng trnh sau c mt nghim thc:

chng minh a l gi tr nh nht, ta vit phng trnh di dng (cos(a-x)-1)2 + cos 3x x cos +1 + 2a 2a 3

Li gii: Gi tr nh nht ca a l 6. Phng trnh tha mn khi a=6, x=8.

.c

cos2 (a x) 2 cos (a x) + cos

3x x +2=0 cos + 2a 2a 3

=0

Do c hai s hng v tri u khng m nn ng thc xy ra th chng phi cng bng 0. T cos(a-x)-1 = 0 ta c x phi l mt s nguyn ng d vi a trong php chia cho 2. T s hng th hai bng 0, ta thy cc gi tr cosin phi nhn gi tr bng 1 v -1. Nu cos x + = 1 th x + = 2k 2a 3 2a 3 6a vi gi tr k nguyn v nhn hai v vi ta c 3x 4a (mod12a). Khi

2, v vy x phi chia ht cho 2 v a cng phi tha mn iu . Hn na, c hai trng hp ta cng u c -2a v 4a cng phi chia ht cho 3, v th a phi chia ht cho 3. Tm li ta c 6 phi l c ca a v a=6 l gi tr nh nht cn tm.

th nu cos x + = 1 th x + = (2k + 1) v nhn hai v vi 6a ta 2a 3 2a 3 c 3x 4a (mod12a). Trong c hai trng hp ta u c 3x chia ht cho

Chng 3 thi olympic Canada


3.11.C bao nhiu cp s (x; y) nguyn dng vi x y tho mn gcd(x, y) = 5! v lcm(x, y) = 50! ?

V ta c 15 s nguyn t nn c 215 cp, v trong bt k cp no cng hin nhin c x = y ( do gcd v lcm khc nhau), do c 214 cp vi x y.

Do , vi mi s nguyn t p, ta c f (x, p) = f (5!, p) f (y, p) = f (50!, p) f (y, p) = f (5!, p) f (x, p) = f (50!, p)

3.12.Cho trc mt s hu hn cc khong ng c di bng 1 sao cho hp ca chng l khong ng [0, 50], chng minh rng tn ti mt tp con ca cc khong khng giao vi tt c cc khong khc. Li gii: Xt I1 = [1 + e, 2 + e] , I2 = [3 + 2e, 4 + 2e] , ..., I24 = [47 + 24e, 48 + 24e]

lm cho bi ton n gin hn, ta xc nh f (a, b) l m ln nht ca b chia cho a. (Ch rng g(50!, b) > g(5!, b) vi mi b < 50.)

.c

Li gii: Trc ht, ch l c 15 s nguyn t t 1 n 50: (2, 3, 5, 7, 11, 13, 17, 19, 23, 29, 31, 37, 41, 43, 47).

14

Nguyn Hu in, HKHTN H Ni

trong e nh 48 + 24e < 50. hp cc on cha 2k + ke, ta phi c mt on m phn t nh nht nm trong Ik. Tuy nhin, s khc nhau gia mt phn t trong tp Ik v Ik + 1 lun ln hn 1, v vy cc tp ny khng chng ln nhau. T 24 khong ban u v [0, 1] ( phi tn ti v hp l [0, 50] ) ta c 25 khong ri nhau m tng di tt nhin bng 25. 3.13.Chng minh rng:
1 1999 3 < 1 . 4 . ... . 1997 < 2 1998 1 44 1 1997 4 1998 Li gii: t p = 2 . 3 . ... . 1998 v q = 2 . 5 . ... . 1999 . Ch rng p < q, v vy 4 3 2 p2 < pq = 1 . 3 . ... . 1998 = 2 1999 1 . 1999

Do , p< 1 1999
1 2

<

1 44

ng thi 21998 = Do .

1998

.c

p=

1998 1998! = 21998 999 )2 (999!.2 999

+ ... + p>

1998 1998

Li c

< 1999

1998 999

3.14.Cho O l mt im nm trong t gic ABCD sao cho AOB + COD = . Chng minh rng OBC = ODC. Li gii: Tnh tin ABCD theo vect AD th A v D nh nhau, v v vy B v C nh nhau. Ta c COD + CO D = COD + A O D = 1800 nn t gic OC O D ni tip. Do ODC = OO C. 3.15.Biu din tng sau
n k=0 (1)k k 3 +9k 2 +26k+24

1 1999

n k

p, q l cc a thc vi cc h s nguyn.

v dng p(n)/q(n), trong

thi olympic Canada Li gii: Ta c


n k=0 (1)k k 3 +9k 2 +26k+24 n

15

n k

n k n+4 n+4 k

=
k=0 n

(1) (k+2)(k+3)(k+4)

=
k=0

k+1 (1)k (n+1)(n+2)(n+3)(n+4) n+4 k=4

1 (n+1)(n+2)(n+3)(n+4)

(1)k (k 3)
n+4 k=0

k+4

n+4 k=0

n+4

=
k=0 n+4

k=1

= = Do
n+4 k=4

(1)k (k 3)
3

1 n+4

(1 1)n+3 = 0

1 n+4

n+4 k=1

(1)k

n+3 k1

n+4 k

k n+4 2 =
(n+1)(n+2) 2

= 3

k=0

n+4 0

(1)k (k 3) 2

n+4

n+4 1

(1)k k

3 (1 1)n+4

n+4

(1)k k

n+4 k

.c

(1)k (k 3)

n+4 k

m
(1)k n+4 k

16 v tng cho bng

Nguyn Hu in, HKHTN H Ni 1 2 (n + 3) (n + 4)

.c

Chng 4 thi olympic Chine


4.16. Cho x1 , x2 , ..., x1997 l cc s thc tha mn iu kin 1 (a) 3 xi 3 vi mi i = 1; 2; ...; 1997 (b) x1 + x2 + ... + x1997 = x12 1997 Li gii: Do x12 l mt hm li ca x nn tng cc ly tha bc 12 ca xi l cc i nu mi gi tr xi l u mt ca khong quy nh. 1 Gi s c n gi tr xi bng 3 , 1996 n c gi tr bng 3 v cc gi tr cui cng bng n 318 3 + 3(1996 n) 3 1 Do gi tr cui cng ny phi nm trong min 3 ; 3 nn 1 318x3 + n 3x(1996 n) 3 tng ng vi 1 4n 6942 3 c duy nht mt s nguyn n = 1736 tha mn. Khi , gi tr cui cng l
2 3

4 1736x36 + 260x36 + ( )6 3

v gi tr ln nht cn tm l:

.c

18

Nguyn Hu in, HKHTN H Ni

4.17. Cho t gic li A1 B1 C1 D1 v mt im P nm trong t gic li . Gi s cc gc P A1 B1 v P A1 D1 l cc gc nhn, tng t cho ba nh cn li. Xc dnh Ak , Bk , Ck , Dk l hnh chiu ca P ln cc ng thng Ak1 Bk1 , Bk1 Ck1, Ck1 Dk1 (k = 2, 3, ...) a) Trong cc t gic Ak Bk Ck Dk (k = 1, 2, 3, ..., 12) th t gic no ng dng vi t gic th 1997 b) Gi s rng t gic th 1997 l ni tip. Hi trong 12 t gic u tin th t gic no cng ni tip ng trn Li gii: Ta c Ak chnh l chn ca cc ng vung gc t P n Ak1 Bk1 v tng t nh vy cho cc im cn li. Do cc t gic ni tip vi cc ng knh P Ak , P Bk , P Ck , P Dk ta c

P Bk+3Ck+3 = P Ak+4 Bk+4

4.18. Ch ra tn ti v s s nguyn dng n sao cho cc s1, 2, 3, ..., 3n c th

theo th t ny tha mn iu kin sau: a) a1 + b1 + c1 = an + bn + cn b) a1 + a2 + ... + an = b1 + b2 + ... + bn = c1 + c2 + ... + cn l bi ca 6 Li gii: Tng cc s nguyn t 1 n 3n l
3n(3n+1) 2

a1 , a2 , ..., an , b1 , b2 , ..., bn , c1 , c2 , ..., cn

c gn cho

Do vy, cc t gic th 1, 5, 9 ng dng vi t gic th 1997. Nu t gic th 1997 l ni tip th cc t gic th 3, 7 v 11 cng vy.

Mt khc, ta cng c P Bk Ak =P Bk+1Ak+1 v tng t nh vy cho cc gc cn li.

.c

P Ak Bk = P Dk+1Ak+1 = P Ck+2Dk+2

trong ta i hi phi

va l bi ca 6n v 9. V th, n phi l bi ca 3 ng d vi 1 theo mddun4. Ta s ch ra rng tn ti s sp xp cho n = 9m . Vi n = 9 ta c s sp xp sau:

thi olympic Chine 8 1 16 17 10 15 26 19 25 3 5 7 12 14 4 9 24 16 2

19

21 23 13 18

11 22 27 20

(trong , dng u tin l a1 , a2 , ..., an v tip tc). iu ny chng t t s sp xp cho m v n dn n s sp xp cho mn ai+(j1)m = aj + (m 1)aj (1 j m, 1 j n) v tng t cho bi , ci . 4.19. Cho ABCD l mt t gic ni tip. Cc ng thng AB v CD ct nhau ti P . Cc ng thng AD v BC ct nhau ti Q. Gi E v F l giao im tip tuyn t Q vi ng trn ngoi tip t gic ABCD. Chng minh rng P, E, F thng hng.

A E

h v
D

.c
B F

C Li gii: Gi X l tip tuyn ca ng trn ti im X nm trn ng trn. chng minh P, E, F thng hng ta chng minh cc im cc ca n trng nhau. Cc tip tuyn E v F ti cc im E v F ct nhau ti Q. Do P l giao ca AB v CD nn im cc ca P l ng thng i qua giao im ca A giao vi B v C giao vi D .

20

Nguyn Hu in, HKHTN H Ni

Ta s chng minh nhng im ny thng hng vi Q. Tuy nhin theo nh l Pascal cho lc gic suy bin AADBBC th Q v giao im ca AC vi BD s cng tuyn. Tng t, p dng nh l Pascal cho lc gic suy bin ADDBCC ta cng c kt qu tng t. 4.20. Cho A = {1, 2, ..., 17} v hm f : A A tho mn K hiu f [1] = f (x) v f [k+1] (x) = f (f [k] (x)) vi k N

Tm s t nhin ln nht M sao cho tn ti song nh f : A A tha mn iu kin sau: a) Nu m < M v 1 i 17 th f [m] (i + 1) f [m] (i) khng ng d vi 1 theo mun 17 b) Vi 1 i 17 th

l 8 v nh vy M 8.

4.21. Cho a1 , a2 , ... l cc s khng m tha mn am+n am +an (m, nN). Li gii: Bng phng php quy np vi k an kam +anmk vi k < m n

n Chng minh rng: an ma1 + ( m 1)am vi mi nm.

t n = mk + r vi r{1, 2, ..., m} th an kam +ar = nr am +ar ma1 m (Do am ma1 v ar ra1 )

bc. Suy ra, M 8. Nu c mt qu o khc, mt hoc thm ch hai qu o c di ln nht

Nu 1 v 16 cng trn mt qu o ca hon v f th qu o ny c di ln nht l 16 v 1 hoc 16 phi nh x vi nhng gi tr khc nhau sau 8

Ch rng bng cch sp xp vi chu trnh chuyn ta c th gi s rng f (17) = 17, do M l s nguyn u tin sao cho f [M ] (1)) bng 1 hoc bng 16. Cng nh vy cho 16.

.c

Li gii: nh x f (x) = 3x(mod17) tha mn yu cu cho M = 8 v ta s ch ra rng n l gi tr ln nht.

f [m] (i + 1)f [m] (i) 1(mod17) ( y f [k] (18)) c xc nh bng f [k] (1)))

Chng 5 thi olympic Colombia


m
5.22. Cho mt bng k kch thc n n v 3 mu. Ta s t mu mi on

mi c t mu A th s c 2 cch chn mu cho 2 on cn li. Do vy: a( n + 1) = 2an v an = 3.2n . Tr li yu cu ban u, c 3n cch t mu gc trn v 3.2n cch t mu mi dng. Nh vy c tt c 3m+n .2m.n cch t mu.

mt vung n v kp vo cnh phi ca dng c dng mi c kch thc 1 (n + 1), mu trn ca vung mi bit. Nu on trn

5.23. Ta chi mt tr chi vi tam gic u ca n.(n+1) ng xu (vi n ng 2 xu trn mi cnh). u tin, tt c cc ng xu u t sp. Trong mi ln lt ta c th lt 3 ng xu lin tip lin k. Mc ch l tt c cc ng xu bi

By gi ta tm an+1 t an . Cho bt k mu no ca dng 1 n, gi thit WLOG rng on bn phi nht c t mu A. By gi ta tng tng thm

Vi n = 1, gi s WLOG on trn cng ca bng c t mu A, khi c 3 cnh chn on khc c t mu A v c 2 cch chn mu cn li v nh vy c tt c a1 = 6 cch t mu.

Li gii: Gi 3 mu trn l A, B, C. Gi an l s cch t mu ca 1 n dng u tin ca bng.

.c

ca li bi mt trong 3 mu trn sao cho mi vung n v c 2 cnh cng mu v 2 cnh cn li cng mu khc. Hi c bao nhiu cch t mu c th?

22

Nguyn Hu in, HKHTN H Ni

lt nga. Hi n bng bao nhiu hon thnh vic ? Li gii: Tr chi ny s c hon thnh vi cc gi tr ca n m chia 3 d 0 hoc 2. R rng thy ngay trng hp n gin nht. Bi ton ng vi n = 2 v n = 3 (mi trng hp c bn kh nng lt). Vi cc gi tr n ln hn, chn mi ln lt 3 ng xu, s ng xu cn d c lt mt ln, v nhng ng xu dc theo cc cnh ca tam gic c th c lt 3 ln. V vy tt c cc ng xu u nga. Trong khi , mi ng xu bn trong tam gic c lt 6 ln, v chng lp thnh mt tam gic c s ng xu trn mi cnh l n 3. Theo phng php quy np, cc gi tr n nh trn u tha mn. Nu n chia 3 d 1, ta t cc ng xu bi cc mu vng, v xanh sao cho bt k 3 ng xu no cnh nhau cng c mu khc nhau. Cng vy, 3 ng

5.24. Cho ABCD l mt hnh vung c nh. Xc nh tt c cc v tr c th ca S hnh vung P QRS vi P v R nm trn 2 cnh khc nhau ca ABCD; Q nm trn ng cho ca ABCD. Xc nh tt c cc v tr c th ca im S. Li gii: Cc v tr to thnh cc hnh vung khc nhau, quay 450 vi tm l giao ca hai ng cho ca hnh vung. Gi s ta a ra cc s phc sao cho A = 0; B = 1; C = 1 + i v D = i. Trc tin, gi s P v R nm trn 2 cnh lin nhau ca ABCD. Khng mt tnh tng qut, gi s P nm trn AB v R nm trn BC. Trong trng hp ny Q phi nm trn AC. (Vi bt c im no nm trn BD khng trng

nga mu vng v mu . c th l mt trng hp nu tt c cc ng xu u lt nga. Vy cc ng xu khng th c sp xp.

ca s ng xu nga mu . T s khc nhau v tnh chn, l ca s ng xu nga ca mi mu, chng ta khng th kt thc nu khng c s nh nhau v tnh chn, l ca s ng xu

s ng xu mu xanh hoc mu l 1 ng. Lc ny tnh chn, l ca s ng xu nga mu vng khc vi tnh chn, l

.c

xu lin tip bt k trn mt hng cng c mu khc nhau. Nu cc ng xu gc u c mu vng th s ng xu mu vng nhiu hn

thi olympic Colombia

23

vi tm ca hnh vung, php quay vi gc quay 900 m AB khng trng vi AD.) Nu P = x; Q = y + yi th R = (2y x)i v S = (x y) + (y x)i, trong cc bin dc theo hnh vung cho. Nu P v R nm trn 2 cnh i din ca ABCD, khng mt tnh tng qut,

mt im duy nht trn CD. Nh vy P = 2y 1; R = i v S = y 1+(1y)i vi cc bin dc theo hnh vung cho. 5.25. Chng minh rng tp hp cc s nguyn dng c th chia thnh v hn cc tp c v hn s A1 , A2 , . . . , (cc tp ri nhau) sao cho nu x, y, z, w thuc Ak vi k no , khi x y v z w cng thuc tp Ai (trong i khng nht thit bng k) khi v ch khi A
x y

ta gi s P nm trn AB, R nm trn CD v Q nm trn AC. Hn na, ta gi 1 thit Q = y + yi vi 2 y 1. Quay cnh AB mt gc 900 ta c Q trng vi

z . w

.c m a t h

o
R chia ny s tha mn yu cu ra. Tht vy, gi s x, y, z, w Ak vi x > y v z > w Ta c: x = (2k 1)(2( a + b)), y = (2k 1)2a , z = (2k 1)2( c + d), w = (2k 1)2c . Khi x y = (2k 1)(2b 1)(2a ), z w = (2k 1)(2d 1)(2c )
z z Do x = 2b ; w = 2d , x = w khi v ch khi b = d khi v ch khi x y v z w y y c c s l chung ln nht.

Li gii: Gi Ak l tp bao gm tt c cc s c dng (2k 1)2n v cch phn

Chng 6 thi olympic Czech v Slovak Repubulick


.c
6.26. Cho tam gic ABC c ba cnh ln lt l a, b, c v ba gc tng ng

a2 b2 (a b) = 8R3 sin2 sin2 (sin sin )

= 8R3 sin2 3 sin2 (sin 3 sin ) = 8R3 (sin 3 sin )2 (sin 3 + sin ) = 8R3 8 cos2 2 sin2 sin2 cos = 8R3 sin2 1800 4 = 8R3 sin2 (sin ) = bc2 (sin )

15o , = 125o , = 40o

Ni chung, chiu ngc li sai; ta c th ly = 3 360o , v d nh =

Li gii: Theo h qu ca nh l Sin ta c a = 2R sin , b = 2R sin , c = 2R sin , vi R l tm ng trn ngoi tip tam gic ABC. V vy:

, , . Chng minh rng: nu = 3 th (a2 b2 ) (a b) = bc2 , xt xem chiu ngc li c ng khng.

thi olympic Czech v Slovak Repubulick

25

6.27. Mi cnh v ng cho ca mt n-gic u (n 3) c t mu hoc mu xanh. Ta chn mt nh v thay i mu ca cc on thng nhn im lm u mt t mu thnh mu xanh v ngc li. Chng minh rng, vi bt k cch t mu lc u th no, ta vn c th bin s cnh mu xanh xut pht t mi nh l s chn. Chng minh rng, kt qu cui cng ca vic t mu c quy nh da trn cch t mu ban u. Li gii: Nhn thy, th t chn cc nh khng nh hng n kt qu t mu cui cng. V vic chn mt nh hai ln khng nh hng n kt qu t mu. V th, vic chn mt tp hp cc nh cng cho kt qu nh vic chn cc nh cn li: Qu trnh sau cng tng t nh vic chn mt tp hp cc nh u tin, sau chn tt c cc nh ( y, trong tp hp cc nh cn li, nhng nh ban u c chn s l ln by gi c chn theo s chn

Ta xt mt cch t mu vi kt qu nh mong mun. Cui cng, s on mu xanh ai xut pht t nh th i l ai + B bi (mod2). Khi , s on mu xanh xut pht t cc nh l bng nhau, do , bj bk khi v ch khi lc u aj ak . V vy, hoc bi 1 khi v ch khi ai 1 hoc l bi 1 khi v ch khi ai 0, ta c kt qu t mu nh trn. Do kt qu t mu l duy nht.

Bi ton c chng minh Ch : vi mt 2n-gic (n 2), th vic chn mt nh s lm thay i tnh chn l ca tt c cc ai . V th, ta khng th c c kt qu tt c cc ai l s chn, nu cc ai ban u khng cng tnh chn l. V nu c tt c cc ai l s chn th kt qu t mu cui cng l khng duy nht.

, tt c cc nh u c mt s chn cc on mu xanh. Vy ta chng minh c kt qu t mu cui cng l duy nht.

ny. Tnh chn l ca cc s ai thay i 2x 1 ln thnh s chn. Tnh chn l ca cc s ai cn li th thay i 2x ln gi nguyn l s chn. Do

khc i mu nn ai cn li thay i tnh chn l. Tnh tng s ai th cho ra kt qu l hai ln tng s cc on mu xanh, v th c mt s chn cc nh vi ai l s l -gi l 2x cc nh. Chn cc nh

.c

t nh th i, gi bi l s ln mi nh c chn v B = bi . Khi chn nh k th ak tr thnh 2n a ak ; mt khc, mi on t nh k ti mt nh

ln v ngc li). t tn cc nh l 1, 2, ..., 2n + 1. Gi ai l s cc on mu xanh xut pht

26

Nguyn Hu in, HKHTN H Ni

6.28. Cho t din ABCD c chia thnh 5 khi a din li sao cho mi mt ca t din ABCD l mt mt ca khi a din (khng c mt no b chia), v hai khi a din bt k trong 5 khi a din hoc c mt nh chung, hoc c mt cnh chung hoc c mt mt chung. Hi 5 khi a din c tng s mt t nht l bao nhiu? Li gii: Tng s mt nh nht l 22. Khng c khi a din no c chung hai mt vi t din ABCD, nu khng, do tnh li ca khi a din nn n s l ABCD. Do , c mt khi a din P khng c chung mt mt vi ABCD v cc mt ca n nm bn trong t din ABCD. Do , mi mt ca P phi l mt chung ca P vi mt khi a din khc, c ngha l P c chung t nht 3 mt vi mt trong nhng khi a din cn li. ng thi, bt k mt no ca khi a din khng l mt mt ca t din ABCD th phi l mt mt ca khi a din khc. Tc l, tng s cc mt ca 5 khi a din l s chn.

Li gii: Ly pk l s nguyn t th k, k 1 . Chn a1 = 2. Vi n 1 , ly an+1 l s nguyn nh nht ln hn an m an+1 p (modpk+1 ) vi mi k n. Nhng s nguyn ny tn ti theo nh l Thng d Trung Hoa. V vy, vi mi k + an 0 (modpk+1 ) vi n k + 1 . Do , trong dy {k + an }, gi tr ln nht trong k + 1 c th l s nguyn t; t s hng th k = 2 tr

i, cc s hng l bi ca pk+1 v phi l hp s .Ta c iu phi chng minh. 6.30. Vi mi s t nhin n 2, hy tm gi tr ln nht ca biu th sau: Vn = sin x1 cos x2 + sin x2 cos x3 + + sin xn cos x1

k 0, m dy {k + an } ch cha mt s hu hn s nguyn t.

6.29. Ch ra rng tn ti mt dy cc s t nhin tng dn {an } vi mi n=1

mn cc iu kin bi m tng s mt ca 5 khi a din ny bng 22.

l A, C, D. iu ny l v l. Do , tng ca cc mt ln hn hoc bng 22. Ta s ch ra trng hp du bng xy ra. Ln lt ly P v Q gn vi A v B. Khi 5 khi a din AP CD, P QCD, BQCD, ABDP Q v ABCP Q tha

nht 2 nh chung vi ABCD. V nu n c 2 im chung vi ABCD, gi s l A v B, khi , n s c ti a 2 nh chung vi t din m 3 trong 4 nh

.c

Do mi khi a din phi c t nht 4 mt, nn tng s mt t nht l 20. Gi thit, y l tng. Khi mi khi a din l mt t din c 4 nh v P c t

thi olympic Czech v Slovak Repubulick vi x1 , x2 , , xn l cc s thc bt k.

27

Li gii: p dng bt ng thc 2ab a2 + b2 ta c: Vn sin2 xn + cos2 x1 n sin2 x1 + cos2 x2 sin2 x2 + cos2 x3 + ++ = 2 2 2 2
4

Du bng xy ra khi x1 = x2 = = xn =

6.31. Cho hnh bnh hnh ABCD m ABD l tam gic nhn, v BAD = . 4 Trn cc cnh ca hnh bnh hnh, ly cc im K thuc AB, L thuc BC, M thuc CD, N thuc DA sao cho KLMN l t gic ni tip c bn knh bng bn knh ng trn ngoi tip cc tam gic ANK v CLM. Tm qu tch cc

giao im ca ng cho ca t gic KLMN. Li gii: Do cc cung cha cc KLN , KMN , LKM , LNM trn ng trn ngoi tip t gic KLMN v cc cung cha KAN , LCM ln lt trn ng

AB bin thnh CD v BC bin thnh DA, v vy S nm trn on BD.

giao im ca KM v NL, l cc tam gic vung cn ti S v ng dng vi nhau. Khi , qua php ng dng, K bin thnh M, L bin thnh N,

.c

trn ngoi tip tam gic AKN v CLM c cng s o, cc gc u bng nhau v c cng s o l 45o . Cc tam gic SKL v SMN vi S l

Chng 7 thi olympic Php


7.32. Ti mi nh ca 1997- gic c gn mt s nguyn, sao cho tng ca chng bng 1. Bt u t mt nh no , ta gn theo chiu ngc kim ng

Li gii: C. Gi bk l tng ca k s nguyn u tin, ta c b1997 = 1. Gi x

Li gii: Ta c hnh tr ct hnh cu theo mt ng trn bn knh s < r. Khong cch t tm ca hnh cu n mt phng cha ng trn ny l R2 s2 . Li c hnh tr cng ct hnh nn theo mt ng trn bn knh s, khong cch t tm hnh cu n mt phng cha ng trn bng s R2 /r 2 1. (V khong cch t tm ca hnh cu n mt phng cha ng trn l R2 r 2 ). V vy, th tch ca hnh tr l: s2 R 2 s2 s R2 /r 2 1

Chng ta tm gi tr ln nht ca biu thc trn bng cch cho o hm theo

mt hnh cu tm O bn knh R v mt hnh nn nh O ct hnh cu theo mt ng trn bn knh r, c cng trc vi hnh nn.

7.33. Tm th tch ln nht ca mt hnh tr c cha trong phn chung ca

l gi tr nh nht ca bk ta tm mt s k ln nht m bk1 = x. Sau ta bt u t k nh th mi tng s s l s dng.

.c

h quanh a gic. Hi c th chn mt nh bt u m tng ca k s nguyn u tin l dng vi k = 1, 2, .., 1997

thi olympic Php s bng 0: s3 0 = 2s R2 s2 3s2 2 s2 R

29

R2 /r 2 1

Chuyn v v bnh phng ta c:

s4 4R2 s2 + 4R4 9s2 R2 s2 r 2 = R 2 s2 r2 Gii phng trnh ta c: s2 = 3R2 + r 2 + (9R2 r 2 ) (R2 r 2 ) 6

V thay s2 vo cng thc th tch trn cho ta th tch ln nht. 7.34. Tm din tch ln nht ca hnh chiu vung gc ca hnh lp phng Li gii: Nhn thy hnh chiu ca hnh lp phng l tng hnh chiu ca n v ln mt mt phng.

ca cc ng trung tuyn ca n. Vi mi s thc dng , gi () l s thc c xc nh bi : a + b + c = () (m + n + p ) (a). Tnh (2) (b). Tnh gii hn ca () khi dn ti 0. (c). Vi iu kin no ca tam gic ABC th () khng ph thuc vo . Li gii: (a). Gi m, n, p l di ca cc ng trung tuyn tng ng vi cc cnh a, b, c v gi s a b c. D dng tnh c m2 = (2b2 + 2c2 c2 )/4

7.35. Cho tam gic ABC vi a, b, c l di ca cc cnh v m, n, p l di

gc vi mt v mt phng chiu. Nh vy nu cc tch l x, y, z th gi tr ln nht ca din tch hnh ca chiu hnh lp phng bng gi tr ln nht ca tng x+y +z vi iu kin x2 + y 2 +z 2 = 1. Mt khc, theo bt ng thc Cauchy-Shwarz 3 x2 + y 2 + z 2 (x + y + z). Du bng xy ra khi v ch khi x = y = z. Khi , gi tr ln nht ca din tch bng 3

.c

3 mt ca hnh lp phng i mt vung gc vi nhau. Din tch hnh chiu ca mi mt bng gi tr tuyt i ca tch hai vect n v ln lt vung

30

Nguyn Hu in, HKHTN H Ni


2 3

v tng t vi hai trung tuyn cn li, v vy (2) = (b). Nu x y z v 0 th x (x + y + z )1/ 31/ x

v do (x + y + z )1/ dn ti x. Vy chng ta c lim () =


0

a p

(c). () khng ph thuc vo ta phi c


c 23 , n

a2 p2

4 = 3 , dn n a2 + c2 = 2b2 .

= b 23 , p = a 23 . Kt hp vi gi thit ta c m = Vy () l hng s khi tam gic ABC tha mn iu kin trn.

.c

Chng 8 thi olympic c


m
8.36. Xc nh tt c cc s nguyn t p sao cho h p + 1 = 2x2 p2 + 1 = 2y 2 c nghim x, y nguyn.

p2 + 1 = 2(p x)2 = 2p2 4xp + p + 1, nn p = 4x 1, 2x2 = 4x, x l 0 hoc 2 v p l 1 hoc 7. Hin nhin 1 khng l s nguyn t, v p = 7, (x, y) = (2, 5) l nghim ca bi ton. 8.37. Mt hnh vung Sa ni tip mt tam gic nhn ABC vi hai nh nm trn cnh BC v mt nh nm trn AB, mt nh nm trn AC. Cc hnh vung Sb , Sc c xy dng tng t. Vi nhng trng hp no ca tam gic ABC th cc hnh vung Sa , Sb , Sc l bng nhau. Li gii:

tnh tng qut c th gi s rng x, y 0. Ch rng p + 1 = 2x2 l chn, nn p = 2. Hn na, 2x2 1 2y 2 (mod p) suy ra x y (mod p) v p l l. T x < y < p, ta c x + y = p. Do

Li gii: Ch c duy nht s nguyn t p = 7 tha mn bi ton. Khng mt

.c

32

Nguyn Hu in, HKHTN H Ni

A xa

t R l bn knh ng trn ngoi tip tam gic ABC v t xa , xb , xc l di cc cnh ca Sa , Sb , Sc tng ng. K hiu , , l cc gc BAC, CBA, ACB. Gi s rng Sa c cc nh P, Q nm trn cnh BC trong P gn B hn. Khi

2R sin = BC = BP + P Q + QC = xa cot + xa + xa cot . 2R sin = 1 + cot + cot 2R sin sin sin = sin sin + cos sin + cos sin 2R sin sin sin = sin sin + sin

v tng t cho xb , xc . T xa = xb suy ra sin sin + sin = sin sin + sin 0 = (sin sin )(sin 1). T tam gic ABC l nhn, ta c sin = sin , suy ra = v trong trng hp tri li th + = l khng th xy ra trong tam gic. Tng t nh vy = , nn ABC l tam gic u. 8.38. Trong mt cng vin, 10000 cy c trng theo mt hnh li vung. Xc nh s ln nht cc cy c th n h m sao cho t mi gc cy n, bn khng th nhn thy mt gc bt k khc. (Gi s rng bn knh ca cc cy l khng ng k so vi khong cch ca cc cy k nhau.)

xa

.c

thi olympic c

33

nh vo 2500 hnh vung ri nhau thc s, do , c th n h nhiu nht 2500 cy.

Li gii: Ta nhn thy rng trong mt hnh vung gm bn cy k nhau ta ch b i c nhiu nht mt cy. T li 100 100 ta c th chia 10000

phi l l, do , cy (a + p, b + q) s chn tm nhn gia cc cy (a, b) v (c, d). 8.39. Cho mt hnh vin phn AMB vi gc trung tm AMB nh hn 90o . T mt im P bt k trn cung AB h cc ng vung gc P C v P D xung MA v MB (C MA, D MB). Chng minh rng di on thng CD

chn. Xt hai gc bt k (a, b) v (c, d) vi a, b, c, d l chn. Xt p/q l mt biu din ca (d b)/(c a) vi cc hng t l b nht ( y p, q c cng du vi d b, c a, tng ng), khi , t nht mt trong hai s a + p v b + q

Ta s ch ra mt cch n 2500 cy tha mn bi ton. ng nht cc cy vi ta (x, y) trn li, 0 x, y 99, v n h tt c cc cy c cc ta

a
M

t
P B

T P CM = P DM = /2, nn t gic P CMD ni tip ng trn ng knh MP . Do , p dng nh l hm s sin cho tam gic MCD, ta c, CD = MP sin CMD l mt hng s. 8.40. Trong mt hnh vung ABCD xy dng bn cung trn vung, mi cung trn c tm tng ng l A, B, C, D v cha hai nh k vi tm. Bn cung trn ny ct nhau ti bn im E, F, G, H nm bn trong ABCD, cc im

h
A

.c

Li gii:

khng ph thuc vo v tr im P trn cung AB.

34

Nguyn Hu in, HKHTN H Ni

Li gii: K hiu [C] l din tch ca cung trn C, v [S] l din tch ca hnh vung S. Khng mt tnh tng qut c th coi E l giao im ca cc cung

ny cng lm thnh mt hnh vung gi l S. Gi C l ng trn tip xc vi c bn cung trn trn. Hy so snh din tch ca S v C.

Vy din tch ca ng trn C ln hn din tch ca hnh vung S.

.c

tng ng. Li do tnh i xng th A, I, K, C thng hng. Khi 2AB = AK + IC = AC + IK = 2AB + IK, nn, IK = (2 2)AB. Do vy (3 2 2) AB 2 > (2 3)AB 2 = [S]. [C] = IK 2 = 4 2

trn m gn AB nht, v G l giao im gn CD nht. H cc ng vung gc EE xung AB, v GG xung CD. Theo tnh cht i xng, E , E, G, G l thng hng. T AB = BG = AG ta c ABG l tam gic u v ng cao GE = 3AB/2. Tng t, G E = 3AB/2. Do , 3AB = GE + G E = AB + EG, nn EG = ( 3 1)AB v [S] = EG2 /2 = (2 3)AB 2 . t I, K l cc tip im ca ng trn C vi cc cung trn tm C v A,

m
K

C F

m
E

G I H

8.41. K hiu u(k) l c l ln nht ca s t nhin k. Chng minh rng u(k) 2 1 . 2n k=1 k 3
2n

thi olympic c

35

Li gii: t v(k) l c ln nht ca k c dng ly tha ca 2, nn u(k)v(k) = k. Trong {1, 2, ..., 2n} c 2ni1 gi tr ca k sao cho v(k) = 2i vi i n 1, v mt gi tr sao cho v(k) = 2n . Do , v tri bng 1 2n
2n

k=1

1 u(k) = n+ k 4

n1

i=0

2ni1 . 2n+i

T tng ca chui hnh hc ta c 1 2n


2n

k=1

u(k) 2 2 = 4n + (1 4n ) > . k 3 3

8.42. Tm tt c cc s thc tha mn h x3 = 2y 1 y 3 = 2z 1

thun. Tng t, nu x < y th y < z v z < x, mu thun. Nn cc nghim ca h phng trnh c dng x = y = z = t vi t l nghim ca phng trnh t3 = 2t 1. Vy, nghim ca h phng trnh l 1 + 5 1 5 . , x = y = z = t, t 1, 2 2 8.43. nh ngha cc hm s f (x) = x5 + 5x4 + 5x3 + 5x2 + 1 g(x) = x5 + 5x4 + 3x3 5x2 1. Tm tt c cc s nguyn t p m tn ti s t nhin 0 x < p, sao cho c f (x) v g(x) u chia ht cho p, v vi tng gi tr ca p, hy tm tt c cc gi tr ca x tng ng.

Li gii: Trc ht ta ch ra rng x = y = z. Gi s tri li rng x = y. Nu x > y, th y = (x3 + 1)/2 > (y 3 + 1)/2 = z, nn y > z, v tng t z > x, mu

.c

z 3 = 2x 1.

36 Li gii: Ch rng

Nguyn Hu in, HKHTN H Ni

f (x) + g(x) = 2x3 (x + 1)(x + 4). Nn nu p l c ca f (x) v g(x) th v p l s nguyn t nn p phi l c ca t nht mt trong cc s sau 2, x, x + 1, x + 4. T f (0) = 1 v f (1) = 17 dn n p = 2. Hn na, p cng khng th l c ca x, v khi , f (x) 1 (mod p) l v l. Nu p l c ca x + 1 th f (x) 5 (mod p), ko theo, p l

c ca 5 nn p = 5 v c ngay rng x = 4. Trng hp p l c ca x + 4 th f (x) 17 (mod p) nn p = 17 v d thy x = 13 l tha mn. Vy cc li gii ca bi ton l p = 5, x = 4 v p = 17, x = 13.

.c

Chng 9 thi olympic Irland


m
9.44. Tm tt c cc cp s nguyn (x, y) sao cho 1 + 1996x + 1998y = xy 19972 Do 1997 l s nguyn t, nn ta c: x 1998 = 1; 1997; 19972. Vy c 6 (x, y) = 1999, 19972 + 1996 , 1997, 19972 + 1996 , (3995, 3993) , (1, 1) 19972 + 1998, 1997 , 19972 + 1998, 195 (x 1998) (y 1996) = xy 1998y 1996x + 1996.1998 =

gi tr (x, y) tha mn l

F DE = . 2 Li gii: T cc t gic ni tip MDBF v MDCE ta c MDE = MCE v MDF = MBE do F DE = MCB + MBC = 2 M nm trn cung trn i qua B v C.
6

9.45. Cho ABC, M l im trong tam gic. Goi D,E,F ln lt l hnh chiu ca M xung BC, CA, AD. Tm tp hp tt c cc im M tha mn

.c

Li gii: Ta c:

hay BMC =

5 6

9.46. Tm tt c cc a thc P (x) sao cho i vi mi x ta c : (x 16) P (2x) = 16 (x 1) P (x) .

38

Nguyn Hu in, HKHTN H Ni

Li gii: Goi d = degP v a l h s ca x trong P (x) vi s m ln nht. Khi h s ca x m ln nht bn tri l 2d a phi bng 16a do d = 4 Do v phi lc ny chia ht cho (x 1), nhng trong trng hp v phi li chia ht cho (x 2), tng t l chia ht cho (x 4) v (x 8). Vy a thc

9.47. Cho a, b, c l cc s thc khng m sao cho a + b + c abc. Chng minh rng a2 + b2 + c2 abc. Li gii: Gi s phn chng rng vi a, b, c > 0 m a2 + b2 + c2 < abc do abc > a2 a < bc. Lm tng t ta cng c b < ca, c < ab. Do abc a2 +b2 +c2 ab+bc+ca. Theo bt ng thc AM-GM v ab+bc+ca > a+b+c suy ra abc > a + b + c. Tri vi gi thit. Vy bi ton c chng minh. 9.48. Cho tp hp S = {3, 5, 7, ...}. Vi mi x S ta t (x) l xc nh mt s nguyn duy nht sao cho: 2(x) < x < 2(x)+1

P (x) l bi ca (x 1)(x 2)(x 4)(x 8) l tt c cc a thc tha mn.

a b = 2(a)1 (b 3) + a

v a b 2m1 (2n+1 4) + 2m+1 1 = 2m+n 1. V vy (a b) = m + n 1 Nu 2p < c < 2p+1 th

(a b) c = 2m1 (b 3) + a c = 2m+n2 (c 3) + 2m1 (b 3) + a V a(b c) = a 2m1 (c 3) + b = 2m1 2n1 (c 3) + b 3 +a = (a b)c. 9.49. Cho t gic li ABCD c mt ng trn ni tip. Nu A=B= 2 , D = , BC = 1 3 2

a b = 2m1 (b 3) + a 2m1 (2n 2) + 2m + 1 = 2n+m1 + 1

b, Nu 2m < a < 2m+1 , 2n < b < 2n+1 th

Li gii: a, Hin nhin

a, Chng minh rng nu a, b S th a b S b, Chng minh rng nu a, b, c S th (a b) c = a (b c).

.c

i vi a, b S ta nh ngha php ton

thi olympic Irland Tm di AD

39

Li gii: Goi I l tm ng trn ni tip . Do ABC l tam gic u, BIC = 1050 , ICB = 150 , AID = 750, IDA = 450 nn AD = BI AD sin 150 sin 750 = = 2 sin 150 . BC AI sin 1050 sin 450

9.50. Gi A l tp con ca {0, 1, 2, ..., 1997} gm hn 1000 phn t. Chng minh rng A ch gm nhng ly tha ca 2 hoc hai phn t phn bit c tng l ly tha ca 2. Li gii: Gi s tp A khng tha mn bi ton. Khi A s bao gm hn na s nguyn t 51 ti 1997 m chng c chia thnh tng cp c tng l 2048 (V D : 51 + 1997 = 2048...). Tng t nh vy, A bao gm nhiu nht

gi thit A gm hn 1000 s nguyn t tp {0, 1, 2, ..., 1997}.

v hai s lin tip khc nhau 2 n v do tn cng theo th t l 1, 3, 5, 7, 9 do an+1 an 0 1 0 0 0 1 0 1 0 0 bn bn+1 0 1 0 1 0 cn = cn+1 0 0 1 0 1 dn dn+1 en+1 en 0 0 0 0 1 Gi A l ma trn vung trong biu thc . Ta tm gi tr ring ca ca A,

Li gii: t an , bn , cn , dn , en l s trong khai trin ca n, l nhng s l

b, Tt c cc s trong khai trin l s l. c, Hai phn t bt k lin nhau trong khai trin ca n hn km nhau 2 n v

9.51. Xc nh s t nhin n tha mn nhng iu kin sau: a, Khai trin thp phn ca n gm 1000 s

.c

na s nguyn t 14 ti 50, gm nhiu nht na s nguyn t 3 ti 13, v c th c s 0, do A c tng cng 937 + 18 + 5 + 1 = 997 s nguyn, tri vi

40

Nguyn Hu in, HKHTN H Ni

gi s Av = v vi v = (v1 , v2 , v3 , v4 , v5 ). Do v2 = v1 v3 = v2 v1 = (2 1) v1 v4 = v3 v2 = (3 2) v1

v5 = v4 v3 = (4 32 + 1) v1

v v4 = v5 , do 5 33 + = 3 2. Gii pt ny ta c = 0, = 1, = 3 tng ng ta c cc vect ring x1 , x2 , x3 , x4 , x5 l (1, 0, 1, 0, 1) , (1, 1, 0, 1, 1) , (1, 1, 0, 1, 1) , 1, 3, 2, 3, 1 2 3 1 2+ 3 x4 + x5 (1, 1, 1, 1, 1) = x1 3 6 6

V th kt qu ca bi ton l 8.3499 .

= 3499 , 2.3499 , 2.3499 , 2.3499 , 3499

(a1000 ,b1000 , c1000 , d1000 , e1000 ) = 2 3 999 2 + 3 =3 2 1, 3, 2, 3, 1 1, 3, 2, 3, 1 6 6

.c

V vy

Nguyn Hu in

OLYMPIC TON NM 1997-1998 49 THI V LI GII (Tp 5)


h
NH XUT BN GIO DC

.c

v n m a t h .c o m

Li ni u
th gi lnh lamdethi.sty ti bin son mt s ton thi Olympic, A m cc hc tr ca ti lm bi tp khi hc tp L TEX. ph v cc bn ham hc ton ti thu thp v gom li thnh cc sch in t, cc bn c th tham kho. Mi tp ti s gom khong 51 bi vi li gii. chnh xc vy mong bn c t ngm ngh v tm hiu ly. Nhng y l ngun ti liu ting Vit v ch ny, ti c xem qua v ngi dch l chuyn v ngnh Ton ph thng. Bn c th tham kho li trong [1]. Rt nhiu on v mi hc TeX nn cu trc v b tr cn xu, ti khng c thi gian sa li, mong cc bn thng cm.

H Ni, ngy 2 thng 1 nm 2010 Nguyn Hu in

51 89/176-05 GD-05

.c

Rt nhiu bi ton dch khng c chun, nhiu im khng hon ton

M s: 8I092M5

Mc lc
Li ni u . . . . . . . . . . . . . . . . . . . . . . . . . . . . . . . . . . . . . . . . . . . . . . . . . . . Mc lc. . . . . . . . . . . . . . . . . . . . . . . . . . . . . . . . . . . . . . . . . . . . . . . . . . . . . . . 3 4 5 8 15 18 22 25 30 38

Chng 8. thi olympic Poland . . . . . . . . . . . . . . . . . . . . . . . . . . . . . . . .

Chng 7. thi olympic Korean . . . . . . . . . . . . . . . . . . . . . . . . . . . . . . .

Chng 6. thi olympic Japan . . . . . . . . . . . . . . . . . . . . . . . . . . . . . . . . .

Chng 5. thi olympic Italy . . . . . . . . . . . . . . . . . . . . . . . . . . . . . . . . . .

Chng 4. thi olympic Ireland . . . . . . . . . . . . . . . . . . . . . . . . . . . . . . .

.c

Chng 3. thi olympic Iran . . . . . . . . . . . . . . . . . . . . . . . . . . . . . . . . . . .

Chng 2. thi olympic Hungary . . . . . . . . . . . . . . . . . . . . . . . . . . . . . . .

Chng 1. thi olympic Hy Lp . . . . . . . . . . . . . . . . . . . . . . . . . . . . . . . .

Chng 1 thi olympic Hy Lp


m
1.1. Cho P l mt im nm bn trong hay trn 1 cnh bt k ca hnh vung

f ( P) = ABP + BCP + CDP + DAP

C B t cc nh ca hnh vung tng ng vi cc gi tr 1, i, -1, -i trong mt phng v coi P l s phc z. Khi f(P) l argument ca s phc z tho mn z4 1 z1 zi z+1 z+1 = i + 1 1 i i + 1 1 + i 4 z4 1 Khi | P| 1, chy trn min phng c gii hn bi ng trn bn 4 knh 1/4, tm c to -1/4. Do gi tr ln nht ca gc t c ti 1 im trn bin ca hnh trn trn, iu xy ra khi P nm trn cnh ca hnh vung. Do vai tr ca cc cnh l nh nhau, khng mt tng qut ta c th gi s cnh l AB.

Li gii:

.c

ABCD. Hy xc nh gi tri ln nht v gi tr nh nht c th c ca hm s

Nguyn Hu in, HKHTN H Ni

Li gii: t d=gcd(x,y), t x=dx1 , y=dy1 Khi phng trnh cho tng ng vi 1997(13)y2 +1997(1996)x2 =d2 zy2 x2 1 1 1 1 Khi x1 v y1 nguyn t cng nhau, ta phi c x2 |1997 13, 1

y2 |1997 1996 1 D dng kim tra c rng 1997 khng phi s chnh phng v r rng n nguyn t cng nhau vi 13 v 1996. Hn na 1996 = 22 .499, v cng d dng kim tra c rng 499 khng phi s chnh phng.

n ; BCP gim t n 0; Khi P chy t A n B th CDP gim t 2 4 4 Hai gc cn li nhn cc gi tr l v 0. 2 5 3 Vy ta c gi tr ln nht v nh nht ca f ( P) ln lt l v 4 4 1.2. Cho hm f : (0; ) R tho mn cc iu kin sau: (a) f tng nghim ngt 1 vi mi x>0 (b) f(x)> x 1 (c) f(x)f(f(x)+ )=1 vi mi x>0 x Tnh f(1). 1 1 Li gii: t k=f(x)+ . V k>0 nn f(k)f(f(k)+ )=1 x k 1 1 1 + Mt khc f(x)f(k)=1. Do f(x)=f(f(k)+ )=f( ) 1 k f (x) f (x) + x 1 1 + Do f tng nghim ngt nn ta c x= 1 f (x) f (x) + x 1 5 . Gii ra ta thu c f(x)= 2x 1 5 D dng kim tra c rng ch c tho mn cc yu cu ca bi. 2x 1 5 Do f(1)= 2 1.3. Tm tt c cc s nguyn tho mn phng trnh sau:

z 13 1996 + 2 = 2 x y 1997

.c

thi olympic Hy Lp Khi (x1 , y1 ) = (1, 1) hoc (1,2) Bi ton c chia thnh 2 trng hp: * Trng hp 1: (x1 , y1 ) = (1, 1). Khi

d2 z = (13 + 1996)1997 = 1997.72.41 Khi 1997 nguyn t cng nhau vi 7 v 41 th d=1,7. T ta c kt qu ln lt l: (x,y,z)=(1,1,4011973), (7,7,81877) * Trng hp 2: (x1 , y1 ) = (1, 2).Khi d2 z = (13 + 499)1997 = 1997.29 Do d=1,2,4,8,16. Ta li c cc kt qu ln lt l: (x,y,z)=(1,2,1022464),(2,4,255616),(4,8,63904), (8,16,15976), (16,32,3994) l cc kt qu thu c. 1.4. Cho P l mt a thc vi cc h s nguyn c 13 nghim nguyn phn bit. Hy ch ra rng nu n Z khng phi l nghim ca P th | P(n)| 7(6!)2 . Hy cho 1 v d khi du bng xy ra. Li gii: Phn tch a thc vi cc h s nguyn thnh tch ca cc a thc cng c h s nguyn vi bc nh hn th P(x) c th vit di dng ( x r1 )( x r2 )...( x r13 )Q( x )

|(1)(1)(2)(2)...(6)(6)(7)(1)| = 7(6!)2 . T ta c iu phi chng minh. Mt v d khi du bng t c l khi x = 0 v P(x)=(x+1)(x-1)(x+2)(x-2)...(x+7)

phn bit vi 1 s nguyn khc. R rng gi tr tuyt i nh nht ca kt qu trn l

trong rs l 1 trong 13 nghim phn bit ca a thc . Do vi tt mi s nguyn x, P(x) c gi tr bng tch ca 13 s nguyn

.c

Chng 2 thi olympic Hungary


2.5. Mi thnh vin trong hi ng xp hng cc ng vin A, B, C theo th t. iu ch ra rng phn ln cc th hng hi ng A cao hn nhiu so vi B v cng c th l phn ln cc th hng B cao hn nhiu so vi C. C phi m theo A cao hn C. Li gii: Khng. Gi s gi hi ng c ba thnh vin, mt trong nhng

2.6. Cho php a, b, c c cc bn, m a , mb , mc l cc cao, v d a , db , dc l cc khong cch t nh vo trong mt trng tm tam gic. Chng minh rng. m a d a + mb db + mc dc = a2 + b2 + c 2 2

ba c hai thch A n B, v th nht v th hai thch c hai B to C, nhng ch l ngi u tin thch A n C.

ngi xp hng A > B > C, mt trong nhng ngi xp hng B > C > A, v l mt trong nhng ngi xp hng C > A > B. Sau , th nht v th

.c

thi olympic Hungary

Li gii: Cho D, E, F l chn ca chiu cao t A, B, C tng ng, v cho H l trc tm ca hnh tam gic ABC, Sau hnh tam gic ACD l ging vi hnh tam gic AHE. Vy m a d a = AD AH = CE AE = AE b. Tng t hnh tam gic ABD l ging vi hnh tam gic AHF. Vy m a d a = AD AB = AF AE = AB c. Do ma da = Tng t mb db = v mc dc = Do BF c + BD a 2 AE b + AF c 2

m a d a + mb db + mc dc 1 = ( AE b + AF c + BF c + BD a + CD a + CE b) 2 1 = (( BD + CD ) a + (CE + AE) b + ( AF + BF) c) 2

2.7. Cho R l bn knh hnh tam gic ABC v G, H l trng tm v trc tm tng ng. Cho F l trung im ca GH. AF2 + BF2 + CF2 = 3R2

a2 + b2 + c 2 = 2

CD a + CE b 2

.c

10

Nguyn Hu in, HKHTN H Ni

Li gii: Chng ta s dng Vector vi gc ta ti tm hnh tam gic ABC. Sau chng ta c cng thc H = A + B + CvG = H/3. Vy F =

(G + H )/2 = 2H/3v2( A + B + C) = 3F. Do AF2 + BF2 + CF2


=(A-F).(A-F) + (B-F).(B-F) + (C-F).(C-F)

= A A + B B + C C 2( A + B + C) F + 3F F = 3R2 F (2( A + B + C) 3F) = 3R2


2.8. Mt hp cha 4 qu bng trng v 4 qu bng , chng ta cn v t ci hp theo mt s th t m khng cn thay th. Trc khi v chng ta cn on mu ca qu bng s v. Con s c mong i ca cc d on chnh xc l bao nhiu?

i, j 0, aij biu th con s mong i ca cc d on chnh xc khi c i qu bng trng v j qu bng . Gi s i>j>=1. Sau d on ca chng ta l chnh xc vi xc sut i/(i+j), a ra con s mong mun ca cc d on chnh xc

ca

v sai vi xc sut j/(i+j), a ra con s mong mun ca ai,j1 Vy aij = if i>j Cng vy, chng ta c aij = a ji i i (1 + ai 1,j ) + ai,j1 i+j i+j

1 + ai 1,j

.c

Li gii: Con s c ch i ca cc d on chnh xc l 373/70. Cho

thi olympic Hungary cho i, j 0 Nu i=j1 sau chng ta on vi xc sut 1/2 v aij = Nh ai,i 1 = ai 1,i . Cui cng, iu kin bt u l: 1 1 1 (1 + ai 1,j ) + ai,i 1 = + ai,i 1 2 2 2

11

Li gii: cc gii php l: (-2,6), (-3,4), (-4,-4), (-5,-6) Cho P( x ) = x3 + ( x + 1)3 + ( x + 2)3 + ... + ( x + 7)3 = 8x3 + 84x2 + 420x + 784. Nu x0 th:

(2x + 7)3 = 8x3 + 84x2 + 294x + 343 < P( x ) < 8x3 + 120x2 + 600x + 1000 = (2x + 10)3

x3 + ( x + 1)3 + ( x + 2)3 + ... + ( x + 7)3 = y3

2.9. Tm tt c cc gii php cho nhng s nguyn ca phng trnh

a4,4 = 373/70

Chng ta c th s dng nhng phng trnh ny cho vic tnh ton

.c

i0

vi

ai0 = a0i = i

12

Nguyn Hu in, HKHTN H Ni

Vy 2x+7 < y < 2x100: do y l 2x + 8 hoc 2x + 9 nhng c hai phng trnh P( x ) (2x + 8)3 = 122 + 36x + 272 = 0 P( x ) (2x + 9)3 = 24x2 + 66x + 55 = 0 c bt k cn nguyn. Do vy nn ko c gii php bo vi x0 Tip theo, ch rng P tha mn P (-x-7) = -P(x), vy (x.y) l 1 gii php nu (-x-7,-y) l mt gii php. Do vy khng c gii php no vi

khng mt ly tha 3. v

vy (-2,6) v (-3,4) v ch cc gii php vi

Chng ta c P(-1) = 440

Do (-4,-4) v (-5,-6) ch l gii php vi

Vy p n ch c th l (-2,6), (-3,4), (-4,-4) v (-5,-6).

P(2) = 216 = 63 P(3) = 64 = 43

Cho

3 x 1

3 x 1

6 x 4

6 x 1

.c

Do vy (x.y) l mt gii php. Chng ta phi c

x 7

thi olympic Hungary

13

2.10. Chng ta c 1997 s nguyn dng khng trng nhau, bt k 10 trong s c cng t nht chung. Tm s ln nht c th ca cc s nguyn t cng nhau gia chng. Li gii: s ln nht ca tng i s nguyn t trong tp hp ny l 9. Trc tin, gi s c 10 s nguyn t.. n1 , n2 , ....n10 Sau t nht ca 10 thnh vin ca tp hp ny l lcm(n1 , n2 , ....n10 ) = (n1 n2 ...n10 ) C bit, cho bt k N khc trong tp hp ny

n1 c quan h vi cho chi cho N. Tng t n1 chia cho N vi mi nj

nh vy ni c quan h nguyn t. n1 n2 ...n10

2 j 10

i {2......10}

Nh

.c

n1

l chia ht cho

lcm( Nn2 , ....n10 ) (n1 n2 ...n10 )

14 chia cho N. Nhng

Nguyn Hu in, HKHTN H Ni

N lcm( N.n2 ....n10) = n( 1)n( 2)...n( 10) Vy chng ta phi c N = n( 1)n( 2)....n( 10) T y n lu gi mi thnh phn ca tp hp ca chng ta hn n( 1).......n( 10) Tp hp ca chng ta ch c th cha 11 thnh phn, mt s mu thun. By gi chng ta khi to mt v d m c 9 s nguyn t. Cho p ( n) biu th s nguyn t th n v cho

khi vi

R rng bt k 2 thnh phn ca n( 1).........n( 9) l cp nguyn t.

n( 9) = p( 9) p( 10)......p( 1988)

n( i ) = p ( i ) 1i8

.c

S=

p1 p2 ...p1988 |1 j 1988 pj

m
{n1 , n2 ....n9 }

Chng 3 thi olympic Iran


3.11. Gi s w1 , ..., wk l nhng s thc phn bit vi tng khc khng. CMR : tn ti cc s nguyn n1 , ..., nk sao cho : n1 w1 + ... + nk wk > 0 v mt s hon v ca {1, .. ., k} khng ng nht bng nhau. Ta c :

: bi a (i ) bi ai iu ny l hin nhin v nu i < j nhng (i ) > ( j) th thay bi nhng s hp thnh vi s chuyn v ca i v j tng ln th tng bng a j ai b j bi Gi s rng w1 < ... < wk v s = | wi | t = min {wi +1 wi } v chn 1 s t nhin N = Ta t (n1 , n2 , ..., nk ) = ( N, 2N, ..., kN ) + p(1, ...., 1)

Nu a1 < ... < an , b1 < ... < bn l nhng s thc, = min { ai +1 ai }, = min { bi +1 bi } th vi 1 vi hon v khng thng thng ca {1, ..., n}

Li gii: u tin, ta sp xp li bt ng thc :

n1 w1(1) + ... + nk w (k) < 0

.c

y p l s nguyn duy nht sao cho ni wi (0, s] y l nh l bao hm rng = 1

ni w (i) ni wi N s N < 0

16 .

Nguyn Hu in, HKHTN H Ni

3.12. Gi s im P di ng dc theo cung BC ca ng trn ngoi tip ABC, v cho I1 , I2 tng ng l tm ng trn ni tip PAB, PAC. CMR : a) ng trn ngoi tip PI1 I2 i qua mt im c nh. b) ng trn ng knh I1 I2 i qua mt im c nh. c) Trung im ca on I1 I2 nm trn mt ng trn c nh. Li gii: Cho B1 , C1 l im gia ca cc cung AC, AB. Do I1 , I2 l tm ng trn ni tip ca cc tam gic ABP, ACP, ta c : C1 A = C1 B = C1 I1 , B1 A = B1 C = B1 I2 . Gi I l tm ng trn ngoi tip ABC v Q l giao im th 2 ca ng trn ngoi tip ABC v PI1 I2 . Do C1 I1 v B1 I2 i qua P nn QI1 C1 v
1 I1 1A 1 QI2 B1 ng dng, vy : QC1 = C1 I2 = C1 A khng i. QB B B Do Q l giao ca ng trn ngoi tip ABC vi ng trn Apollonius c nh, nn Q c nh v phn a) c chng minh. ( B+C) T : I1 QI2 = I1 PI2 = C1 PB1 = 2 Cc tam gic QI1 I2 vi Q = P u ng dng vi nhau. V th nu M l trung im ca I1 I2 th cc tam gic QI1 M cng u ng dng.

Cui cng ta tnh c gc I1 I I2 = . V th ng trn ng knh I1 I2 i 2 qua I c nh v phn (b) c chng minh. 3.13. Gi s f : R+ R+ l hm lin tc, gim sao cho x, y R+ , f ( x + y ) + f ( f ( x ) + f (y)) = f ( f ( x + f (y))) + f (y + f ( x )) CMR : f ( f ( x )) = x. Li gii: t y = x, ta c: f (2x ) + f (2 f ( x )) = f (2 f ( x + f ( x ))) (1)

Do C1 I1 = C1 A khng i, I1 chuyn ng trn mt cung ca ng trn c nh nn M nm trn mt ng trn c nh v (c) c chng minh.

Nu k = QM , = MQI1 , ngha l M l nh ca I1 qua qua cc php di hnh QI1 tm Q vi gc v t s k.

.c

thi olympic Iran Thay x bi f ( x ) , ta c : f (2 f ( x )) + f (2 f ( f ( x ))) = f (2 f ( f ( x ) + f ( f ( x )))) (2) Ly (2) tr (1) , ta c :

17

f (2 f ( f ( x ))) f (2x) = f (2 f ( f ( x ) + f ( f ( x )))) f (2 f ( x + f ( x ))) Nu f ( f ( x )) > x th v tri ca phng trnh l m, do : f ( f ( x ) + f ( f ( x ))) > f ( x + f ( x )) V f ( x ) + f ( f ( x )) < x + f ( x ), mu thun. iu mu thun tng t cng xy ra nu f ( f ( x )) < x. Vy : f ( f ( x )) = x. Aii = 1 vi mi i. Hy ch ra rng tn ti mt tp con ca cc dy m tng tt c thnh phn ca vct l l. Gi s ngc li , tn ti mt vct (v1 , ..., vn ) sao cho i vi wi = 0 vi mt vi dy (w1 , ..., wn ) nhng vi = 0 (Tt c cc s y l chia ht cho 2) . Cng trn tt c cc dy, ta c : 3.14. Cho A l mt ma trn gm cc s 0 v 1 i xng Aij = A ji , i, j sao cho

vi Aij v j = 0
j i i i

Do ma trn l i xng, iu ny c quy v v2 Aii = 0 hoc vi = 0 i

(v1 {0, 1}), mu thun .

Li gii:

.c

Chng 4 thi olympic Ireland


m
4.15. Tm tt c cc cp s nguyn ( x, y) sao cho 1 + 1996x + 1998y = xy Li gii: Ta c: ( x 1998) (y 1996) = xy 1998y 1996x + 2 1996.1998 = 1997 Do 1997 l s nguyn t, nn ta c: x 1998 = 1; 1997; 19972. Vy c 6

gi tr ( x, y) tha mn l

4.16. Cho ABC, M l im trong tam gic. Goi D,E,F ln lt l hnh chiu ca M xung BC, CA, AD. Tm tp hp tt c cc im M tha mn FDE = . 2 Li gii: T cc t gic ni tip MDBF v MDCE ta c MDE = MCE v MDF = MBE do FDE = MCB + MBC = hay BMC = 5 M 2 6 6 4.17. Tm tt c cc a thc P( x ) sao cho i vi mi x ta c :

nm trn cung trn i qua B v C.

( x 16) P (2x ) = 16 ( x 1) P ( x ) .

(3995, 3993) , (1, 1) 19972 + 1998, 1997 , 19972 + 1998, 195

( x, y) = 1999, 19972 + 1996 , 1997, 19972 + 1996 ,

.c

thi olympic Ireland

19

Li gii: Goi d = degP v a l h s ca x trong P( x ) vi s m ln nht. Khi h s ca x m ln nht bn tri l 2d a phi bng 16a do d = 4 Do v phi lc ny chia ht cho ( x 1), nhng trong trng hp v phi li chia ht cho ( x 2), tng t l chia ht cho ( x 4) v ( x 8). Vy a

thc P( x ) l bi ca ( x 1)( x 2)( x 4)( x 8) l tt c cc a thc tha mn. rng a2 + b2 + c2 abc.

4.18. Cho a, b, c l cc s thc khng m sao cho a + b + c abc. Chng minh

a b = 2 m 1 ( b 3 ) + a 2 m 1 ( 2 n 2) + 2 m + 1 = 2 n + m 1 + 1 v a b 2m1 2n+1 4 + 2m+1 1 ( a b) = m + n 1 Nu 2 p < c < 2 p+1 th

Li gii: a, Hin nhin b, Nu 2m < a < 2m+1 , 2n < b < 2n+1 th

b, Chng minh rng nu a, b, c S th ( a b) c = a (b c).

a, Chng minh rng nu a, b S th a b S

a b = 2(a)1 (b 3) + a

.c

4.19. Cho tp hp S = {3, 5, 7, ...}. Vi mi x S ta t ( x ) l xc nh mt s nguyn duy nht sao cho: 2(x) < x < 2(x)+1 i vi a, b S ta nh ngha php ton

m
=

abc a2 + b2 + c2 ab + bc + ca. Theo bt ng thc AM-GM v ab + bc + ca > a + b + c suy ra abc > a + b + c. Tri vi gi thit. Vy bi ton c chng minh.

Li gii: Gi s phn chng rng vi a, b, c > 0 m a2 + b2 + c2 < abc do abc > a2 a < bc. Lm tng t ta cng c b < ca, c < ab. Do

2m+n 1. V vy

( a b ) c = 2m 1 ( b 3) + a c = 2m + n 2 ( c 3) + 2m 1 ( b 3) + a

a ( b c) = a 2m1 (c 3) + b = 2m1 2n1 (c 3) + b 3 + a = (a b) c.

20

Nguyn Hu in, HKHTN H Ni

4.20. Cho t gic li ABCD c mt ng trn ni tip. Nu A=B= Tm di AD Li gii: Goi I l tm ng trn ni tip . Do ABC l tam gic u, BIC = 1050 , ICB = 150 , AID = 750 , IDA = 450 nn AD = 2 , D = , BC = 1 3 2

sin 150 sin 750 BI AD = = 2 sin 150 . 0 sin 450 BC AI sin 105

4.21. Gi A l tp con ca {0, 1, 2, ..., 1997} gm hn 1000 phn t. Chng minh rng A ch gm nhng ly tha ca 2 hoc hai phn t phn bit c tng l ly Li gii: Gi s tp A khng tha mn bi ton. Khi A s bao gm hn na s nguyn t 51 ti 1997 m chng c chia thnh tng cp c tng l

Li gii: t an , bn , cn , dn , en l s trong khai trin ca n, l nhng s l v hai s lin tip khc nhau 2 n v do tn cng theo th t l 1, 3, 5, 7, 9 do 0 1 0 1 0 1 0 1 0 0 0 1 0 0 0 a a n n +1 0 0 bn bn + 1 1 0 c n = c n +1 0 1 d n d n +1 e n +1 en 0 1 0 0

b, Tt c cc s trong khai trin l s l. c, Hai phn t bt k lin nhau trong khai trin ca n hn km nhau 2 n v

4.22. Xc nh s t nhin n tha mn nhng iu kin sau: a, Khai trin thp phn ca n gm 1000 s

th c s 0, do A c tng cng 937 + 18 + 5 + 1 = 997 s nguyn, tri vi gi thit A gm hn 1000 s nguyn t tp {0, 1, 2, ..., 1997}.

2048 (VD : 51 + 1997 = 2048...). Tng t nh vy, A bao gm nhiu nht na s nguyn t 14 ti 50, gm nhiu nht na s nguyn t 3 ti 13, v c

.c

tha ca 2.

thi olympic Ireland

21

Gi A l ma trn vung trong biu thc . Ta tm gi tr ring ca ca A, gi s Av = v vi v = (v1 , v2 , v3 , v4 , v5 ). Do v2 = v1 v3 = v2 v1 = 2 1 v1 v4 = v3 v2 = 3 2 v1

v5 = v4 v3 = 4 32 + 1 v1 v v4 = v5 , do 5 33 + = 3 2. Gii pt ny ta c = 0, = 1, = 3 tng ng ta c cc vect ring x1 , x2 , x3 , x4 , x5 l

(1, 0, 1, 0, 1) , (1, 1, 0, 1, 1) , (1, 1, 0, 1, 1) , 1, 3, 2, 3, 1 2 3 1 2+ 3 x4 + x5 (1, 1, 1, 1, 1) = x1 3 6 6

V th kt qu ca bi ton l 8.3499 .

= 3499 , 2.3499 , 2.3499 , 2.3499 , 3499

(a1000 ,b1000 , c1000 , d1000 , e1000 ) = 999 2 + 2 3 3 1, 3, 2, 3, 1 1, 3, 2, 3, 1 =3 2 6 6

.c

V vy

Chng 5 thi olympic Italy


5.23. Cho mt di giy hnh ch nht c chiu rng 3 cm, chiu di v tn. Gp di giy li ch bng mt np gp.Hi phn di giy b ph bi vic gp c th c Li gii: Phn di giy b ph l mt tam gic.K hiu ba nh ca tam

SABC = SABA SACA 1 1 = .3.(3 cot x ) .3.(3 cot 2x ) 2 2 9 = (cot x cot 2x ) 2 9 = csc 2x 2 Gi tr nh nht ca csc2x ch c th l 1 khi x = nht l
9 2

ACA = 2x. Ta c:

trng hp : Trng hp 1 : Gi s 0 < x

t
4,

gic l A, B, C trong AB l np gp v gc BAC nhn. H cc ng cao AA , BB , CC ca ABC. Ch BAB = BAC. t BAB = x. Ta xt hai th th C nm gia A v B, ABA = x v

.c

din tch nh nht l bao nhiu ?

4.

Do din tch nh

5.24. Cho f l mt hm gi tr thc sao cho vi mi s thc x ta c (a) f (10 + x ) = f (10 x )

thi olympic Italy

23

(b) f (20 + x ) = f (20 x ) Chng minh rng f l hm l ( f ( x ) = f ( x )) v tun hon (tc l tn ti T > 0 sao cho f ( x + T ) = f ( x )). Li gii: Chn x = n 10, t (a) ta c f (n) = f (20 n)

Chn x = n t (b) ta c f (20 n) = f (20 + n) f (n) = f (n + 20) T suy ra . Do f (n + 40) = f (n)). V vy f (n + 20) = f (n + 40) f tun hon v c chu k l T = 40 Ta cng c f (n) = f (20 + n) = f (20 n) = f (n). Vy f l hm l.

5.25. Gc phn t th nht ca mt phng to c chia thnh cc hnh vung dn v bi cc ng li.C th t mu cc hnh vung n v tho mn cc iu

| D | + 1 hnh vung gn trc nht. Xt ng y = x + D vi D 0. Dc ng ny hnh vung u tin c t nm trn ct th 1 v dng th D + 1. Hnh vung cui cng c t mu nm trn ct th D + 1 v dng th 2D + 1. Do cc hnh vung nm bn tay phi ca hnh vung ny ( pha trn ng y = x) l phn m cc ng cho c cc hnh vung c t t hn, th th khng mt hnh vung no trong s c t c. Nu ta k hiu (i, j) l hnh vung dng th i v ct th j th hnh vung (i, j) c t khi v ch khi:

Li gii: C th thc hin nh sau: Trn mi ng y = x + D, ta t mu

cc hnh vung c t mu.

.c

cc trc to th cha nhiu hnh vung n v c t hn cc hnh vung dn v khng c t. (b)Mi ng song song vi ng phn gic ca gc phn t th nht ch i qua

kin sau? (a)Vi mi hnh vung ln c mt nh dt ti gc, v cc cnh song song vi vi

j i, i D + 1 i ( j i + 1) i hoc j
i +1 2 .

j+1 2

Tng cc hnh vung c t trong n.n hnh vung l Cn = 2(


k=1 n

k+1 )1 2

24
1 Nu n chn, ta c Cn = 2 n2 + n 1

Nguyn Hu in, HKHTN H Ni

1 Nu n l , th Cn = 1 n2 + n 2 2 1 Do Cn 2 n2 vi mi n s tho mn iu kin bi.

5.26. Cho t din ABCD.Gi a l di ca AB v S l din tch hnh chiu ca t din ln mt phng vung gc vi AB.Hy xc nh th tch ca t din ABCD theo S v a. Li gii: Ta k hiu cc to A = (0, 0, 0), B = (0, 0, n), C = (0, b, c), D =

Li gii: Gi s n c nhiu hn hoc bng 3 ch s. Gi AB l hai ch s cui th th s c hai ch s cui theo th t l BA cng thuc An . Vy hiu ca hai s trn l : | BA AB| = |10B A 10A + b| = 9 | A B| 81. Nu dn l c ca hai s trn th dn l c ca 81 vy dn 81

nht ca dn = 90;nu n = 10 th An = {10}. Suy ra dn = 90 > 81 Vy gi tr ln nht c th ca dn l 90.

Nu n gm hai ch s , c hai ch s u khc 0 th lp lun nh trn. Nu n gm hai ch s m mt trong hai ch s l 0 th An ch cha n. ( Vd n = 90 th An = {09, 90} = {90}) trong trng hp ny gi tr ln

V d n = 47 th An {125, 152, 251, 215, 521, 512}.

nhau. Vi mi n X, t An l tp hp tt c cc s m cc ch s ca n l mt hon v ca cc ch s ca n.

5.27. Cho X l tp hp tt c cc ss t nhin m cc ch s ca n i mt khc

.c

(i, j, k). Khi mt phng z = 0 vung gc vi AB, v hnh chiu ca t din ln mt phng ny l mt tam gic c nh A = B = (0, 0, 0), C = (0, b, 0), D = (i, j, 0). Tam gic ny c y l b v chiu cao tng ng l i. i Vy S = b 2 v a = AB = n. tm th tch , ta xt t din nh l hnh chp c y l tam gic ABC.Khi mt phng y c phng trnh x = 0 v chiu cao h t d c di l i. a Din tch ca tam gic ABC l b n . Vy th tch ca t din l : b in = S 3 . 2 6

Gi dn l CLN ca tt c cc s trong An . Tm gi tr ln nht c th ca dn .

{47, 74} , n

125 th An

Chng 6 thi olympic Japan


6.28. Chng minh rng bt k 9 im bt k nm trong mt ng trn ng knh 5, tn ti hai im c khong cch nh hn 2.

Chia hnh trn thnh 9 phn: mt phn l hnh trn bn knh 1 ng tm vi ng trn cho v 8 hnh qut bng nhau l giao ca phn cn li vi ng trn. Sau kim tra c rnng hai im trong mi phn c khong cch nhiu nht l 2. 6.29. Cho a, b, c l cc s dng. Chng minh bt ng thc sau:

( c + a b )2 ( a + b c )2 3 ( b + c a )2 + + 5 ( b + c )2 + a2 ( c + a )2 + b 2 ( a + b )2 + c 2

a
O

.c

Li gii:

26 v xc nh khi no du bng xy ra. Li gii: u tin, rt gn a2

Nguyn Hu in, HKHTN H Ni

2ba + 2bc 2ca + 2cb 12 2ab + 2ac + 2 + 2 2 + c2 + 2bc 2 + c2 + 2ac 2 + c2 + 2ab 5 +b a +b a +b

t s = a2 + b2 + c2 . Sau quy ng kh mu s ta c

5s2 (ab + bc + ca) + 10s(a2 bc + ab2 c + abc2 ) + 20(a3 b2 c + ab3 c2 + a2 bc3 )

6s3 + 6s2 (ab + bc + ca) + 12s(a2 bc + ab2 c + abc2 ) + 48a2 b2 c2


n gin hai v 6s3 + s2 (ab + bc + ca) + 2s(a2 bc + ab2 c + abc2 ) + 48a2 b2 c2

+(3b6 + 2b5 c 2b4 c2 + 3b4 ca + 2b3 c3 12b3 c2 a + 12b2 c2 a2 )+

p dng bt ng thc Schur, ta c:

(4a4 bc 8a3 b2 c + 4a2 b2 c2 ) + (4b4 ca 8b3 c2 a + 4b2 c2 a2 ) + (4c4 ab 8c3 a2 b + 4c2 a2 b2 ) 0,


chng minh bt ng thc (*), ta cn chng minh:

(3a6 + 2a5 b 2a4 b2 a4 bc + 2a3 b3 4a3 b2 c)+ +(3b6 + 2b5 c 2b4 c2 b4 ca + 2b3 c3 4b3 c2 a)+ +(3c6 + 2c5 a 2c4 a2 c4 ab + 2c3 a3 4c3 a2 b) 0.

+(3c6 + 2c5 a 2c4 a2 + 3c4 ab + 2c3 a3 12c3 a2 b + 12b2 a2 b2 0. ()

(3a6 + 2a5 b 2a4 b2 + 3a4 bc + 2a3 b3 12a3 b2 c + 12a2 b2 c2 )+

Thay s v khai trin biu thc ca s

.c

10s(a2 bc + ab2 c + abc2 ) + 20(a3 b2 c + ab3 c2 + a2 bc3 ).

thi olympic Japan

27

Ta c th chng minh bt ng thc trn bng bn biu thc khng m bi bt ng thc AM-GM: 02 02 02 02 2a6 + b6 3 a4 b2 a,b,c

4a6 + b6 + c6 a4 bc 3 a,b,c 2a3 b3 + c3 b3 a3 b2 c 3 a,b,c

2a5 b + a5 c + ab5 + ac5 a3 b2 c. 6 a,b,c

6.30. Cho G l th 9 nh. Gi s rng vi bt k 5 nh ca G u tn ti t

S cnh nh nht l 9, t c bi 3 chu trnh ri nhau. Tht vy, gi an l s cnh nh nht ca th n nh tha mn iu kin

Li gii:

.c

nht hai cnh c im u v im cui thuc vo 5 im . Hi rng s cnh nh nht c th c ca G l bao nhiu?

Du bng xy ra khai v ch khi a = b = c.

28 bi ra. Ta s chng minh rng an+1

Nguyn Hu in, HKHTN H Ni

n+1 an . Vi mi th n nh gi li l n1 s cnh ca th nhn c bng cch b i nh th i v tt c cc cnh

gn vi nh th i. (Ta c li an , mt khc l1 + l2 + ... + ln+1 = (n 1)an+1 . V mi cnh c m cho mi cnh khc tr hai im u cui ca n). T , a5 = 2, ta nhn c a6 3, a7 5, a8 7, a9 9. 6.31. Cho A, B, C, D l bn im khng ng phng. Gi s rng AX + BX + CX + DX t gi tr nh nht ti X = X0 khc A, B, C, D. Chng minh rng AX0 B = CX0 D. Li gii: D

h
X = X0 C B Gi s A, B, C, D v P c cc ta ( x1 , y1 , z1 ), ..., ( x4 , y4 , z4 ) v ( x, y, z). Ta c hm s f ( P) =

( x x i )2 + ( y y i )2 + ( z zi )2

. t gi tr nh nht, ba o hm ring ca n phi bng khng, nhng PA c ba hm ta ca ua + ub + uc + ud , y ua l vct n v || P A|| v tng t. Do tng ny bng khng, v ua .ub = uc .ud ti im P = X0 , t ta c iu phi chng minh.

.c

thi olympic Japan

29

6.32. Cho n l mt s nguyn dng. CMR: c th gn cho mi nh ca mt a gic 2n nh, mt trong cc ch ci A hoc B sao cho cc dy n ch ci nhn c bng cch c bt u t mt nh no theo ngc chiu kim ng h, l lun khc nhau. Li gii: Xt mt th c hng nh sau: mi nh ca th l mt dy c di n 1, hai nh l k nhau nu n 2 ch ci cui ca nh ny trng vi n 2 ch ci u ca nh kia. (Ch l: y l mt th c hai

vng). Mi nh ca th c mt cnh i vo v mt cnh i ra, v th tn ti mt ng i c hng i qua mi cnh ng mt ln. Ta c th c mt chu trnh cn thit bng cch bt u t mt nh bt k, vit ra dy ch tng ng vi n, sau vit thm vo ch cui cng ca mi dy, ta s gp ht cc dy trn ng i .

.c

Chng 7 thi olympic Korean


7.33. Chng minh rng vi bn im bt k trong mt ng trn n v th tn ti hai im m khong cch gia chng khng vt qu 2.

Ta c Pi OPi +1 2 on Pi Pi +1 nm trong tam gic OQi Qi +1 v vy ta c: 7.34. Cho hm s: f : N N tha mn hai iu kin:

Ta c t gic li Q1 Q2 Q3 Q4 , vi { Qi } l giao im ca {OPi } vi ng trn. Khi : P1 OP2 + P2 OP3 + P3 OP4 + P4 OP1 2

Nu khng c im no trong bn im trng tm ng trn th ta k hiu cc im ln lt l P1 , P2 , P3 , P4

minh. - Trng hp 2:

.c

Li gii: - Trng hp 1: Nu mt trong 4 im l tm ng trn th ta c ngay iu phi chng

Pi Pi +1 max (OQi , Qi Qi +1 , Qi +1O) = max (1, 2 sin OQi Qi +1 ) a. n N, f(n + f(n)) = f(n) b. n0 N, f(n0 ) = 1

Li gii: Trc ht ch rng nu n N f(n) = 1, sau f(n + 1) = f(n + f(n)) = f(n) = 1

Chng minh rng: f(n) = 1, n N

thi olympic Korean Cho f(n0 ) = 1,f(n) = 1, n n0 t S = {n N | f (n) = 1 }

31

iu ny mu thun vi N = max S Vy S = v f(n) = 1, n N 7.35. Biu th tng


k=1 n

Nu S = , gi N = maxS. Ta c f(N + f(N)) = f(N) = 1 N + f(N) S, N + f(N) > N

k theo cc s hng ca n v a =

Li gii: Ta s chng minh kt qu l:

( n + 1)

a(a + 1)(2a + 1) 6

Do :
k=1

vi k {1, ..., n} m k j2 , j a, j2 n th s l n + 1 j2 a n a( a+1)(2a+1) k = ( n + 1 j2 ) = ( n + 1) V vy 6


k=1 j =1

k=1

7.36. Cho C l mt ng trn tip xc vi cc cnh ca gc xOy v C1 cng k mt ng trn tip xc vi cc cnh gc v i qua tm ca C. Gi A l giao im th hai ca ng knh ca C1 qua tm ca C vi C1 v gi B l giao ca ng knh vi C. Chng minh rng ng trn tm A i qua B tip xc vi cc cnh ca gc xOy Li gii: Gi T v T1 l tm ng trn C v C1 ; r, r1 ln lt l cc bn knh ca hai ng trn . V ng vung gc TT , T1 T1 v AA ti Ox.

By gi tng: j2 k m c

k = j2 k = j2 k
k=1 j =1 j =1 k=1

j =1

t
n

Gi P l mt mnh , [P] c gi tr bng 1 nu P ng, ngc li P c gi tr 0 nu P sai. Ch rng k l s nguyn dng v bnh phng ca n gn k nht. a k = j2 k V vy:

.c

Tht vy, ta quy c vic s dng du [ ] nh sau:

32

Nguyn Hu in, HKHTN H Ni

Ta c: TT = r, T1 T1 = r1 . V T1 l trung im ca AT nn T1 T1 = AA + TT AA = 2T1 T1 TT = 2r1 r 2

V vy AB = AT BT = 2r1 r. Suy ra pcm.

7.37. Tm tt c cc s nguyn x, y, z tha mn iu kin x2 +y2 +z2 2xyz = 0

Li gii: Ta chng minh nghim duy nht l x = y = z = 0. - Trc ht ta c x, y, z khng th l cc s l v khi tng x2 +y2 +z2 2xyz . l mt s l nn khc 0, do xyz . .2 Mt khc: x2 +y2 +z2 = 2xyz chia ht cho 4 khi tt c cc bnh phng chia ht cho 4 hoc chia cho 4 d 1, x, y, z phi l cc s chn, do ta li vit

Lp li qu trnh ny ta c dy cc s nguyn ( x1 , x2 , ..., xn ) tha mn xi = 2xi +1 . Li c x = 2n xn x. n , n 1 x = 0. .2 Chng minh tng t ta c y = z = 0. Ghi ch: Nu th x = yz - w ta c bi ton USAMO 76/3 7.38. Tm s nguyn k nh nht tn ti hai dy { ai }, {bi }, tha mn: a.ai , bi 1, 1996, 19962, ... , i = 1...k

b.ai = bi , i = 1...k

Tng t cc s xn ,yn ,zn l chn nn ta c th vit xn = 2xn+1, yn = 2yn+1, zn = 2zn+1 tha mn: x2 +1 + y2 +1 + z2 +1 = n n n n +2 x 2 n +1 y n +1 zn +1

2 Hay x2 + y2 + z2 = 8x2 y2 z2 2 2 Tip tc qu trnh ta c: n 1, x2 + y2 + z2 = 2n+1 xn yn zn n n n

2 Thay vo ta c: 4x2 + 4y2 + 4z2 = 32x2 y2 z2 2 2

x1 = 2x2 , y1 = 2y2 , z1 = 2z2

.c

2 Hay x1 + y2 + z2 = 4x1 y1 z1 1 1 V v phi chia ht cho 4 nn nn ta li vit c

x = 2x1 , y = 2y1 , z = 2z1 2 Ta c: 4x1 + 4y2 + 4z2 = 16x1 y1 z1 1 1

thi olympic Korean c.ai ai +1 , bi bi +1 d. ai = bi


i =1 i =1 k k

33

Li gii: Ta chng t s k phi tm l s 1997. Gi s c hai dy { ai }, {bi } tha mn cc iu kin ca bi ton, vi k 1996 T iu kin b. cho ta a1 = b1

V vy khng mt tnh tng qut, gii s a1 < b1 T iu kin a. ta c 0 m < n : a1 = 1996m , b1 = 1996n chia ht cho 1996n

T bi b1 i = 1, 2, ..., (K c.) v mi gi tr bi mt ly tha ca 1996, bi


i =1

k . Do t iu kin d, ai . .1996n

1 + 1 + 2 2 + ... + 2 ( 2) Vi j = {1, 1} , j

n ( 2)n

Tm s phn t ca An v tng ca tt c cc tch ca hai phn t phn bit ca An Li gii: Trc ht ta chng minh b :

7.39. t An l tp tt c cc s thc c hnh thnh t tng:

Vi k = 1997 ta c v d: a1 = a2 = ... = a1996 = 1, a1997 = 19962 b1 = b2 = ... = b1997 = 1996

iu ny mu thun vi m < n. V vy k 1997

Do 1996m+1 = ai = bi b1 = 1996.1996n = 1996n+1


i =1 i =1 i =1

n n 1, 21 + 42 + ... + 2n | i {1, 1} = 2n , jl, | j| < 2n Tht vy Ta chng minh bng phng php quy np. 1 1 Khi n = 1 th c hai tp hp u l 2 , 2 , n 1, j {1, 1}

.c
j

v th t 1996 v t k 1996 V vy ta c t = k = 1996.

i =1

Ta c t.1996m = ai = 0( mod1996m+1 )

Ta k hiu t l mt s jn m a j = 1996m .

i =1

34

Nguyn Hu in, HKHTN H Ni


j 1 2n

n t j = 2n1. 1 + ... + 20 . n , vi j l v 21 + 42 + ... + 2n = 2n n n 1 T 21 + 42 + ... + 2n 21 + 42 + ... + 2n = 2 + 1 + ... + 4

<1

ta c | j| < Khi tp hp v tri cha trong tp hp v phi. 2n


j 1 2 ,

Xt vi j l v | j| < 2n . Khi hoc nguyn lin tip.

hoc

j +1 2

l s l v chng l hai s

tj0 l mt trong hai s l , khi | j0 | Do | j| 2n , v j0 l s l, | j0 | 2n1

| j|+1 2

2n +1

V vy theo gi thit quy np tn ti 1 ,..., n1 sao cho j0 2 n 1 1 2 + 4 + ... + 2n1 = 2n1 t n = j 2j0 {1, 1}
n n 1 Khi 21 + 42 + ... + 2n1 + 2n = Vy b c chng minh.

j0 2n 1

j2j0 2n

j 2n

T b ta c: An = 1 +

2[n/2]

By gi ta c th ghp i phn t 1 +

By gi nu X, Y l hai tp cc s thc hu hn vi x = y = 0, ta c

j 2 2 + 2k[n/2] v1 2[n/2] 2k[n/2] 2[n/2] V th trung bnh ca cc phn t ca An l 1; t a = | An | = 2n j a An xX y2

a,b An ,a< b

a An

a An

j v k l. V vy An cha phn t 2 2 2 = 2n tnh tng tt c cc phn t khc nhau ca An ta s dng cng thc:
2

2 + 2k[n/2] j, k, | j| 2

n/2

ab =

1 2

a2

.c

, |k | 2

n/2

T ba s hng khc khng theo gi thit ta c tng: j2 = 1 2m((2m)2 1) 3


jl?,| j|<2m

x X y Y

( 1 + x + y ) = | X | |Y | + |Y |

x2

xX

+ |X|

y Y

y Y

d dng chng minh c bng php quy np theo m. V th


j 2[n/2] j 2[n/2] 2 + 2k[n/2]

a2 =
a An

jle | j|2
n/2

kle | k|2
n/2

1+ 1+
n/2

a An

a2

jle | j|2
n/2

kle | k|2

k 2 2[n/2]

thi olympic Korean a2 = 2 n +


a An 1 3

35
n/2

n/2 2

(22 n/2 1) 22 n/2

+2

n/2 2

n/2

(22

n/2

n/2 1

1)

a An 2

a2 = 2 n + 1 1

Ch n chn v l khc nhau bc n cui cng, do : ab =


a,b An ,a< b 1 2

7.40. Cho tam gic nhn ABC vi AB khc AC. Gi V l giao im ca ng phn gic gc A vi BC v gi d l chn ng cao h t A ti BC. Gi E v F l cc giao im phn bit ca ng trn ngoi tip tam gic AVD vi CA v AB,chng minh rng 3 ng thng AD, BE, CF ng quy.

a An

1 a2 = 2 (22n 2n+1 + 1) a An

CEV = ta c tam gic BFV v tam gic BDA ng dng; tam gic CEV 2 v CDA cng ng dng. AB AC V vy BD = VB ; CD = VC BF CE AB AC Nhng VB = VC ( theo nh l ng phn gic) V th BD = CD BF CE

.c

Li gii: T ADV =

v A, D,V, E, F nm trn ng trn, BFV =

3 v tr. chng minh rng s t khc nhau gia x v y khng qu 5 v tr l 38. Li gii: Khng mt tnh tng qut gi s x=00000000, y=00000111. Gi z l mt t khc gia x v y m c t nht 5 v tr khc nhau nu v ch nu a + b 5 vi a l ch s 1 u trn trong 5 ch s khc nhau ca z v b l ch s 1 cui cng trong 3 ch s ca z. Hn na ta c bt ng thc 2a

7. iu phi chng minh l (4,1),(4,2 ) v (5,b) vi {0, 1, 2, 3} Hai iu u tin cho 5 3 3 + = 30 4 1 2

7.41. Mt t l mt dy gm ch s 0 hoc 1. t x v y l 2 t khc nhau ng

ng quy theo nh l Ceva.

Li c FAV = VAE, AE = AF, t

BD CE AF DC . EA . FB

BD BF CD CE

= 1 v AD,BE,CF

36

Nguyn Hu in, HKHTN H Ni

t cho z v cc trng hp khc cho 23 = 8. V th c 38 t khc nhau gia x v y m c 5 v tr khc nhau. 7.42. Tm tt c cc cp hm s f,g:R R tha mn a. Nu x<y th f(x) < f(y) b. x, y R, f ( xy) = g(y). f ( x ) + f (y) xm , x 0 Li gii: Xt cp (f,g ) cho bi: f ( x = t(1 g(t))) g( x ) = | x|m , x < 0

t t = f (0), ta c f (y) f (y) = t(1 g(y)). Do f l hm ng bin nn ta khng c t = 0. Thay cng thc ny cho f b cho ta t(1 g( xy)) = g(y)t(1 g( x )) + t(1 g(y)). T ta c: 11 g( xy) = g(y)(1 g( x )) + 1 g(y) = 1 g( x ) g(y)

vi t < 0, m > 0 l nghim duy nht. t x = 0, t b ta c: f (0) = f (0) g(y) + f (y), suyra f (y) = f (0)(1 g(y)).

t m = h(1), t ta c m > 0 v h l hm tng v g( x ) = x m , x > 0 Ta c g(1) < 0, m ( g(1))2 = g(1) = 1 nn g(-1) = 1. xm , x 0 T g(-x) = -g(x) nn g( x ) = | x |m , x < 0 Ta cng c f(x) = t(1-g(x)). D dng kim tra c rng cp (f,g) l nghim

yi > x v s dng tnh n iu chng t rng h(x) = x.h(1), x R

h(0) = log( g(e0 )) = 0 v h n iu ngt. Ta cng c h(x+y) = h(x) + h(y) suy ra h(nx) = n.h(x) vi n N v h(-x) = -h(x), do h(al phax) = .h(x) vi Q. Xt dy hu t {yi } , {yi } xi < x

Ta c g l hm tng nn f cng tng, ta c t < 0..V g( x ) > 0, x > 0. Gi h: R R vi h = log.g.exp. Ta ch( x + y) = log( g(e x+y )) = log( g(e x ) g(ey )) = log( g(e x )). log( g(ey )) = h ( x ) + h (y )

.c

hoc g(xy) = g(x)g(y) vi mi x, y thuc R. f T g = 1 t l hm n iu ngt, v vy g(1) = 0 Nhng li c g(1) = g2 (1), v th g(1) = 1.

thi olympic Korean

37

m > 0 > t.

7.43. Cho a1 , a2 , ...,an l cc s dng v k hiu: A =


1 n

a1 +...+ an n

G = (a1 ...an ) H = 1 n 1
A A a. Nu n chn, chng minh rng H 1 + 2( G )n 2( n 1) A A b. Nu n l, chng minh rng: H n2 + n ( G )n n n 1 a1 +...+ an

Li gii: Ch rng theo bt ng thc Mclranh ta c:


G H

a1 ...an ( a1 1 +...+ a1 ) n n

1 n

j =1

a1 ...an aj

1 n

j =1

aj

= A n 1

V th

.c

A ( G )n A A A A T A G, ( G )n 1 nn H ( G )n 1 + 2( G )n a. A A A A Vi b, H ( G )n ( G )n + n2 (( G )n 1) n A A H n2 + 2(nn 1) ( G )n n Ta c pcm.

A H

Chng 8 thi olympic Poland


8.44. Cho cc s nguyn x1 , x2 , ..., x7 tha mn iu kin x6 = 144, xn+3 = xn+2 ( xn+1 + xn ); n = 1, 2, 3, 4. Li gii: Nhn phng trnh cho vi n = 1, 2, 3 v kh nhn t chung ta c

144 = x6 x5 ( x4 + x3 ) 2x2 (3x3 ) 144 6x3 x3 = 1, 2. 3 3 + Xt trng hp 1: x3 = 1 T [1], 144 = ( x1 + x2 )( x2 + 1)( x1 + x2 + 1). Nhn c 2 v vi 144 lin tip cc cp s nguyn (1, 2), (2, 3), (3, 4), (8, 9). T x1 + x2 v x1 + x2 + 1 vi h s 144 cc s nguyn lin tip v khi x1 + x2 2. Ta c 3 trng hp:

1a. x1 + x2 = 2 6( x2 + 1) = 144 t suy ra x2 = 23, x1 = 21. Tuy nhin n khng tha mn, vy xi l s nguyn xc nh.

c th xy ra: x4 = 8, x5 = 16, x6 = 144. Nh vy th ( x1 , x2 , x3 ) = (7, 1, 1)

1b. x1 + x2 = 3 12( x2 + 1) = 144 t suy ra x2 = 11, x1 = 8. Khng tha mn. 1c. x1 + x2 = 8 72( x2 + 1) = 144 t suy ra x2 = 1, x1 = 7. Th nghim

x4 = x3 ( x2 + x1 ) 2x3 x5 = x4 ( x3 + x2 ) 2x2 3

144 = x3 ( x1 + x2 )( x2 + x3 )( x3 + x4 ).[1] Mt khc t phng trnh cho ta c bt ng thc:

.c

Tnh x7 .

thi olympic Poland ta c x7 = 3456 + Xt trng hp 2: x3 = 2

39

144 = 2( x1 + x2 )( x2 + 2)(2x1 + 2x2 + 2) 36 = ( x1 + x2 )( x2 + 2)(x1 + x2 + 1). Nhn c 2 v vi h s lin tip cc cp s nguyn (1, 2), (2, 3), (3, 4) ta c cc trng hp sau: 2a. x1 + x2 = 2 x2 = 4, x1 = 2. (Khng tha mn)

2b. x1 + x2 = 3 x2 = 1, x1 = 2. Ta th cc nghim sau: x4 = 6, x5 = 18, x6 = 144. Nh vy vi ( x1 , x2 , x3 ) = (2, 1, 2) th x7 = 3456. Vy gi tr x7 cn tm l 3456. 8.45. Gii h phng trnh sau vi x, y, z l cc s thc: 3 ( x 2 + y 2 + z2 ) = 1 x2 y2 + y2 z2 + z2 x2 = xyz( x + y + z)3

.c

Vi 3 s thc a, b, c ta c: (a b)2 + (a c)2 + (b c)2 0 hay a2 + b2 + c2 ab + ac + bc, du "=" xy ra nu v ch nu a=b=c.

Li gii: Ta c x, y, z hoc (x+y+z) khng th bng 0 v xyz( x + y + z) = x 2 y2 + y2 z2 + z2 x 2 0 ( x + y + z )2

8.46. Bi 3: Trong t din ABCD, gia cc mt ABD, ACD, BCD, t nh D

mt khc.

Li gii: Cho 900 l 1 gc, u tin ta chng minh = 900 . Nu a, b, c tng ng l di 3 cnh BC, CA, AB v m a , mb , mc tng ng l 3 ng trung tuyn ng vi 3 cnh BC, CA, AB th din tch ca tam gic DAB l 1 mc c sin . Ch rng gi tr tuyt i ca nhng tch gia 2 vect l D A+ B v A-B l mc ccos= 2cot [ DAB] = DA2 DB2 . 2 Chng ta c 3 din tch [ ABD] , [ ACD] , [ BCD] tng ng t l l DA2 DB2 , DA2 DC2 , DB2 DC2 Ta chng minh

nhn cc cnh AB, AC, BC to thnh cc gc tng ng bng nhau. Chng minh rng mi mt tam gic ny c din tch nh hn hoc bng tng ca din tch ca 2

chnh l nghim ca h phng trnh.

Vy 1 = 3( x2 + y2 + z2 ) ( x + y + z)2 = xyz(x+y+z) xyz(x+y+z) = 1 Du "=" xy ra khi v ch khi x = y = z, t suy ra (x,y,z) l 1 1 1 1 1 1 3; 3; 3 ; 3 ; 3 ; 3 .

x 2 y2 + y2 z2 + z2 x 2

xy2 z+ x2 yz+ xyz2

900 .

40

Nguyn Hu in, HKHTN H Ni

ADB, y = BDC, z = ADC (0 < x, y, z < 180). Ch rng 3 gc x, y, z l 3 gc tam din, chng ta c: x + y > z, x + z > y, y + z > x v x + y + z 360. Khi din tch ca tam gic ADB l AD.BD.sinx ( tng t i vi din tch 2 cc tam gic BDC v ADC) v AD=BD=CD( khi = 900 ), chng ta cn chng minh rng: sin x + sin y > sin z xy x+y z z Ta c sin x + sin y = 2 sin 2 cos 2 , sinz = 2sin 2 cos 2
x+y xy z x+y 180 2 2 z > cos 2 . Khi hm cosin z 2

z z sin 2 > sin 2 . Nh vy t 2 < 2 cos 2 gim trong [0, 180] x+y xy z z T suy ra sin 2 cos 2 > sin 2 cos 2 .(pcm)

z z Ta cn chng minh: sin 2 cos 2 > sin 2 cos 2 x+y z T x + y + z 360 2 180 2 . rng 0 < x+y xy xy

10 trong tt c cc on nhng m s cui cng ca Bn cho bi a[ n ] . Nh vy:


2

an = a[ n ] + 1 = a[ n ] + (1)n(n+1)/2 2 2 Tng t vi n = 1, 2(mod4) ta c: a[ n ] 1 = a[ n ] + (1)n(n+1)/2 2 2 Vy ta chng minh xong. Nh vy vi k s nguyn cho chng ta cn tm s ca n s nguyn tha mn 2k n 2k+1 v s ca 00 v 11 on bng s ca 01 v 10 on.

tr i s 01 hay 10 trong Bk . u tin xt trng hp khi n 0, 3(mod4) th Bn gii hn trong 00 hay 11. Nh vy an bng mt s dng ca s 00 hay 11 trong tt c cc on nhng m ch s ca Bn tr i cc s 01 hay

quy np. Gi s an l s hng ca 00 hay 11 on trong Bn tr i s ca 01 hay 11 trong Bn . Vi k=1,2,3,...,n-1, ak l s hng ca 00 hay 11 trong Bk

Li gii: t Bn c ngha l c s 2 i din ca n s. Ta chng minh bng

.c

2k n 2k+1 van = 0

Vi mi s nguyn k 0. Tm n tha mn:

8.47. Cho dy a1 , a2 , ..., xc nh bi : a1 = 0, an = a[ n/2] + (1)n(n+1)/2 , n > 1

thi olympic Poland

41

Ch rng Bn c k+1 s. Vi mi Bn chng ta xy dng dy mi Cn ca 0 s v 1 s ging nh nhau. Bt u t cc s ca Bn v ch s gn cui ca Bn , ta cng thm vo dy s Cn gi tr tuyt i ca s khc gia v ch s bn tri. V d vi B1 1 = 1011 v C1 1 = 110. Khi 00 hay 11 l on trong Bn sinh ra 0 trong dy Cn v 01 hay 10 trong Bn sinh ra 1 trong Cn . Chng ta cn tm thy s ca n s nguyn. Nh vy c iu phi chng minh. 8.48. Cho ng gic li ABCDE vi DC=DE v BCD = DEA = trung im on AB, khi AF = BF FCE = FDE, FEC = BDC.
AE BC . 2.

Cho F l

Chng minh rng:

=
Suy ra ADF = BDC.

AE sin ADE AD = BC sin CDB BD

AG sin DCQ QC sin RBG CD sin QRG sin DBA = = GB DE sin GQR sin BAD sin ERD RG sin GAQ

.c

im thuc DA, DB vi chu vi ca t gic CDEF. Gi G = QC RF ta c:

Li gii: Gi P = EA BC v xt im C, D, E, P. Gi Q, R tng ng l

8.49. Xt n im(n 2) trn ng trn. Chng minh rng s ln nht n ca 3 cung trn vi im cui trong s n im c chiu di ln hn hoc bng 2. Li gii: Xy dng th cho bi cc nh bi mi cp im c khong cch ln hn hoc bng 2 Chng ta s chng minh rng khng tn ti K4 . Gi s tn ti K4 , gi cc nh theo th t l ABCD. Cnh c chiu di ln hn 2 i din cung c di ln hn
2.

Nh vy

mi cung AB, BC, CD, DA nh hn 2pi v chng cng nh hn 2pi, suy ra mu thun. (pcm)

Nguyn Hu in

OLYMPIC TON NM 1997-1998 48 THI V LI GII (Tp 6)


h
NH XUT BN GIO DC

.c

v n m a t h .c o m

Li ni u
th gi lnh lamdethi.sty ti bin son mt s ton thi A Olympic, m cc hc tr ca ti lm bi tp khi hc tp LTEX. ph v cc bn ham hc ton ti thu thp v gom li thnh cc sch in t, cc bn c th tham kho. Mi tp ti s gom khong 51 bi vi li gii.

Rt nhiu on v mi hc TeX nn cu trc v b tr cn xu, ti khng c thi gian sa li, mong cc bn thng cm.

H Ni, ngy 2 thng 1 nm 2010 Nguyn Hu in

51 89/176-05 GD-05

th tham kho li trong [1].

ly. Nhng y l ngun ti liu ting Vit v ch ny, ti c xem qua v ngi dch l chuyn v ngnh Ton ph thng. Bn c

.c

Rt nhiu bi ton dch khng c chun, nhiu im khng hon ton chnh xc vy mong bn c t ngm ngh v tm hiu

M s: 8I092M5

Mc lc
Li ni u ... . . . . .. . . . . .. . . . . .. . . . . .. . . . . .. . . . . .. . . . . .. . . . . Mc lc .. . . . .. . . . . .. . . . . . .. . . . . .. . . . . .. . . . . .. . . . . .. . . . . .. . . . 3 4 5 9 12 16 23 27 34

Chng 7. thi olympic Anh.. . . . .. . . . . .. . . . . .. . . . . .. . . . . .

Chng 6. thi olympic Ukraina .. . . . . . .. . . . . .. . . . . .. . . . .

Chng 5. thi olympic Th Nh K .. . . . . .. . . . . .. . . . . .. .

Chng 4. thi olympic i Loan .. . . . .. . . . . . .. . . . . .. . . . .

.c

Chng 3. thi olympic Ty Ban Nha.. .. . . . . .. . . . . .. . . . .

Chng 2. thi olympic Nam Phi .. .. . . . . .. . . . . .. . . . . . .. . . .

Chng 1. thi olympic Russian.. .. . . . . .. . . . . .. . . . . .. . . . . .

Chng 1 thi olympic Russian


m
1.1. Chng minh rng cc s t 1 n 16 c th vit c trn cng 1 dng nhng khng vit c trn 1 ng trn, sao cho tng ca 2 s bt k ng lin nhau l 1 s chnh phng. Li gii: Nu cc s vit trn 1 ng trn th ng cnh s 16 l s x, y khi 16 + 1 16 + x, 16 + y 16 + 15, suy ra: 16 + x = 16 + y = 25 mu thun. Cc s c th c sp xp trn 1 dng nh sau:

1.2. Trn cnh AB v BC ca tam gic u ABC ly im D v K trn cnh AC v ly im E v M sao cho DA + AE = KC + CM = AB. Chng minh rng gc gia DM v KE bng . 3 Li gii: Ta c: CE = AC AE = AD. V tng t: CK = AM. 2 bin K Xt php quay tm l tm ca tam gic ABC, gc quay 3 thnh M, bin E thnh D, t suy ra iu phi chng minh. 1.3. Mt cng ty c 50.000 cng nhn, vi mi cng nhn tng s ngi cp trn trc tip v cp di trc tip ca anh ta l 7. Vo th 2 mi cng nhn a ra mt s ch dn v gi bn photo ca n cho mi cp di trc tip ca anh ta (nu anh ta c). Mi ngy sau mi

16, 9, 7, 2, 14, 11, 5, 4, 12, 13, 3, 6, 10, 15, 1, 8.

.c

Nguyn Hu in, HKHTN H Ni

cng nhn gi tt c cc ch dn m anh ta nhn c vo ngy hm trc v gi bn photo ca chng cho tt c cp di trc tip ca anh ta nu anh ta c hoc anh ta phi t thc hin nu khng c cp di trc tip. C nh th cho n th 6 khng cn ch dn no a ra. Hay ch ra rng c t nht 97 cng nhn ko c cp trn trc tip. Li gii: Gi s k l s cng nhn ko c cp trn trc tip, vo ngy th 2 s ch dn c a ra nhiu nht l 7k, vo ngy th 3 nhiu nht l 6.7k vo ngy th 4 nhiu nht l 36.7k vo ngy th 5 mi cng nhn nhn c 1 ch dn ko c cp di trc tip, v vy mi cng nhn c 7 cp trn trc tip, mi ngi a ra nhiu nht l 6 ch dn v c nhiu nht l 216.7k/7 cng nhn nhn c ch dn. Chng ta c:

th 2 l 1 th s th 3 l bao nhiu. Li gii: Gi s cui cng l x, x phi l c ca tng tt c cc s l 37x19, v x = 19 v s th 3 phi l c ca 38 khc 1 hoc 19, vy s th 3 l 2. 1.6. Tm cc cp s nguyn t p, q sao cho p3 q 5 = (p + q)2 .

Li gii: Ch c nghim duy nht l (7, 3), u tin gi s c p v q u khng bng 3. Nu chng ng d vi Module 3, v tri th chia ht cho 3 nhng v phi th khng, nu chng khng cng ng d

1.5. Cc s t 1 ti 37 c th c vit trn 1 dng sao cho mi s l c ca tng tt c cc s ng trc n. Nu s u tin l 37 v s

th c ph bi hnh vung m ng cho ca n l AB v tng t i vi tam gic IBC v tam gic ICA.

Li gii: Gi I l giao im ca 3 ng phn gic ca tam gic ABC, v cc gc l nhn nn IAB. IBA < 45 v vy tam gic IAB c

vung K1, K2, K3. Chng minh rng mim trong ca tam gic ABC c th c ph bi 3 hnh vung.

.c

1.4. Cc cnh ca tam gic nhn ABC l cc ng cho ca hnh

50.000 k + 7k + 42k + 252k + 216k = 518k v k 97.

thi olympic Russian

module 3 th v phi chia ht cho 3 nhng v tri th khng v th khng xy ra kh nng ny. Nu p = 3 ta c q 5 < 27, khng c s nguyn t no tha mn. V vy q = 3 v p3 243 = (p + 3)2 ch c nghim duy nht l p = 7. 1.7. (a) a. thnh ph Mehico hn ch giao thng mi xe oto ring u phi ng k 2 ngy trong 1 tun vo 2 ngy oto khng c lu thng trong thnh ph. Mt gia nh cn s dng t nht 10 chic oto mi ngy. Hi h phi c t nht bao nhiu chic oto nu h c th chn ngy hn ch cho mi chic oto. (b) b. Lut c thay i cm mi oto ch 1 ngy trong 1 tun nhng cnh st c quyn chn ngy cm . Mt gia nh hi l cnh st gia nh c quyn chn 2 ngy lien tip khng b cm cho mi xe v ngay lp tc cnh st cm xe oto vo 1 trong nhng ngy khc. Hi gia nh cn t nht bao nhiu xe oto nu h s dng 10 chic mi ngy. (a) Nu n xe oto c s dng, s ngy c s dng l 5n. M mi gia nh s dng t nht l 10 xe nn 7x10 5n v th n 14.

xe khng tha mn. Khi c n ngy xe b cm, mi ngy nhiu bn bn xe c lu thng nhng n 11, < 10. i vi nht l 7 7 n = 12, gia nh cn a ra 2 ngy lin tip cho mi xe trong nhng ngy xe ko b cm lu thng.

1.8. Mt a gic u 1997 nh c chia bi cc ng cho ko ct nhau to thnh cc tam gic. Hy ch ra rng c t nht mt tam gic nhn. Li gii: ng trn ngoi tip a gic u 1997 nh cng l ng trn ngoi tip mi tam gic. V tm ca cc ng trn khng nm

th 7 v ch nht, 2 xe b cm vo ngy ch nht v th 6. (b) 12 xe oto l s xe h cn, u tin chng ta ch ra rng n 11

Trong thc t 14 xe tha mn yu cu ca u bi ton: 4 xe b cm vo ngy th 2 v th 3, 4 xe b cm vo ngy th 4 v th 5, 2 xe b cm vo ngy th 6 v th 7, 2 xe b cm vo ngy

.c

Li gii:

Nguyn Hu in, HKHTN H Ni

trn bt k ng cho no nn n phi nm trong mt tam gic, v th tam gic phi l tam gic nhn. 1.9. Vit cc s t 1 n 1000 trn bng, Hai ngi chi ln lt xa i 1 s trong cc s , cuc chi kt thc khi cn li 2 s: Ngi chi th 1 thng nu tng cc s cn li chia ht cho 3, cc trng hp cn li ngi chi th 2 thng. Ngi chi no c chin thut chin thng. Li gii: Ngi chi th 2 c chin thut chin thng, nu ngi chi th 1 xa i s x, ngi chi th 2 xa i s 1001 x v th tng

ca 2 s cui cng l 1001.

By gi cc cp nng nht v nh nht to thnh 4 nhm, khng cn nng ca nhm no gp 2/3 ln nhm khc v 3 e f g h l cc cp th e + h 3e (f + g) 2 3 f + g 2e + h (e + h). 2

b + c 3a + d 2a + 2d.

a b c d th a + d 4a 2b + 2c

ht. Ch rng khng cp no nng hn 2 ln cp khc. Nu a, d v b, c l 2 nhm vi

tng i mt qu nh nht vi qu nng nht, sau li em 1 qu nh nht tip theo vi 1 qu nng nht tip theo tip tc cho n

.c

1.10.C 300 qu to, khng c qu no nng hn 3 ln qu khc. Hy ch ra rng c th chia cc qu to ny thnh 4 nhm m khng c 11 ln nhm khc. nhm no c cn nng hn 2 Li gii: Sp xp cc qu to tng dn theo trng lng, v ghp

Chng 2 thi olympic Nam Phi


m
2.11.Cho A0 B0 C0 v mt dy A1 B1 C1 ,A2 B2 C2 ... c xy dng nh sau: Ak+1 , Bk+1 , Ck+1 l im tip xc ca ng trn ngoi tip Ak Bk Ck vi cc cnh Bk Ck ,Ck Ak ,Ak Bk theo th t. (a) hy xc nh Ak+1 Bk+1 Ck+1 t Ak Bk Ck (b) chng minh: lim Ak Bk Ck = 600
k

Ak .Tng t ta c: Ck Bk+1 Ck+1 = 900 Ck . 2 2 (Ak +Ck ) Bk 0 = 90 2 2 0 (b) Ta c Bk+1 600 = 900 Bk 600 = Bk 60 2 2 0 V Bk 600 = B0 60 Hin nhin lim Bk = 600 (2)k k

(a) Ta c Ak Bk+1 = Ak Ck+1 (v y l 2 tip tuyn xut pht t mt im).V vy Ak+1 Bk+1 Ck+1 l tam gic cn vi Ak Bk+1 Ck+1 =

Li gii:

.c

2.12.Tm tt c cc s t nhin tho mn:khi chuyn ch s u tin xung cui,s mi bng 3,5 ln s ban u. Li gii: Cc ch s nh th c dng sau: 153846153846153846. . . 153846 Hin nhin nhng s tho mn gi thit phi bt u bi 1 hoc 2.
7 Trng hp 1: S c dng: 10N + A viA < 10N .V 2 (10N + A) =

900

Hn na Bk+1 =

10 10A + 1 A =
(7.10N 2) 13

Nguyn Hu in, HKHTN H Ni

Trng hp 2: S c dng 2.10N + A, A < 10N .Theo chng minh N trn A = (14.10 4) .Nhng v A < 10N ,tc 10N < 4.V l. 13 2.13.Tm tt c cc hm: f : Z Z sao cho: f (m + f (n)) = f (m) + n, m, n Z Li gii:

Ta c 10N 1, 3, 4, 9, 10, 12(mod13).V th A s l mt s nguyn t nu: N 5(mod6).T ta c kt qu nh trn.

R rng: f (n) = kn vi k = 1, k = 1.Ta s chng minh y l kt qu duy nht.Cho n = 0 ta c f (m + f (0)) = f (m).Xt 2 trng hp: Trng hp 1: f (0) = 0.cho m = 0 c f (f (n)) = n.Gn f (n) bi n ta c: f (m + f (f (n))) = f (m + n) = f (m) + f (n) f (n) = nf (1) v n = f (f (n)) = n(f (1))2 f (1) = 1.y l kt qu trn. Trng hp 2: f (0) = 0. f (n) l hm tun hon v b chn .t f (M) f (n), n.nhng f (M + f (1)) = f (M) + 1.Mu thun gii thit.

Li gii: Cu hi l tm gi tr nh nht ca

gia PQ vi mt phng nm ngang.Hy ch ra v tr im Q sao cho thi gian di chuyn t P n Q l t nht.


PQ sin

P n im Q trn ng trn di nh hng ca trng lc.Khong cch i t P trong thi gian t l: 1 gt2 sin vi g khng i v l gc 2

2.14.Cho mt ng trn v mt im P pha trn ng trn trong mt phng to .Mt ht nh di chuyn dc theo mt ng thng t

.c

hoc gi tr ln nht ca

sin . PQ

Biu din mt php nghch o im P vi nh l qu o ca chnh n.im cc i trn s c(gi l Q) s vch ra mt im Q vi gi tr ln nht P Q sin , vi cao khc nhau gia P v Q .Nh vy P l im pha trn ng trnQ l im pha y ng trn. tm Q hy ch rng P,Q,Q cng thuc mt ng thng.Do cch tm nh sau: (a): Tm im pha y ng trn gi l Q (b): Tm giao ca P Q vi ng trn , l im cn tm.

thi olympic Nam Phi

11

2.15.C 6 im c ni vi nhau tng i mt bi nhng on mu hoc mu xanh..chng minh rng:C mt chu trnh 4 cnh cng mu. Li gii: Gi cc im l A,B,C,D,E,F.D thy lun c mt chu trnh tam gic cng mu(Tht vy:Xt mt s nh no ,s c 5 cnh t cc nh y m t nht 3 trong s chng cng mu.gi s l mu v cc cnh ny i n A,B,C.Nu mt s cnh gia A,B,C l mu ta c iu phi chng minh,nu khng ta cng c iu phi chng minh) Khng mt tnh tng qut,gi cc cnh AB,BC,CA l mu .Nu mt trong cc nh ca cnh mu khc chy n A,B,C,ta c iu cn tm.Nu 2 trong 3 im D,E,F ca cnh mu xanh chy n 2 trong s cc nh cng mu A,B,C,ta cng c iu cn tm.Trng hp duy nht khng xy ra l nu mt trong cc im D,E,F ca

mu xanh th DCEF mu xanh.Ta chng minh c bi ton.

DE,EF,FD.Nu mt cnh trong s chng mu ta c dng hnh trn(v d nu DE mu th DABE l )Nu mt trong s chng

.c

cnh mu to bi nhng im khc A,B,C;khng mt tnh tng qut gi s AD,BE,CF l mu .Cc cnh khng theo l thuyt l

Chng 3 thi olympic Ty Ban Nha


m
3.16.Tnh tng bnh phng ca 100 s hng u tin ca mt cp s cng, vi gi thit tng 100 s hng bng 1 v tng cc s hng th hai, th t, ..., th mt trm bng 1 Li gii: Gi 100 s hng u tin ca cp s cng l x1 , x2 , x3 , , x100 v d l cng sai ca cp s cng theo gi thit th nht ta c: x1 + x2 + x3 + + x100 = Theo gi thit th 2 ta c

Suy ra d =

3 50

ta tm c:

1 v x1 + x100 = x1 + (x1 + 99d) = 50 hay x1 = 149 . T 50

x2 + x2 + + x2 = 100x2 + 2dx1 (1 + + 99) + 12 + + 992 . 1 2 100 1 Vy x2 + x2 + + x2 = 100 1 2 14999 50

3.17.A l mt tp gm 16 im to thnh mt hnh vung trn mi cnh c 4 im. Tm s im ln nht ca tp A m khng c 3 im no trong s cc im to thnh mt tam gic cn

x2 + x4 + + x100 =

1 1 ((x1 + d) + x100 ) .50 = 1 x1 + x100 + d = . 2 25

1 1 (x1 + x100 ) .100 = 1 x1 + x100 = . 2 50

.c

thi olympic Ty Ban Nha

13

Li gii: S im ln nht cn tm l 6 c c bng cch ly cc im hai cnh k nhau nhng b i im chung ca hai cnh . u tin gi 4 im bn trong khng c chn, nhng im cn li to thnh 3 hnh vung, nn nhiu nht 2 ng thng ng t mi hnh vung c chn. Nh vy chng ta c th cho rng mt trong s cc im trong c chn trong s sau l im O D A1 A2 A3 C Z1 O Z2

E B1 B2 B2 C D C E Khng c im no cng tn gi A, B, C, D, E c chn, vy nu ta khng chn Z1 , Z2 mt ln na nhiu nht 6 im c th c chn. Nu chn Z1 nhng khng chn Z2 th A1 , A2 , B1 , B2 cng khng c chn, v c A3 v B3 cng khng c chn, v vy mt trong hai im A v B phi b i, mt ln na s im ln nht l 6. Trng hp chn Z2 nhng khng chn Z1 tng t. Cui cng nu Z1 v Z2 c chn th c Ai v Bi u khng c chn, v vy s im ln 3.18.Vi mi Parabol y = x2 + px + q ct hai trc ta tai 3 im phn bit, v mt ng trn i qua 3 im . Chng minh rng tt c cc Li gii: Tt c cc ng trn u i qua im (0, 1). Gi s (0, q) , (r1 , 0) , (r2 , 0) l 3 im m Parabol qua, do r1 + r2 = p.

Gi s (x a)2 + (y b)2 = r 2 l ng trn lun i qua 3 im trn p do a = 2 v p 1 2 p + (q b)2 = r 4 2


2

ng trn u i qua mt im c nh

nht l 6.

+ b2 =

hay q 2 2.qb = q do b = q+1 , khi im i xng vi im (0, q) 2 qua ng knh nm ngang l im (0, 1) 3.19.Cho p l s nguyn t. Tm tt c k Z sao cho nguyn dng. k 2 pk l s

.c

1 (r1 r2 )2 + b2 4

14 Li gii: Gi tr k cn tm l k =

Nguyn Hu in, HKHTN H Ni


(p1)2 4

vi p l s l ( tr trng hp

p =2 ). Trc ht xt p = 2, trong trng hp ny ta cn k 2 2k = (k 1)2 1 l mt s chnh phng dng trng hp ny khng th xy ra v ch c duy nht hai s chnh phng lin tip l 0 v 1 Gi s p l s l. u tin ta xt trng hp k chia ht cho p, hay k = np, khi k 2 pk = p2 n (n 1), n v n 1 l hai s nguyn t lin

p = m2 n2 = (m + n) (m n). Suy ra m + n = p, m n = 1 v k = 2 hoc m + n = 1, m n = p v k = (p1) 4

l hai s nguyn t cng nhau. k 2 pk l s chnh phng khi v ch khi k v k p l cc s chnh phng, k = m2 , k p = n2 . Do

tip. Do c hai khng th l s chnh phng. Gi s k v p l hai s nguyn t cng nhau, khi k v k p cng
(p+1)2 , 4

i vi trng hp cnh ta s dng cng thc tnh din tch A = (s a) (s b) (s c) (s d) abcd cos2 vi s =
a+b+c+d , 2

khi v ch khi di hai ng cho bng nhau v vung gc vi nhau.

cho. T gi thit cho din tch t gic bng 1 suy ra d1 d2 2, p dng bt ng thc AG-GM suy ra d1 + d2 2 2, ng thc xy ra

li ln hn hoc bng 4 v tng di cc ng cho ca t gic li ln hn hoc bng 2 2. i vi trng hp ng cho ta s dng cng thc tnh din tch A = 1 d1 d2 sin , vi l gc giu hai ng 2

.c

Li gii: Thc t ta cn ch ra rng tng di cc cnh ca t gic

3.20.Chng minh rng trong tt c cc t gic li c din tch bng 1, th tng di cc cnh v cc ng cho ln hn hoc bng 2 2+ 2

B+D , 2

B v D l hai gc i din nhau. T gi thit cho

din tch t gic bng 1 ta suy ra (s a) (s b) (s c) (s d) 1, li s dng bt ng thc AG-MG ta li suy ra 4 (s a) + (s b) + (s c) + (s d) = a + b + c + d , ng thc xy ra khi v ch khi a = b = c = d.

thi olympic Ty Ban Nha

15

T ta suy ra kt lun, c hai ng thc xy ra khi v ch khi t gic li l hnh vung. 3.21.Lng gas chnh xc mt chic t hon thnh mt vng ng ua c t trong n bnh gas t dc ng ua. Chng minh rng c mt v tr m xe c th bt u vi mt bnh gas rng, c th hon thnh mt vng ng ua m khng s ht gas ( gi s xe c th cha mt lng gas khng gii hn) Li gii: Ta s dng phng php qui np theo n, trng hp n = 1 d dng thy c. Cho n + 1 bnh cha phi c mt bnh cha A m t t c th ti c bnh cha B m bnh khng c gas cho mt vng ua. Nu chng ta dn bnh B vo bnh A v b bnh B i, theo gi thit quy np c 1 im xut pht m xe c th hon thnh vng ua, cng im xut pht nh th cho hon thnh vng ua vi lng phn pht ban u ca bnh cha.

.c

Chng 4 thi olympic i Loan


4.22.Cho a l mt s hu t, b, c, d l cc s thc v f : R [1; 1] l 1 hm tha mn: f (x + a + b) f (x + b) = c[x + 2a + [x] 2[x + a] [b]] + d vi mi x R. Chng minh rng hm f tun hon, tc l tn ti s p > 0 sao cho f (x + p) = f (x) vi mi x R. Li gii: Vi mi s nguyn n ta c: f (x + n + a) f (x + n) = c [x b + n] + 2a + [x b + n] 2 [x b + n + a] [b] + d = c [x b] + n + 2a + [x b] + n 2 [x b + a] + n [b] + d = c [x b] + 2a + [x b] 2 [x b + a] [b] + d = f (x + a) f (x) Ly s nguyn dng m sao cho am l mt s nguyn. Khi vi mi s t nhin k ta c:

.c

thi olympic i Loan f (x + kam) f (x)


x m

17

=
j=1 i=1 m

(f (x + jam + ai) f (x + jam + a (i 1)))

=k
i=1

(f (x + ai) f (x + a (i 1)))

= k (f (x + am) f (x)) Do f (x) [1; 1], f (x + kam) f (x) b chn nn f (x + kam) f (x) phi bng 0. Suy ra f (x + kam) = f (x), v vy f (x) l hm tun hon. 4.23.Cho on thng AB. Tm tt c cc im C trong mt phng sao cho tam gic ABC sao cho ng cao k t A v trung tuyn k t B c di bng nhau.

tho mn ai1 + ai+1 = ki ai vi dy k1 , k2 , ..., kn l dy s nguyn dng bt k. (trong a0 = an v an+1 = a1 ). Chng minh rng 2n k1 + k2 + ... + kn 3n Li gii: Bt ng 2n k1 + k2 + ... + kn

4.24.Cho s nguyn n 3, gi thit rng dy s thc dng a1 , a2 , ..., an

hp c bit, khi tam gic ABC cn ta thy rng mi ng trn u c bn knh AB v tm sao cho ABQ = 2/3 (khng c hng).

im C bao gm hai ng trn bng nhau, ct nhau ti B, P (Mt tng ng vi gc /6 v mt tng ng vi gc /6 ). Trong trng

BE//P C, v th P CB = EBC v khng i. Qu tch tt c cc im C sao cho P CB khng i l mt ng trn. Do , qu tch cc

Khi EF//AD v E l trung im ca AC, v vy EF = 1/2(AD) = 1/2(BE) v EBC = /6 (tt c cc gc u c hng tr khi c ). By gi, cho P l mt im sao cho B l trung im ca AP . Khi

.c

Li gii: Gi D l chn ng cao k t A v E l chn ng trung tuyn k t B. Gi F l chn ng vung gc k t E xung BC.

18

Nguyn Hu in, HKHTN H Ni

c chng minh da vo AM-GM vi ch rng:


n

k1 + k2 + ... + kn =
i=1

ai ai+1 + ai+1 ai

minh k1 + k2 + ... + kn 3n 2 vi n 2 , bng phng php quy np theo n. Vi n = 2, nu a1 a2 th 2a2 = k1 a1 , v th hoc a1 = a2 v k1 + k2 = 4 = 3.2 2, hoc a 1 = 2a2 v k1 + k2 = 4 = 3.2 2. Vi n > 2, ta c th gi thit tt c cc ai khng bng nhau, khi tn ti i sao cho ai aa1 , ai+1 m du bng khng xy ra t nht mt trong hai

chng minh c bt ng k1 + k2 + ... + kn 3n, ta cn chng

trng hp. Khi ai aa1 + ai+1 < 2ai v do ki = 1. Ta kt lun rng dy m b i s hng ai cng tha mn iu kin cho vi ki1 v

Ngc li, nu k l s nguyn t 3 3(k1)/2 k 1

mt khc (k 1) l c ca d nn d = k 1 (bi vi d phi nh hn k). Do , k l s nguyn t.

vi 3(k1)/2 1( mod k). V vy 3k1 1( mod k). Vi d sao cho 3d 3 mod k. V vy, d khng l c ca (k 1)/2 nhng l c ca k 1,

Li gii: Gi s k l mt c ca 3(k1)/2 + 1. iu ny tng ng

rng k l mt s nguyn t khi v ch khi k l mt c ca 3(k1)/2 + 1.

.c
3 k

4.25.Cho k = 22 + 1 vi n l s nguyn dng bt k. Chng minh


n

u nh hn hoc bng 3n 2, do ta c iu phi chng minh.

m
=

ki+1 gim i 1 n v v b i s hng ki . Theo gi thit quy np, tng ca ki s hng nh hn hoc bng 3(n 1) 2, tng ca cc s ki ban

4.26.Cho t din ABCD. Chng minh rng: (a) Nu AB = CD, AD = BC, AC = ABC, ACD, ABD, BCD l cc tam gic nhn.

k 3

2 3

= 1

BDth cc tam gic

(b) Nu cc tam gic ABC, ACD, ABD, BCD c cng din tch th AB = CD, AD = BC, AC = BD Li gii: (a) Theo gi thit 4 mt ca t din bng nhau, ta c gc tam din mi nh c to bi ba gc khc nhau ca mt mt. Gi M l trung im ca BC. Theo bt ng thc trong tam gic,

thi olympic i Loan

19

AM + MD > AD = BC = 2MC. Cc tam gic ABC v DBC l bng nhau, v th AM = DM. Do 2MD > 2MC; ngha l, MD ln hn bn knh ca ng trn nm trn mt phng BCD vi ng kinh BC. Do , D nm ngoi ng trn v gc BDC l gc nhn. Tng t nh vy, ta chng minh c cc gc cn li (bi ton ny l bi ton USAMO thng 2/1972, tham kho cc cch gii khc quyn sch USAMO ca Klamkin). (b) V AB v CD khng song song (v ABCD)l hnh t din), ta c th chn hai mt phng song song l mt phng (P ) cha AB v (Q) cha CD. Gi khong cch gia mt phng (P ) v (Q) l d. Gi A , B ln lt l hnh chiu ca A, B trn (Q), gi C , D ln lt l hnh chiu ca C, D trn (P ). V cc tam gic ACD v BCD c cng din tch v chung y CD, nn chng c cng chiu cao h. Ta v hnh

CD, qua php chiu, mt phng (Q) bin thnh mt phng (P ), ng thng AB chia i on thng C D . AB v C D ct nhau ti trung im mi ng nn AC BD l hnh bnh hnh, do ta c AC = BD (gi khong cch gia chng l x) v BC = AD (gi khong cch gia chng l y). Vy ta c: AC = AC 2 + C C 2 = BD 2 + D D 2 AC = BC 2 + C C 2 = AD 2 + D D 2
1/2 1/2

l (h2 d2 )1/2 . V vy, ng thng C D chia i on thng AB. Tng t, ta chng minh c ng thng A B chia i on thng

mt ng thng i qua A v mt ng thng i qua B. Hai ng thng song song vi nhau v cch u ng thng C D mt khong

tr ti mt hoc hai ng thng song song vi CD. A v B khng th cng nm trn mt trong hai ng thng ny, v th hnh tr v mt phng (P ) s giao nhau ti hai ng thng,

.c

tr vi trc CD v bn knh h; r rng A, B thuc hnh tr ny. Hai im ny cng thuc mt phng (P ) v mt phng (P ) giao vi hnh

= x2 + d2 = BD = y 2 + d2 = AD

1/2

1/2 1/2

1/2

20

Nguyn Hu in, HKHTN H Ni

S dng hai cp cnh khc lc u, ta cng c AB = CD 4.27.Cho X l mt tp hp cc s nguyn cho bi cng thc: a2k 102k + a2k2 102k2 + ... + a2 102 + a0 Vi k l mt s nguyn khng m v a2i {1, 2, ..., 9} vi i = 0, 1, ..., k. Chng minh rng, mi s nguyn c dng 2p 3q , vi p, q l nhng s nguyn khng m, chia ht mt phn t no ca X. Li gii: Mi s nguyn khng chia ht cho 10 th u chia ht cho mt s hng no ca X. Ta nhn thy rng lun tn ti mt phn t trong Xc2p 1 ch s l bi ca 4p , vi mi s nguyn p khng

Li gii: Hm f nh trn tn ti vi k = 0 v k = 1. Ta ly a = b, thay vo (b) ta c f (a2 ) = (k + 2)f (a). p dng hai ln ta c: f (a4 ) = (k + 2)f (a2 ) = (k + 2)2 f (a) Mt khc ta c: f (a4 ) = f (a) + f (a3 ) + kf (a) = (k + 1)f (a) + f (a3 ) = (k + 1)f (a) + f (a) + f (a2 ) + kf (a) = (2k + 2)f (a) + f (a2 ) = (3k + 4)f (a)

(a)f (1997) = 1998 (b) Vi mi a, b N, f (ab) = f (a) + f (b) + kf (gcd(a, b))

4.28.Xc nh tt c cc s nguyn dng k tn ti mt hm f : N Z tha mn:

10d+1 1. Bng h qu Euler ca nh l Fermat, 10f k ( mod f k) . V vy m 10(d+1)(f k) 1 / 10d+1 1 chia ht cho 2p k v thuc X

vi k l. Khi vi b bn trn ta c th tm ra bi s ca 2p trong tp hp X. Gi m l bi s v d l s ch s ca m v f =

.c

By gi, ta chng minh rng bt k s nguyn n no khng chia ht cho 10 th chia ht cho mt s hng no ca x. Gi n = 2p k

m. iu ny dn ti php quy np theo p: vi p = 0, 1 v nu x l mt bi s vi p = k, th ta c th chn a2k sao cho x + a2k 102k 0( mod 4k+1 ) v 102k 1( mod 4k )

thi olympic i Loan

21

Chn a = 1997 f (a) = 0 suy ra (k + 2)2 = 3k + 4 c nghim k = 0; k = 1. Vi k = 0 ta c: f (pe1 ...pen ) = e1 g (p1 ) + ... + en g (pn ) 1 n vi m l c nguyn t ca 1997, g(m) = 1998 v g(p) = 0 vi mi s nguyn t p = m. Vi k = 1 ta c f (pe1 ....pen ) = g (p1 ) + ... + g (pn ) 1 n 4.29.Cho tam gic ABC nhn vi O l tm ng trn ngoi tip v bn knh R. AO ct ng trn ngoi tip tam gic OBC D, BO ct ng trn ngoi tip tam gic OCA E, v CO ct ng trn ngoi Li gii: Gi D , E , F ln lt l giao im ca AO v BC , BO v CA, CO v AB. Do chng l nh ca D, E, F qua php ca ng trn ngoi tip tam gic ABC, Do OD .OD = OE .OE = OF .OF = R2 . V AO BO CO OD OE OF

vy, bt ng thc ca bi ton tng ng vi: 8

(BCh2 )/(CE h3 ) v CO/OF = (CAh3 )/(AF h1 ). Do ta c: (AF + F B) (BD + D C) (CE + E A) AB.BC.CA = AF .BD .CE AF .BD .CE AF .F B.BD .D C.CE .E A 8 AF .BD .CE F B.D C. A =8 =8 AF .BD .CE Du bng xy ra khi v ch khi AF = F B, BD = D C, CE = E A, hay ABC l tam gic u. t ca X sao cho hai tp Fk bt k c nhiu nht k 2 phn t chung. 4.30.Cho X = 1, 2, ..., n vi n k 3 v Fk l mt tp con gm k phn

Gi h1 , h2 , h3 ln lt l khong cch t O n AB, BC, CA. Khi AO/OD = [AOB]/[BOD ] = (ABh1 )/(BD h2 ), tng t BO/OE =

.c

tip tam gic OAB F . Chng minh rng OD.OE.OF 8R3 .

22

Nguyn Hu in, HKHTN H Ni

Ch ra rng tn ti mt tp Mk ca X c t nht [log2 n] + 1 phn t khng c cha trong mt tp con no ca Fk . Li gii: Nu k log2 n th iu phi chng minh hin nhin ng, v th ta gi s k < log2 n. t m = [log2 n] + 1. Vi mi tp con k 1 s hng ca tp Xthuc ti a mt tp con ca Fk v mi s hng ca Fk bao gm cc tp con k(k 1) phn t. Ta c: n n 1 1 = (Fk ) k nk+1 k1 k Mt khc, chn mt tp con m s hng bt k ca X, s s hng ca Fk l: m (F 1 k) nk+1 k k n k m

iu c th chng minh rng con s sau nh hn 1, v th mt tp con m phn t no khng c cha s hng no ca Fk . m m = 2m v ta cng c th chng minh Hin nhin i k i m 3.2m3 vi m k 3 bng phng php quy np theo m. V k vy ta c: vi n 3. Ta c iu phi chng minh. m 3n 1 <1 nk+1 4 (n k + 1) k

.c

Chng 5 thi olympic Th Nh K


m
2

5.31.Cho tam gic ABC vung ti A, gi H l chn ng cao k A. Chng minh rng tng bn knh cc ng trn ni tip cc tam gic ABC, ABH, ACH bng AH. Li gii: t a = BC, b = CA, c = AB v s =

.c

a1 = , b1 = , an+1 = an bn , bn+1 = an + bn vi mi n 1 C bao nhiu b s thc (, ) tha mn a1997 = b1 v b1997 = a1 ? Li gii: Lu rng a2 + b2 = (2 + 2 ) an + bn , tr = = 0. n+1 n+1 2 2 Chng ta cn + = 1. V vy c th t = cos , = sin , t bng phng php quy np ta ch ra = cos n, = sin n. T c
2

k 1998 b s: (0; 0) v (cos ; sin ) vi = 3998 , k = 1, 3, ..., 3997

5.33.Trong mt hip hi bng , khi mt cu th chuyn t i X c x cu th sang i Y c y cu th, lin on nhn c y x triu la t i Y nu y x nhng phi tr li x y triu la cho i X nu

5.32.Dy s {an } , {bn } c cho bi: n=1 n=1

trn ni tip nhn vi na chu vi, suy ra bn knh cn tm l ab ; a ab ; b ab v tng ca chng l ab = AH a+b+c c a+b+c c a+b+c c

v ACH ng dng vi tam gic ABC vi t s tng ng a/c v b/c p dng cng thc din tch tam gic bng bn knh ng

a+b+c . 2

Cc tam gic ABH

24

Nguyn Hu in, HKHTN H Ni

x > y. Mt cu th c th di chuyn ty thch trong sut ma chuyn nhng. Hip hi bao gm 18 i, tt c cc i u bt u ma chuyn nhng vi 20 cu th. Kt thc ma chuyn nhng, 12 i kt thc vi 20 cu th, 6 i cn li kt thc vi 16,16,21,22,22,23 cu th. Tng s tin ln nht m lin on c th kim c trong sut ma chuyn nhng l bao nhiu? Li gii: Chng ta tha nhn rng s tin ln nht kim c bi khng bao gi cho php mt cu th chuyn n i nh hn. Chng ta cng c th gi k lc bng mt cch khc. Mt i bng c x cu th th c ghi l x trc khi giao dch mt cu th hoc x trc khi nhn mt cu th v s tin m lin on kim c bng tng ca cc s . By gi ta xem xt cc s c ghi bi mt i m kt thc c nhiu hn 20 cu th. Nu s lng cu th ti a ca i trong sut qu trnh chuyn nhng l k > n th cc s

5.34.Ng gic ABCDE li c cc nh nm trn ng trn n v, cnh AE i qua tm ng trn . Gi s AB = a, BC = b, CD = c, DE = d v ab = cd = 1/4. Tnh AC + CE theo a, b, c. Li gii: Nu gi 2, 2, 2, 2 l cc cung chn bi cc cnh a, b, c, d tng ng th: AC = 2 sin ( + ) = Tng t vi CD. Tng qut vi R l bn knh ng trn ngoi tip ng gic, th th a 2 1 b2 b + 4 2 1 a2 4

(20 + 20 + 21 + 20 + 21 + 20 + 21 + 22) 2(20 + 19 + 18 + 17) = 17

S sp xp dn n s tin kim c l ln nht. Trong trng hp , tng l:

Tng t nh vy, tng ca cc s trong i bng kt thc c n < 20 cu th t nht l 20 19 ... (n + 1). V nhng con s ny chnh

xc l nhng con s c c bi vic lun chuyn cu th t i kt thc t hn 20 cu th sang i kt thc c nhiu hn 20 cu th.

.c

k 1 v k xut hin lin tip v b i 2 s th tng s tng ln. V th tng ca cc s trong i bng t nht l 20 + 21 + ... + (n 1).

thi olympic Th Nh K AC 2 + BD2 = 1 m AC = a R2 b2 + b R2 a2

25

Khi , dn n biu thc cha R2 di du cn v ta gii phng trnh i vi R theo cc s a, b, c, d. nguyn dng n v cc s nguyn x1 , x2 , ..., xn , y1 , y2 , ..., ym khng chia ht cho p sao cho:
2 x2 + y1 x2 (modp) 1 2 2 x2 + y2 x2 (modp) 2 3

5.35.Chng minh rng vi mi s nguyn t p 7, tn ti mt s

.....

Li gii: t S = x1 + x2 + ... + xn S dng BT Chebyshevs cho hai dy dy tng). Ta c: x5 i S xi

Vi x1 , x2 , ..., xn l cc s thc tha mn x2 + x2 + ... + x2 = 1. 1 2 n


xi Sxi

x5 x5 x5 1 2 n + + ... + x2 + x3 + ... + xn x3 + x4 + ... + xn + x1 x1 + x2 + ... + xn1

5.36.Cho cc s nguyn n 2. Tm gi tr nh nht ca :

H thc c dng 52n 32n (modp) (ng).Vy ta c iu phi chng minh.

.c

Li gii: Gi n l cp ca 5/3 mod p, v t xi = 3n1i 5i1 , yi = 43n1 5i1 th mi ng d trn l tng ng tr h thc cui cng.

m
v x4 (c hai dy u l i x4 + x4 + ... + x4 1 2 n n

2 x2 + yn x2 (modp) 1 n

x2 xn x1 + + ... + S x1 S x2 S xn

p dng bt ng thc hm li ta c: x2 xn 1 x1 + + ... + S x1 S x2 S xn n1

26

Nguyn Hu in, HKHTN H Ni

p dng bt ng thc gi tr trung bnh ta c: x4 i n Ta c kt lun: x5 1 1 1 i n . 2 = S xi n1 n n (n 1) ng thc xy ra khi x1 = x2 = ... = xn =


1 . n

1/ 2

x2 1 i = n n

.c

Chng 6 thi olympic Ukraina


6.37.Mt li hnh ch nht c t mu theo kiu bn c, v trong mi c mt s nguyn. Gi s rng tng cc s trong mi hng v

mu l s chn.)

Tng cc hng th nht, th ba, . . . (t trn xung), v cc ct th nht, th ba, . . . (t tri sang) bng tng cc s trong cc mu en tr i hai ln tng tt c cc s trong cc mu . V tng ny l s chn nn tng cc s trong cc en l chn.

trn l mu . (V tng tt c cc s trong li hnh ch nht l chn nn iu cn chng minh cng tng ng vi tng cc

Li gii: Gi s cc mu t l v en, trong vung gc tri

.c

tng cc s trong mi ct l s chn. Chng minh rng tng tt c cc s trong en l chn.

28

Nguyn Hu in, HKHTN H Ni

6.38.Tm tt c cc nghim thc ca h phng trnh sau: x +x ++x 1 2 1997 = 1997 x4 + x4 + + x4 = x3 + x3 + + x3 .


1 2 1997 1 2 1997

Li gii: Ta s chng minh h trn ch c nghim x1 = x2 = = x1997 = 1. t Sn = xn + + xn . Theo bt ng thc lu tha trung bnh 1 , 1 1997 S4 1997 v S4 1997 do
1/4 1/4

S3 1997

S1 =1 1997
1/3

Ta xc nh hm f : N N sao cho

x1 = = x1997 = 1

6.39.K hiu d(n) l s l ln nht trong cc c s ca s t nhin n. 2n d(n)

.c

S4 1. 1997 V vy bt ng thc lu tha trung bnh xy ra du bng, ngha l

S4 1997

1/3

Li gii: Ch c mt gi tr ca k l 499499. Vi mi s chn (2a )b, b l s l v a 1, f ((2a )b) = (2a1 )b + (2a )b = (2a1 )b + 2a = (2a1 )(b + 2). b

Do ly tha cao nht ca hai trong f ((2a )b) bng mt na lu tha cao nht ca hai trong (2a )b. V vy, ly lp a ln ca f ti (2a )b c mt s l.
1

"power mean inequality" (trong bn ting Anh), tm dch l "bt ng thc lu

tha trung bnh" (ND)

ln.

Tm tt c cc s k sao cho f (f (. . . f (1) . . .)) = 1997, f c lp k

f (2n 1) = 2n

v f (2n) = n +

vi mi n N

thi olympic Ukraina

29

Ngoi ra, c l ln nht ca f ((2a )b) l b + 2, ln hn c l ln nht ca (2a )b hai n v. V vy sau a ln lp f ti (2a )b, c l ln nht l b + 2a. Do sau a ln lp f ta nhn c s l nn f a ((2a )b) phi bng b + 2a. Sau mt ln lp f ti 2n 1, n 1, ta nhn c 2n . t a = n v b = 1. Ta thy phi lp thm n ln na mi nhn c s l, s l, b + 2a = 2n + 1. V vy phi lp n + 1 ln cho f ti 2n 1 nhn c s l tip theo, 2n + 1. Ta d thy l khng c s l no xut hin hai ln khi lp f ti 1, bi v dy cc s l nhn c t vic lp f ti bt k s nguyn dng no cng l mt dy tng ngt. c bit, tn ti mt gi tr k sao cho f (k) (1) = 1997, v s k l duy nht. By gi ta chng minh bng quy np rng f
n(n+1) 1 2

nn k = 499499 tho mn yu cu bi, v do lp lun trn, k l 6.40.Hai ng gic u ABCDE v AEKP L trong khng gian sao cho DAK = 60o . Chng minh rng hai mt phng ACK v BAL vung gc.

duy nht.

999.1000 1 2

(1) = f 499499 (1) = 1997,

V 1997 = 2(999) 1, ta c

h
E

(k+1)(k+2) 1 2

(1) = f k+1 f

k(k+1) 1 2

(1) = f k+1(2k 1) = 2k + 1

.c
K T A

D thy iu ng ti n = 1. Sau gi s n ng vi n = k,

D O C B

(1) = 2n 1.

m
P L

30

Nguyn Hu in, HKHTN H Ni

Li gii: Nu ta quay AEKP L quanh trc AE, bt u v tr trng nhau vi ABCDE, th gc DAK tng cho n khi AEKP L li nm trn mt phng cha ABCDE. (iu ny c th nhn c bng cch tnh ton thy tch v hng ca vect AD v AK tng trong mt khong xc nh). V vy c mt gc duy nht gia cc mt phng sao cho gc DAK = 60 . Tht vy, iu ny xy ra khi cc hnh ng gic l hai mt ca khi mi hai mt u 2 (do tnh cht i xng, tam gic DAK l u trong trng hp ny). c bit, mt phng BAL l mt phng cha mt th ba qua nh A ca khi mi hai mt u, ta gi mt l BALT O. S dng thm du phy sau tn im k hiu hnh chiu tng ng ca cc im xung mt phng ABCDE. Khi K nm trn ng phn gic ca gc DEA. Lp h trc ta sao cho ABCDE nm trn mt phng Oxy theo chiu kim ng h, v K E cng chiu vi chiu dng ca trc Ox. Xt hai vect EK v EA. Vect th nht khng c thnh phn y, v vect th hai khng c thnh phn z. Do tch v hng ca hai vect bng tch cc thnh phn x ca chng. V AE v K E to vi nhau gc 54 , EA c thnh phn x c tnh bi AE(cos 54 ) = cos 54o . EK c thnh phn x l K E. Do tch v hng ca chng bng K E(cos 54 ). Mt khc EK.EA =

K A.AC = (K E + EA).AC

V BO vung gc vi AC v BO song song vi AT , nn AT vung gc vi AC. V vy cc im K , A, T thng hng. V K, A, T khng thng hng nn mt phng AKT vung gc vi mt phng cha ABCDE. Quay xung quanh A, ta suy ra mt phng ACK vung gc vi mt
2

khi thp nh din u (ND)

Cc tnh ton trn vect sau y s chng t K A v AC vung gc:

= K E AC cos 54 CA AC cos 72 = cos 72 cos 54 cos 72 = 0 cos 54

EK.EA. cos KEA = (1).(1). cos(108 ) = cos 72 . Lp phng trnh t cos 72 hai kt qu trn ta nhn c K E = cos 54

.c

thi olympic Ukraina phng cha BALT O, t c iu phi chng minh.

31

6.41.Cho phng trnh ax3 + bx2 + cx + d = 0 c ba nghim thc phn bit. Hi phng trnh sau c bao nhiu nghim thc: 4(ax3 + bx2 + cx + d)(3ax + b) = (3ax2 + 2bx + c)2

Li gii: t P (x) = ax3 + bx2 + cx + d v Q(x) = 2P (x)P (x) [P (x)]2 ,

ta s m s nghim thc ca Q(x) = 0. Ta c th thc hin mt s php rt gn m khng thay i s nghim thc ca Q(x). Trc tin ta chia P cho mt h s t l sao cho a = 1. Sau ta tnh tin x sao cho nghim ng gia ca P l 0, tc l d = 0 v c < 0. Khi :

Li gii: Ch c mt hm s tho mn l f (x) = x. T (a), f (x + n) = f (x) + n vi mi n nguyn dng. t x = p vi p, q nguyn dng. Ta q c: 2 2 2 p p p p + q2 2 = q+f +f = q + 2qf . f q q q q Mt khc, f p + q2 q
2

=f =f

(p + q 2 )2 q2 q 2 + 2p + p2 q2 = q 2 + 2p + f p q
2

(a) f (x + 1) = f (x) + 1 (b) f (x2 ) = f (x)2 .

6.42.K hiu Q+ l tp tt c cc s hu t dng. Tm tt c cc hm s f : Q+ Q+ sao cho vi mi x Q+ :

tc l Q c tt c hai nghim thc.

du ca b. Trong trng hp no th mi dy trn cng ch c ng mt ln i du. V vy Q c mt nghim m v mt nghim dng,

.c

By gi ta p dng quy tc du Descartes m s nghim ca Q(x). Du ca Q(x) l +, s, , v ca Q(x) l +, s, , , s l

Q(x) = 3x4 + 4bx3 + 6cx2 c2 .

32 T hai phng trnh trn suy ra 2qf

Nguyn Hu in, HKHTN H Ni

p q

= 2p

p p v vy f ( ) = . q q 6.43.Tm s nguyn n nh nht sao cho vi mi n s nguyn tu , tn ti 18 s nguyn trong c tng chia ht cho 18. Li gii: Gi tr nh nht ca n l n = 35; tp hp 34 phn t gm 17 s khng v 17 s mt cho ta thy rng n 35. V vy ta ch cn phi chng minh rng vi 35 s nguyn bt k lun tm c 18 s trong c tng chia ht cho 18. Thc ra ta s chng minh rng vi mi n, trong 2n 1 s nguyn lun tm c n s c tng chia ht cho n. Ta s chng minh khng nh trn bng quy np theo n. D thy

ng trong trng hp ny. (Ch : chng minh bi ton cho, cn phi chng minh trc tip cho trng hp p = 2, 3.) 6.44.Cc im K, L, M, N nm trn cc cnh AB, BC, CD, DA ca hnh hp (khng cn l hnh hp ng) ABCDA1 B1 C1 D1 . Chng minh rng cc tm mt cu ngoi tip ca cc t din A1 AKN, B1 BKL, C1 CLM, D1 DMN l cc nh ca mt hnh bnh hnh. Li gii: a vo h ta vi ABCD song song vi z = 0. Gi E, F, G, H l tm ng trn ngoi tip cc tam gic

p1 cho ng d vi C2p1 1 (mod p). Mt khc, tng ca cc s dng e ae1 app vi e1 + + ep p 1 lun chia ht cho p: nu k p 1 cc 1 pk s ei khc khng th mi tch c lp li C2p1k ln, v cui cng l mt bi ca p. S mu thun chng t khng nh trn l

nu xp1 1 (mod p). V vy nu khng nh trn sai th tng cc s dng (a1 + + ap )p1 trn mi tp con {a1 , . . . , ap } ca cc s

2p 1 s; ta c 2q 1 tp nh vy, v li theo gi thit quy np, c q tp trong s c tng (ca pq s) chia ht cho q. By gi gi s n = p l s nguyn t. S x chia ht cho p nu v ch

.c

iu ng vi n = 1. Nu n l hp s, vit n = pq, ta c th ly ra tp p s nguyn m tng chia ht cho p cho n khi cn li t nht

thi olympic Ukraina

33

AKN, BKL, CLM, DMN v gi W, X, Y, Z l tm mt cu ngoi tip cc t din A1 AKN, B1 BKL, C1 CLM, D1 DMN. Vi mi im Q ta k hiu Q1 , Q2 , Q3 l cc ta x, y, z ca Q. K A B N L D M C

B 1 A1

W1 + Y1 = X1 + Z1 v W2 + Y2 = X2 + Z2 . Ta ch cn phi chng minh W3 + Y3 = X3 + Z3 . Ch rng c W v X u nm trn mt phng vung gc vi ABB1 A1 v i qua trung im ca AA1 v BB1 . Do W3 = aW1 +bW2 +c v X3 = aX1 +bX2 +c vi a, b, c l cc hng s. Tng t, Y v Z u nm trn mt phng vung gc CDD1 C1 v qua trung im ca CC1 , DD1 . V DCC1 D1 song song v bng ABB1 A1 , ta c Y3 = aY1 + bY2 + d v Z3 = aZ1 + bZ2 + d vi d l mt hng s khc, cn a, b l cc hng s cng thc trn. Bi vy W3 + Y3 = X3 + Z3 , t hon thnh chng minh rng W XY Z l hnh bnh hnh.

W v E cch u AKN, W E vung gc vi AKN nn vung gc vi mt phng z = 0. Vy W1 = E1 v W2 = E2 , tng t vi X, Y, Z. Vt

chiu di AB , tng t vi CD. Ta cng c lp lun nh vy vi php 2 chiu trn CD. By gi ta c E1 + G1 = F1 + H1 v E2 + G2 = F2 + H2 . Hn na, v

Nhng r rng vi php chiu trn AB, v E v F chiu tng ng xung trung im ca AK v BK, nn on thng gia chng c

.c

minh iu vi hnh chiu ca cc on thng trn hai phng khc nhau (chng hn, xy dng h trc theo cc phng ).

D 1 Trc tin ta s chng minh EF GH l hnh bnh hnh, bng cch chng minh trung im ca EG v F H trng nhau. Ch cn chng

Chng 7 thi olympic Anh


m
7.45.Gi s M v N l hai s nguyn dng c 9 ch s c tnh cht l nu bt k ch s ca M c thay bi ch s ca N tng ng th ta c mt bi ca 7. Chng minh rng vi bt k mt s t c bng

Li gii: Kt qu ng vi bt k d 2 ( mod 7). Vit M =

Tm mt s nguyn d > 9 sao cho kt qu trn vn cn ng khi M v N l hai s nguyn dng c d ch s.

cch thay mt ch s ca N tng ng bi mt ch s ca M cng l mt bi ca 7.

.c

10k , N =

7.46.Trong tam gic nhn ABC, CF l mt ng cao, vi F trn AB, v BM l mt trung tuyn, vi M trn CA. Cho BM = CF v MBC = F CA, chng minh rng tam gic ABC l u. Li gii: Gi s ACF = CBM = A, v gi s CM = AM = m. Th MB = CF = 2m cos A. Theo nh l hm s Sin, MB CM = , sin CBM sin MCB

nk 10k , y mk , nk l cc ch s. Th vi bt k k, 10k (nk mk ) 0 M ( mod 7). Ly tng theo k, chng ta c M N dM 2M ( mod 7), v vy N M ( mod 7), c 10k (mk nk ) N ( mod 7). Vy khi thay bt k ch s trong N bi ch s tng ng trong M chng ta t c mt s chia ht cho 7.

thi olympic Anh

35

v v vy sin MCB = 2 cos A sin A = sin 2A. iu ny a n hai kh nng. Nu MCB +2A = 180, th CMB = A = MBC. Khi CB = MC v MB = 2MC sin A. Cng c MB = CF = AC cos A = 2MC cos A. Do sin A = cos A v vy A 45

MCB 90 , mu thun. V vy chng ta c MCB = 2A, v vy ACF = BCF . Do tam

gic ACF ng dng vi tam gic CBM, v vy CAF = BCM. Do BC = AB, vy tam gic ABC l u. 7.47.Tm s cc a thc bc 5 vi cc h s khc nhau t tp {1, 2, . . . , 9} m chia ht cho x2 x + 1.

li. Tuy nhin, chng ta m c cc s dng x, x + d, x + 2d sinh i, v c 9 2k s. Do , vi k cho trc, chng ta c


3 C9k (9 2k)(7 k) + 9 3k

a thc. Cng li, c (1+4+10+20+35+56)(42+25+12+3)+(3+6) = 53 a thc dng trn, v 53.12 = 636 cc a thc tt c. 7.48.Tp S = {1/r : r = 1, 2, 3, . . .} cc s nguyn dng nghc o lp thnh cp s cng vi di ty . Chng hn, 1/20, 1/8, 1/5 l mt

s c cc h s p + k, q, r + k, p, q + k, r vi k > 0 v p q r. 3 Vi k cho, c C9k cc gi tr ca p, q, r sao cho r + k 9. Tuy nhin, cc h s phi khc nhau, nn chng ta phi tr i. C 9 2k cch la chn 2 s khc nhau theo k, v 7 k cch la chn cc s cn

a/2 b/2 c d/2 + e/2 + f = 0, hay e + 2f + a = b + 2c + d hoc (v a d = e b) a d = c f = e b. iu ny ko theo 1/12 ca a thc

Ni cc khc, v vy i sin 60(a b + d + e = 0), hoc a d = e b v

.c

ae5i/3 + be4i/3 + cei + de2i/3 + eei/3 + f = 0.

v ch nu

Li gii: Cho phng trnh bc 5 l ax5 + bx4 + cx3 + dx2 + ex + f = 0. Cc nghim ca x2 x + 1 khng phi l cc nghim thc ca x3 + 1, l ei/3 v e5i/3 . Do a thc bc 5 l chia ht cho x2 x + 1 nu

36

Nguyn Hu in, HKHTN H Ni

cp s nh vy, vi di l 3 vi phng sai l 3/40. Hn na, c mt cp s cc i trong S vi di 3 v n khng th m rng thm v bn phi v bn tri ca S (1/40 v 11/40 khng l phn t ca S). (a) Tm mt cp s cc i trong S c di 1996. (b) C hay khng mt cp s trong S c di 1997? Li gii: C mt cp s cc i c di n, vi mi n > 1. Theo nh l Dirichlet ko theo rng c mt s nguyn t p c dng 1 + dn vi mi s nguyn dng d. By gi xt cp s 1+d 1 + (n 1)d 1 , ,..., . (p 1)! (p 1)! (p 1)! V cc mu s chia ht t s, nn mi phn thc l mt s nguyn nghc o, nhng vi (1 + nd)/(p 1)! = p/(p 1)! khng phi v p l

.c

s nguyn t. Do c dy cp s cc i. ( gii (a), n gin ly p = 1997.)

You might also like